Slaying a greenhouse dragon. Part III: discussion

By Judith Curry

In Part I, we critiqued Claes Johnson’s chapters in the book Slaying the Sky Dragon.   In Part II, I have posted a published article by Martin Hertzberg, who authored a chapter in the Dragon book.   My original motivation for doing these threads is to try to lay to rest the debate over the fundamental physics of infrared radiative emission of gases such as CO2 and H2O.

Rather than critique the papers myself (I merely provided an opinion, rather than a substantive critique), I decided to see if “crowd sourcing” the critique would work.  This would only work if very knowledgeable experts showed up and made substantive contributions.  In Part I, the contributions of experts such as maxwell, Pekka Perilla, Tomas Milanovic, Dave N., Eli, and several others were particularly illuminating, and they did a much better job that I would have been able to do myself.  Lets see how Part II turns out.

Claes Johnson engaged in Part I with a large number of comments, although nearly everyone whom he replied to seemed to think that his comments were not responsive to the criticisms (some of which were quite devastating to his argument).  Ken Coffman (publisher of the Dragon book) also engaged, and by the end of the thread, he seemed to develop a respect for some of the experts and was seeking to learn from them.  I view this as a good sign.

I am hoping that Johnson  learns from this that if he wants his scientific arguments to be taken seriously, that publishing them  in a politically motivated book  does not help his credibility and does not motivate people to take his arguments seriously.

I am hoping that Ken Coffman is learning that some discrimination and quality control might be needed in the writings that he publishes if his publishing house is to have any continuing credibility regarding science books.  I am hoping that John O’Sullivan will find better scientific horses to back in his quest to debunk AGW (or better yet, just fight the policies he doesn’t like and stay out of the science).

I am especially looking to hear the reactions to these threads from the “lurkers.” Part I has received a very large number of hits (one of the greatest “hits” ever at Climate Etc.), so it seems a large number of lurkers have been reading this.  Did you change your mind about anything?  Did you learn anything?  Who was convincing, who wasn’t?  Etc.

I am also interested in reactions from Claes Johnson, Ken Coffman, John O’Sullivan.   John and Claes asked for this; did it meet your expectations?  I’m not sure that anyone who has followed this closely regards Claes’ arguments as having survived this scrutiny, do you view it this way?

Sent via email:

Here is one comment that I have received via email, that I have permission to post:

It is exasperating that results easily demonstrable by simple laboratory experiment continue to be challenged by some members of this group. However superficially ingenious their arguments, they fly in the face of experiments that even children can perform with readily-available materials, as well as contradicting proven results in astrophysical theory. I do not propose to contribute further to this group: it is not a sensible deployment of my time. –

Monckton of Brenchley


333 responses to “Slaying a greenhouse dragon. Part III: discussion

  1. Interesting read over the last week or so. I am not a scientist. In my day job I must listen to and evaluate information from scientists, statisticians, technicians, and make decisions based on this information. In my view Claes and friends were unable to support any of their conclusions or alter in any way what we know about radiative physics. It is also clear that the fundamentals of RT is well understood and easily tested throughout mainstream science and engineering. “Slaying” is simply not useful to me in advancing my understanding of climate science.

  2. I don’t think it’s necessary to dismiss an argument becuase of where it appears. Even if Johnson spray-painted his equations on a highway underpass there’s no reason not take a solid look at them. But you have to argue in good faith, and I don’t think we got that from Claes.

  3. hear the reactions to these threads from the “lurkers.”
    Hi Judith! To some extent, the debate about CO2’s effectiveness as a warming agent is usually taken out of the context of nature, and into the lab. An unfortunate side-effect is that the differences between daytime, and nighttime back-scatter is ignored, or not addressed. On a calm night, the limiting factor for nighttime cooling is the local humidity level. If the dew point is 60F near sunset, the overnight low will be very close to the same, give or take a couple degrees. I do not “see” CO2 actively participating to any measurable degree in moderating the low.
    Certainly, the CO2 back scatter could have bumped up the humidity by bumping up the daytime high… or the nighttime humidity in winter could be enhanced by the widespread use of highway salt. BTW, road salt came into widespread use around 1950.
    I’ll go back to lurking, now.

    • Gene, you raise an excellent point about “back radiation” or downward longwave radiation, DLR. High humidity indeed raises it, masking the CO2 contribution. However humidity is highly variable, and when it is very low, e.g. in the desert, there is still considerable DLR, as can be measured with a pyrgeometer or even an IR thermometer such as MicroTemp’s MT-250 which is what I use ($26 from Amazon).

      On a cloudy night one might see 10 °C, and on a clear dry night −40 °C. The former evidences 364 W/m2 of back radiation, the latter 167 W/m2.

      Whatever the CO2 contribution to DLR, for a given local surface temperature (which in conjunction with the prevailing lapse rate determines the temperature at every altitude above), it should be independent of both humidity and cloudiness where it originates. However clouds will intercept it so it won’t reach the ground, because condensed water vapor strongly absorbs all relevant IR. Water vapor interferes less. The take-away point is that very little of the 364 W/m2 can be coming from CO2 on a cloudy night, whereas conceivably as much as 2/3 of the 167 W/m2 on a cold dry night could be coming from CO2.

      The basic problem with simply observing DLR under any conditions is that the mere quantity of it gives no clear as to how it is divided in origin between H2O and CO2. The trick is to look for CO2 emission lines. Unfortunately garden-variety IR thermometers can’t do that, as they work simply by converting the radiation to voltage by focusing it on a thermopile. One of these days I’ll beg borrow steal or buy something more discriminating about wavelength.

      However I’m inclined to the view that the only reliable way to actually observe the contribution of increasing CO2 to global warming is via global temperature records like HADCRUT3, GISTEMP, RSSMSU etc. The problem with any other approach is that the rate of increase is many orders of magnitude below the rates of variation due to humidity, which would seem to be basically your point although you talk about total CO2 radiation rather than its gradual increase with increasing CO2.

      Data without theory however is just as unsatisfactory as the converse. What I like here is to theorize about how increasing CO2 shuts off increasingly many of the hundreds of thousands of CO2 absorption/emission lines. Today roughly 600 in the 288K-relevent temperature range are essentially saturated (optical depth greater than unity), leaving hundreds of thousands still remaining. New lines can be calculated to saturate at the rate of one every year or so (14-18 months roughly). This rate will increase with increasing CO2, being several times higher by the time CO2 is 20% of the atmosphere (not that we’ll ever see anything remotely like that, but Venus is 9700% of our atmosphere, and still not all CO2 lines are saturated.

      When I get a bit more time I’ll post how these calculations can be done yourself.

      • Good point Vaughan about separating the warming effect of CO2 vs water vapor to better understand AGW. Death valley is an interesting case. It is one of the driest places on earth so it should be a good location to detect a rise in DLR by observing a rise in Tmin over the last 50 years, a clear verification of the AGW signal. The temp record in Death Valley does not indicate a rise in Tmin over this period. No clear AGW signal in one of the driest places on earth. That does not mean AGW does not exist, only that we cannot find rising Tmin where we might expect to find it in an extremely low humidity environment.
        http://www.nps.gov/deva/naturescience/weather-and-climate.htm

      • [Death Valley] should be a good location to detect a rise in DLR by observing a rise in Tmin over the last 50 years, a clear verification of the AGW signal. The temp record in Death Valley does not indicate a rise in Tmin over this period.

        Two points combine to make detection of rising temperature in Death Valley unlikely.

        1. We’d be looking for a rise of roughly half a degree over the last 50 years, this being the extent to date of observed warming over the 510 million square kilometers of the Earth, with an error bar that could plausibly be estimated as greater than 0.02 °C.

        2. The area of Death Valley is 7800 square kilometers or 1/65400 of the Earth. One can therefore expect error bars for temperature observation of Death Valley to be sqrt(65400) = 256 times those of the Earth.

        So if the error bar for global warming observation of the Earth is at least 0.02 °C, that for Death Valley will be at least 256*0.02 = 5 °C.

        Detecting a 0.5 °C rise in at least 5 °C of noise would be a remarkable accomplishment.

      • Yes exactly! An AGW signal is still far below our ability to detect it in one of the most likely places on earth.

      • Quite right. This is why one should not try to draw conclusions about global warming from either a single local region or local time, the error bars are way too large.

        In this case the time of 50 years was sufficient, but the region was way too small. As we’ve seen often in discussions of climate change, even conclusions about global warming drawn from enlarging the region to entire countries can yield widely divergent results from one country to the next.

        The entire Earth however does seem sufficiently large to see the impact of global warming, where there is a clear rise of 0.5 degrees over the past half-century.

        Fluctuations over a decade are sufficient to make one decade not a sufficient size. Increasing to two decades reduces the error bars sufficiently to remove noise so as to make trends clearly visible, as can be seen here. Further smoothing to 50 years produces an almost perfectly straight line with a slope of 0.12 °C/decade when averaging 50-year intervals of the form [y-25,y+25] for y ranging from 1967 to 1988. (If 0.12 seems low it’s because the plot is using data since 1942 and hence includes the nonrising part from 1942 to 1970 visible in the first plot where the AMO in its downswing and CO2 was not acting as strongly as in the past few decades, cf this closed-form model.)

        Bottom line: the term “global warming” needs to be taken literally. At 1/65000 of the Earth’s area there is nothing global about Death Valley.

      • Vaughan,

        Is CO2 increasing over Death Valley?

      • ipv0:

        > An AGW signal is still far below our ability to detect it in one of the most likely places on earth.

        Vaughan Pratt:

        > Indeed. […] Bottom line: the term “global warming” needs to be taken literally. At 1/65000 of the Earth’s area there is nothing global about Death Valley.

        kuhnkat:

        > Is CO2 increasing over Death Valley?

      • That’s a fair question. The NASA JPL plot of mid-tropospheric CO2 for July 2009 shows a global variation of around 7 ppmv. In particular the CO2 over Death Valley looks to be around 389 ppmv when the corresponding value over Mauna Loa (in Hawaii) looks to be around 387. (The actual reading at Mauna Loa itself, at 3400 m a bit below the mid-troposphere, was 388.1 ppmv for July 2009, close enough to 387 for government work.)

        50 years earlier, July 1959, the reading at Mauna Loa was 316.54, so the CO2 there rose 71.5 ppm during that half-century.

        Your guess is as good as mine as to how close Death Valley CO2 was to that at Mauna Loa back then. I’d take a stab and guess it was likely to be around 318 or so over Death Valley. This would mean CO2 had risen around 70 ppmv during that period or around 22%. The binary log of 1.22 is .26 or around a quarter. Assuming 4 W/m2 forcing per doubling, we should expect an increase of 1 W/m2 over Death Valley.

        Whether an increase of 1 W/m2 could be picked out of the one or two hundred watts of variation from day to day over Death Valley is an interesting question. I’d be inclined to guess not but that’s a nonexpert guess. Perhaps a very carefully instrumented lab at Death Valley might be capable of making the call whether there had been a 1 W/m2 increase over that period.

      • Thank you Vaughan, I believe you are making my point: The effect of changes in LWR from increasing atmospheric CO2 is still below our ability to detect even in the driest areas on earth. Your point about death Valley not being “global” is also valid. In fact global warming over the last 50 years (The period the IPCC attributes to AGW) has not been global either. It was essentially a rise in Pacific ocean SST during a very strong El Nino dominated phase from 1976-1998 (Ref:Dr. Curry, Bob Tisdale). This put a lot of ocean heat at the surface where we could see it and measure it.

      • It was essentially a rise in Pacific ocean SST during a very strong El Nino dominated phase from 1976-1998 (Ref:Dr. Curry, Bob Tisdale).

        Certainly 22 years is better than 10 years when drawing conclusions from observations of global temperature. But if doubling the time is an improvement, would it not be reasonable to expect that extending it to 161 years would give an even clearer picture?

        Look at the HADCRUT3 temperature record from 1850 (inclusive) to 2011 (exclusive), smoothed with a 20-year running average to remove all shorter-term variations. How would you describe that curve?

        To me it looks a little bit like cos(x) + exp(x), where x = 0 is 1940 and x = 1 is 1950. 1976-1998 therefore corresponds to the interval 3.5-5.6 for x.

        Now cos(x) rises over that interval, but so does exp(x). It is that clearly visible upward slope on the right that makes it unreasonable to attribute the whole of the 1976-1998 rise to the sinusoidal component.

        This shape is something like what one would expect if a natural oscillation that had been going on for centuries without any help from us were to be added to the impact on global temperature of an exponentially growing population, with the latter masked out by the former prior to 1940.

        And indeed there is not just one but two strong oscillations, the 56-year Atlantic Multidecadal Oscillation (AMO) and the Pacific Decadal Oscillation (PDO). People seem a bit more coy about the period of the latter but I’ll stick my neck out and put it at 75 years based on analysis of the 12-year-running-average HADCRUT3 temperature record. And as a null hypothesis I’ll assume the two oscillations have equal amplitude.

        Actually exp(x) is not quite right because the impact of anthropogenic CO2 is not linear in population size. Furthermore that CO2 is superimposed on a (currently) larger natural base estimated at 280 ppmv.

        David Hofmann, until recently at NOAA ESRL in Boulder, has proposed modeling CO2 in the year y as 280 + 2^((y − 1790)/32.5) ppmv. This pegs 1790 as the year anthropogenic CO2 had risen to 1 ppmv, and has it doubling every 32.5 years from then on. This formula is in excellent agreement with the Keeling curve, and also with the consensus estimate of 280 ppmv for immediately preindustrial CO2. (Going much further back we find CO2 levels as high as 6000 ppmv.)

        For convenience call this formula the Hofmann law, notated HL(y) giving the level c of CO2 in ppmv as a function of year y. The Hofmann law analyzes CO2 as the sum of a constant (280 ppmv) natural component and an exponentially growing anthropogenic component.

        The Arrhenius law, AL(c), gives the incremental surface temperature relative to some base T0 as a function of CO2 level c. Arrhenius proposed that this function was logarithmic, that is, AL(c) = log(c) − T0 for some choice of base for the logarithm. Since we always speak in terms of doubling CO2 rather than multiplying it by e, the choice of base is best handled by taking it to be 2 (so log becomes lb for binary log) and correcting for the actual base by writing

        AL(c) = s*lb(c) − T0

        Composing AL and HL then gives what one might call the Arrhenius-Hofmann Law, AHL(y), giving the expected surface temperature as a function of year y in the closed form

        AHL(y) = s*lb(280 + 2^((y − 1790)/32.5)) − T0

        The base temperature T0 is arbitrary. A natural choice that fits well with the sequel is the one that makes the mean of AHL(y) zero over the period 1856-2005. (The 12-year smoothing consumes 6 years of data from each end.) If we do the same with the ocean oscillations then every linear combination of AHL(y) and the ocean oscillations will automatically have mean zero. This then leaves s as the only unknown parameter in AHL(y).

        The Atlantic and Pacific ocean oscillations would appear to have drifted into phase roughly simultaneously, with both crossing zero (relative to mean global temperature for say 1850-1915, before CO2 became a significant factor) in 1925. This suggests the formula

        AMO(y) = a*(sin(2 pi(y − 1925)/56) + sin(2 pi(y − 1925)/75))

        (This might be better called APMO to recognize the Pacific component.)

        Like AHL(y), AMO(y) has amplitude a of the oscillation as its only unknown parameter. Since both are top-level coefficients, we can therefore do a routine least-squares fit of AHL(y) + AMO(y) to the HADCRUT3 temperature. The best fit gives s = 1.84 and a = 0.066.

        This fit is shown graphically in this plot. Note in particular the cyan (light blue) curve labeled “preresidue” resulting from subtracting AHL(y) from the HADCRUT3 record. This brings out the sinusoidal nature of the ocean oscillations more clearly, particularly on the right where previously the CO2 contribution had been masking the sinusoidal shape.

        Bottom line: for 1976-1998, AHL and AMO rose together at roughly the same rate. Hence they should accept equal responsibility for the big rise in temperature over that period.

        Could this rise have been predicted in 1980? Certainly not on the basis of the HADCRUT3 record alone, which suggests that after a century of wild swinging, global temperature has at long last settled down to a peacefully calm level.

        But if you look just at the data available from 1850 to 1980, the oscillation is no less clear. In 1980 one could therefore have conjectured the formula AMO(y) with a as the only unknown. Less was clear then about AHL(y), but by then there were 22 years worth of data from Mauna Loa, enough to come up with suitable parameters for AHL(y).

        Performing the same least squares fit for just the 1850-1980 data yields http://thue.stanford.edu/30yearsago.pdf“>this plot. This best fit makes s = 1.83 and a = 0.067. The extra 30 years of data change these only slightly, to 1.84 and 0.066.

        More interestingly, the least squares fit predicts a coming temperature rise that would have been scoffed at not just by sceptics but the scientific establishment, who would likely take the unprecedented rise not as predicting a coming calamity but as proof that the methodology was flawed.

        And as we have witnessed over that period, they would all have been wrong, at least about the temperature rise which is a less subjective concept than “calamity.”

      • note chief hydrologist is discussion this issue over on the new decadal variability of clouds thread

  4. Judith:
    Sorry to be negative, but your lecturing gets really silly:

    “I am hoping that Johnson learns from this that if he wants his scientific arguments to be taken seriously, that publishing them in a politically motivated book does not help his credibility and does not motivate people to take his arguments seriously.

    I am hoping that Ken Coffman is learning that some discrimination and quality control might be needed in the writings that he publishes if his publishing house is to have any continuing credibility regarding science books. I am hoping that John O’Sullivan will find better scientific horses to back in his quest to debunk AGW (or better yet, just fight the policies he doesn’t like and stay out of the science).”

    These are NOT your students!

    • I’m okay with being described as Judith’s student…and I am learning quite a few interesting things, mainly about how climate scientists think about physics and the difference between applied (practical, engineering) and theoretical (academic) physics. In academia, an important idea need only be interesting and consistent with what is known. In engineering, an important idea must be useful.

      • Academia is of limited value without real world experience. (A lot of academics don’t know that;)

  5. I would read section 7.14 on the IR camera, and then read Monckton’s remark. It can easily apply to that section, and be regarded as a damning indictment of that piece. Which side is denying the results of simple experimentation, I would ask? I think Monckton scored an own goal with this remark, or perhaps he actually meant to disown this piece of work. It is not clear at all to me what he was saying in the context of these threads.

    • The email from Monckton was part of an e-dialogue of the Dragon authors and me (originating from my dismissing comment of the book on a previous thread), in response to the greenhouse effect, backradiation, etc. doesn’t exist arguments of the Dragon authors.

  6. What I took away from this dialogue:
    1) Judy Curry read chapter 1 and was alarmed at the elegance of the assault on “back radiation”. She posted the first two chapters to engage physicist heavy hitters to poke holes in the math.
    2) The dialogue was prolonged and eventually it came down to Tomas Milanovic who framed the question regarding the validity of equation (4) and the substance of it derivation. It seems reasonable that the focus was on equation (4) and that the math needs explaining.
    3) Claes Johnson’s arguement boiled down to, at least to me, that the duality of light being both wave and a particle resulted in the loss of the power of Theremodynamics and a reliance upon statistical physics. To preserve Thermodynamics he separated the heat from the photon activity. Photons can move back and forth between bodies proportional to the bodies temperature. Heat on the other hand can only go from higher heat to lower heat; the Second Law of Thermodynamics. He developed an arguement for a string vibrational state that preserves Thermodynamics.
    4) One commenter stated that photons transfered heat also. A photon would strike a willing CO2, excite electrons to a higher state (vibrational & rotation), then either a) relax emitting a photon up or down; b) collide with another molecule elastically and transfer no energy; c) collide inelastically and transmit energy which can be recorded as temperature. It seems to me that there must be some teensy weeny bit of energy lost somewhere in the flying around, bumping electron energy states, relaxing energy states. I just wouldn’t know how much or little that energy would be. Left to probablity equations to address.
    5) The downward radiation surface warming effect proposed by the Radiative Transfer Model is in play at low latitudes and high latitudes. The abundance of water in the tropics makes water vapor the likely dominate green house gas in low latitudes. Whether there is net additive surface warming by green house gases is likely dependent on how water/vapor in the aggregate is handled: evaporation and wind speed, surface longwave radiation, convection via shafts of thunderstorms, advection to the high latitudes, altitude where water vapor may release energy particularly if >20,000 feet by-passing almost any concentration of CO2. Water vapor is the heavy heat lifter, at least during the day. The balmy night, water vapor again has a higher heat capacity than CO2 so the cooling of the night time is slowed in the tropics. At the poles, during winter quiet nights the role of CO2 downward radiation is more likely detectable, yet its scarcity as a trace gas, limits its impart on warming earth’s surface at least at the winter poles.
    The message that I took away after almost 1000 comments, is a nudge towards a role for the physics of the RTM. The role of CO2 in surface warming, elevating the adiabatic lapse rate, influencing atmospheric optical density, etc., is in proportional to its atmospheric concentration, trace. The vastly larger partial pressures of Nitrogen and Oxygen keep the partial pressure of CO2 as a trace gas; hence, its impact is trace. No matter how active a scavenger and emitter of photons CO2 is, it is a boy trying to do a man’s work. Ergo, efforts to manipulate CO2 per se to manipulate weather/climate will have a low impact, except maybe as a plant fertilizer and the benefits for food, and trees, other plant life may contribute to regulating the planet’s energy budget.

    • RiH008,

      I commend you on an excellent post.

      AB

    • Well, I wasn’t alarmed at the elegance of the assault on back radiation. The first chapter is not elegant. The second chapter has some superficial elegance in terms of diving into fundamental physics and lots of mathematics. But it is an incomplete argument (CJ calls it a “sketch”) and it comes up with a result that I know to be wrong based up mountains of empirical evidence. CJ was not going to be convinced by empirical evidence. He was only going to be convinced by someone with a better set of equations. Dave N and Tomas showed CJ that his equations are mathematically incorrect. End of story. Except it seems that CJ doesn’t “get it.”

    • Isn’t it rather incredible that there is such a marked contradiction in interpretation as to how the debate on Johnson’s chapters went? The authors of ‘Slaying the Sky Dragon’ are delighted that on a blog less likely to be favorable to our position, so little if anything was shown to be demonstrably wrong with Johnson’s analysis.
      Frankly, my own view is in line with that of RiHo08. Johnson demonstrated that Dr Curry clearly hadn’t read the book as she claimed; Johnson’s unique approach to unravel the mess climatologist had spun from the various laws succeeded and such that genuine flaws in the foundations of the greenhouse gas hypothesis have been exposed and will never be so glibly glossed over again. Truly, the GHE is not the driver of climate that politicized post normal ‘scientists’ claim it to be.

      • John, your assessment is beyond belief. Johnson’s “theory” that there is no back radiation is falsified by mountains of measurements made by infrared radiometers pointing skywards (measurements that school children can make). Further, Johnson’s mathematics were shown to be seriously in error by Tomas Milanovic and Dave N. In science, either one of these two would be the kiss of death; re CJ’s theory, the combination of both is a death knell. CJ’s failure to have any of this sink in leads me to believe that he does not know anything about the scientific method. His insistence on not even responding to these criticisms leads people to regard him as a crank/crackpot. Based on Part II, he can probably get this published in E&E, but certainly not in any scientific journal of any repute or credibility.

        Given your complete failure to understand all this, I do not see any point to engaging with you further on any topic related to the science of the greenhouse effect. Over the past year, I have very patiently pointed out to you many problems with the ideas and theories you are promoting. By your irrationality on this subject, you are isolating yourself even from skeptics like Monckton. If you think you can make money writing more books like this, go for it. Gullible people will probably by your book. But you will find yourself increasingly marginalized and isolated and disrespected by serious people (including CAGW skeptics).

      • Judith
        When I heard that you had agreed to host a discussion on this book I made the remark that it sounded like the publishers/authors were using it as cheap advertizing. The following nonsense merely confirms my suspicions:

        John Sullivan: “The authors of ‘Slaying the Sky Dragon’ are delighted that on a blog less likely to be favorable to our position, so little if anything was shown to be demonstrably wrong with Johnson’s analysis.”

        I wouldn’t be surprised to see a reference to your blog in the book’s future promotional material. I think you might have been conned as to their intentions.

      • Judy, now here is the rub: Claes Johnson presented a Planckian math model ( not a statistical one-which has thrown you) and no one has been able to refute it. Sure, there has been a lot of hand waving but no direct discussion entered into about Johnson’s new analysis based on deterministic finite precision computation.
        Frankly, the evasion by you of any such discussion suggests to me that you know Johnson is right and there is no greenhouse effect causing extra warming to our climate by so-called “back radiation.” Johnson exposes the error whereby the GHE employs a radiative model working with averaged values where emission properties to and from gases dominates yet physical processes cannot react to average values, ever.
        Moreover, that there is not a shred of empirical evidence to support your beliefs, despite $100 billion and 25 years spent by climatologists to prove the alleged additional warming effects of such feedback, further discredits your unproven hypothesis. QED on the QED!

      • John, CJ’s math is INCORRECT. And his conclusion is flat out incorrect. Satisfying empirical evidence does not exist for many aspects of the climate change problem. However, an enormous body of measurements (www.arm.gov) show that infrared radiative emission from CO2 and H20 gases produce a measurable and substantial flux of heat at the earth’s surface, which CJ’s theory alleges to disprove. So there are very detailed and numerous measurements, particularly over the past two decades, that provide overwhelming empirical evidence to support what CJ is trying to refute.

      • I have two comments about this. The first is that the “finite precision computation” portion of the text is not contained in the available sample chapters, and this may be why it didn’t receive focus. The second is that what John O’Sullivan wrote above is the most disingenuous, dishonorable thing I’ve seen in a long time.

      • John,

        ‘The authors of ‘Slaying the Sky Dragon’ are delighted that on a blog less likely to be favorable to our position, so little if anything was shown to be demonstrably wrong with Johnson’s analysis.’

        Huh? Which blog post were you reading? Or did you just read the comments that claimed to disprove radiative transfer via water bottles and a space heater?

        As someone with explicit training in the physics canon, optics, molecular physics and interactions between light and molecules, I can tell you that, if anything, there is nothing demonstrably correct about anything Dr. Johnson begins with in Chapter 2.

        The fundamental physical assumption on which his entire thesis is based presumes that a blackbody, to which he correctly attributes the ability to absorb ALL frequencies of light, can somehow detect another, colder blackbody nearby via an undetermined physical mechanism that is neither in his math nor can he explain in physical terms. When such a detection occurs, the warmer blackbody becomes a ‘high pass filter’, basically not interacting with lower frequencies of light originating from the colder blackbody…but it’s still a blackbody and must absorb ALL frequencies of light. When this point was brought to his attention, there was no argument, no comment, no equations nor a physical mechanism to explain how the detection occurs or why the warmer blackbody stops, essentially, being a blackbody.

        That is a contradiction in physical understanding of the highest order that I would not expect from even a high school science student.

        Furthermore, when I pressed him several times to tell which version of Maxwell’s equations he used to derive his results (with or without sources), there was no answer. It is my guess that Dr. Johnson is not taking into account the finite, but very small sources of spherical waves. It is also my guess that he does with out of resentment for quantum effects. Without these sources, Dr. Johnson is correct that radiative effects go away…because there are no sources of radiation. But that is not a realistic physical model for the atmosphere.

        I have also found that none of the substantive, constructively critical comments I have posted on his own blog have appear for the rest of readers to take in.

        And yet, you are still saying that no one could demonstrate Dr. Johnson incorrect.

        I’m beginning to think that if you truly believe what he has to say in the argument he has formulated in Chapter 2, there isn’t a way to convince you he is wrong. You’re along for the ride and aren’t interested in the scientific endeavor.

        Too bad.

      • Maxwell, thank you for this summary

      • The authors of ‘Slaying the Sky Dragon’ are delighted that on a blog less likely to be favorable to our position, so little if anything was shown to be demonstrably wrong with Johnson’s analysis.

        They might be less delighted if they showed a bit more reading comprehension. Numerous flaws were pointed out and the “rebuttals” were just embarrassing. The credibility of the authors took a big hit on that thread.

      • The authors of ‘Slaying the Sky Dragon’ are delighted deluded if they think that, that on a blog less likely to be favorable to our position, so little if anything was shown to be demonstrably wrong with Johnson’s analysis.

        Fixed it for you. :)

    • There are a number of common misconceptions in this posting which arise from conceptual and historical frameworks. Thermodynamics arose in the early and middle 19th century, before atomicity was firmly entrenched, before electricity and magnetism were unified, well before the electromagnetic waves were known to exist, before it was realized that light was simply electromagnetic waves, and before we had a clue as to the internal structure of atoms. This means that radiative heating was not known, but even beyond that that the physical mechanisms involved in convection and conduction were also not known.

      In that world, heat flow was an intrinsically a unidirectional directed flow of energy in the same way that a chemical reaction was unidirectional and that chemical equilibria were static. Today we recognize that chemical reactions as well as heat flow are dynamic processes, with energy moving in both directions and the flow in the prevalent direction being the balance between the flow in one direction as opposed to that in the other. We refer to this as the net heat flow, but there is no difference in how each flow is calculated.

      We can easily accommodate this difference into the laws of thermodynamics, and indeed J Willard (hi there) Gibbs who came up with the idea of a ensemble, upon which statistical mechanics, is bases also created much of what we think of as classical thermodynamics. Stat mech and classical thermo are joined at the hip, and quantum mechanics can be integrated into both (although the mathematical complexity is enourmous, or at least that’s how Eli remembers it from grad school).

      In this case it is not to hard to see that proper analysis requires treating the photons as an ensemble with a distribution of energies that has a temperature. The ensemble is in equilibrium with the surface emitting it, but not necessarily with the absorbing surface. How the energy is distributed in the absorbing surface and what happens there is a straight forward energy transfer calculation as Phil Felton points out.

      One of the characteristics of stat mech is that when the size of the ensemble is small, fluctuations are large, and calculation becomes impossible. That is one of the many places Johnson goes wrong. When the ensemble is large (Watts of energy) the fluctuations are incredibly small and the calculation is easy.

      On one level, Johnson is invoking a Maxwell Demon, because in his fallacy the receiving surface, made up of individual atoms, has to recognize the nature of the impinging radiation. Since the actual absorption of the radiation depends on the individual molecules/atoms, they somehow have to get a message from the rest of the surface: Hey our temperature is X, because remember the temperature is an intensive property of the entire surface, and that photon has too much or too little energy for us to absorb. If you make the surface element atomic level, well then you have another problem, the temperature fluctuations are huge.

      As for the real Maxwell Demon (see here if you want to know what Eli is talking about) is an entirely different problem with entirely different energy flows (you have to power the demon and send her information) and a different solution, but again, not Johnson’s.

      And that is only the beginning

    • In a nutshell. You have a talent for clarity.

    • RiHo08

      Thank you for your post, which I believe adds significantly to the discourse, though perhaps not only in the way you intend.

      The critique of Slaying.. in part 1 challenged my skills and knowledge, expanding both greatly and sharpening into clearer focus many ill-defined questions I had not before been able to enunciate for myself. Your own post brings additional focus to some of these technical scientific questions, even if it is in a transparently skewed framing.

      My lament is that had the book, or even the two chapters in question, been as worthwhile as your post, I would have so much more opportunity to learn and understand better about these topics and the processes and methods mentioned. I would gladly wade through ten times as many comments to get to such meaty stuff (too many of the wasted comments admittedly being my own.)

      It is disappointing that the original work falls apart so early as Eq(4) and involves what appear to be simple integration errors, instead of the sort of problems and mistakes your comment demonstrates of interpretation, conceptualisation, syntheses and analyses which make these discussions most intellectually stimulating.

      Your post in defense of the work in question is, as a strategic public relations exercise, a far better document than the original.

      It shows a light touch, adeptly maneuvering the framework and setting tone and mood by touchstones and incidental insult subtly and efficiently.

      Spinning Judith Curry as a careless reader and arch alarmist, while making our mathematical mercenary’s thoroughly discredited sellsword half-baked, phoned-in differential doodles the crusade of a hero laying siege to the infidels?

      Priceless, when set out as you do with rose petals leading down the garden path, into the rabbit hole, and out into a wonderland where one doesn’t need math to be correct by logic nor experiments to be valid by objective standard, so long as they agree with one’s preconceived beliefs.

      Who says scientists can’t handle Policy just as sophistically as politicians and lobbyists?

  7. Dear Judith,
    I’d like to clarify for me, how climate scientists view “back radiation”
    To start, here is my view.
    As I understand it: “backradiation” is the downwelling infrared radiation integrated over all directions that go into the surface.
    I am an experimentalist. For me a physical cause is something that I can switch off, at least in a thought experiment, and get an intended effect.
    If I switch off the sun in a thought experiment, I get cooling of the atmosphere and the surface. If I switch on the sun again, I get warming of the atmosphere and the surface.
    If I switch off ”back radiation” in a thought experiment on the night side I get a higher cooling rate.
    If I switch on “backradiation” on the night side I get a lower cooling rate.
    If I switch on “backradiation” on the day side I get a higher heating rate. Combining my thought experiments I deduct that if I switch on “back radiation” on the day side I get a higher heating rate due to a lower cooling rate because of the “back radiation”
    Moreover, I can deduct that with “back radiation” the earth system gets or stays warmer than a reference system without “back radiation”, because of a lower cooling rate.
    One might also say “back radiation” leads to a warmer surface compared with a reference system.
    However, I do consider sentences as incorrect with respect to physics that state only: “back radiation” is warming the surface.
    Of course in any private conversation we state things like: my new coat is warming me nicely. But in a scientific discussion, we need to employ the correct wording, especially in lectures or papers.
    For my opinion sentences like: “back radiation” is warming the surface
    cause therefore a lot of misunderstanding in the blogosphere. The Gerlich and Tscheuschner discussion is based on it.
    Maybe you can comment on it.
    Best regards
    Günter

    • Sorry one typo
      Correction:
      My sentence should read:
      For me a physical cause is something that I can switch on and off, at least in a thought experiment, and get an intended effect.

    • Everything you say, I would agree with, but it is not always that simple. Can it be said that an increase in back radiation (as with adding CO2) warms the surface, since you go from one equilibrium temperature to a warmer one?

      • Obviously there is no way to switch it off. But clouds do float by. Imagine you are measuring the temperature of the skin layer of the ocean. Bright and sunny, and suddenly a very big cloud decides to spend some time overhead. SW? LW? Temperature of the skin layer?

      • JCH,
        that is, why I said thought experiment or Gedankenexperiment.

      • Humidity. Things do not cool as rapidly in the Amazon as they do in the Sahara.

      • I don’t really see why not. I have an image of a discrete molecule above the earth emitting photons in all directions. It doesn’t care – entropy is king and it just radiates energy. The surface is also radiating and heating the molecule which then reradiates in all directions again. The warmer the molecule gets the more it radiates to the fourth power.

        The net flow of energy is from the Sun to the surface and back to the molecule, back to the surface and out into space. The molecule is warmed much more by surface emissions then the surface is warmed by the molecule – cold to warm. But if there are more and hotter molecules more photons are emitted in the every direction including the surface – so the surface warms a little bit.

      • ChiefHydrologist,
        As I said you can say many things in a colloquial manner.
        However, your sentence: “The molecule is warmed much more by surface emissions then the surface is warmed by the molecule – cold to warm”, is ambiguous as well.
        Warming means a temperature increase. If a hot and a cold system exchange photons, within adiabatic surroundings, the physical effect is a cooling of the hot system and a warming of the cold system.
        Moreover, it is also incorrect to assign to an individual molecule the label ”warmer” from a thermodynamic point of view.
        Of course I know that in spectroscopy one uses for excited molecules sometimes the term “hot” molecules, but this is something else.
        Your sentence:
        “The surface is also radiating and heating the molecule which then reradiates in all directions again. The warmer the molecule gets the more it radiates to the fourth power”
        is also incorrect, if you mean the Stefan-Boltzmann equation. One cannot apply Stefan-Boltzmann to individual molecules.
        Moreover, spontaneous “reradiation” or reemission does not change temperature. It needs at least a collision with another molecule to thermalize the photon energy within the atmospheric layer.
        This is a huge problem in the discussion about climate change that a lot of sentences are applied incorrectly or incompletely with respect to physics.
        Regards
        Günter

      • As they say on The Simpsons – Chief Hydrologist is a vocation not a name. Call me Rob.

        Now I may have bitten off more than I can chew here – but we are nothing if our reach doesn’t exceed out grasp.

        ‘ If a hot and a cold system exchange photons, within adiabatic surroundings, the physical effect is a cooling of the hot system and a warming of the cold system.’

        That is of course a demonstrable net effect based on the 2nd law of thermodynamics – and I think that a net effect is a better assumptions than a one way traffic in photons – which seems pretty improbable.

        ‘Moreover, it is also incorrect to assign to an individual molecule the label ”warmer” from a thermodynamic point of view.’

        It has of course an internal energy. As I understand it, if the atom gets hit by a photon – literally a packet of energy – electrons do a quantum jump into a higher orbit and it gains some internal energy. Hot and cold was a bit loose – but I find it usually find it wise not to mention politics, religion or quantum mechanics.

        ‘One cannot apply Stefan-Boltzmann to individual molecules.’

        Perhaps not – but excited atoms emit photons in the infrared frequencies unless they are super excited in a plasma arc for instance. The more excited the atom the more photons shoot back and forth as heat fluxes in wave/particle duality – something else better not mentioned in polite company.

        ‘Moreover, spontaneous “reradiation” or reemission does not change temperature. It needs at least a collision with another molecule to thermalize the photon energy within the atmospheric layer.’

        Bullshit – because what we perceive as heat is the radiant energy – all these photons moving as waves in the space between molecules.

        ‘This is a huge problem in the discussion about climate change that a lot of sentences are applied incorrectly or incompletely with respect to physics.

        I think you are being too hard and fast – there is a role for science communication in simpler language. Sometimes you get it wrong – and I am always complaining about journalists getting it wrong. I was just trying to be helpful. There was a mention of equilibrium states – and of course nothing is in equilibrium in the atmosphere. I was trying to be helpful, I was a bit colloquial, so sue me.

        On the other hand if you really believe that physics precludes the greenhouse effect because of the 2nd law of thermodynamics – you’ve got the basics wrong.

      • Dear Rob,
        I did not mention the second law, which the greenhouse effect of course obeys. I asked how climate science views “backradiation” and expressed my views about that. I do not want the explanation that climate scientists use for layman. I asked for the view of climate scientists about “backradiation”with respect to physics.
        Could you please identify, which of my sentences in the above are wrong with respect to physics. If you considered my reply to you rude, please excuse me.
        Best regards
        Günter

      • Günter

        I clearly agreed with everything but the idea of thermalisation by collision.

        Forgive me if I was too robust – bullshit is a term of endearment in north Queensland. As in – well hey there you bullshiting old bastard.

        As a humble hydrologist – I don’t think there is any such thing as backradiation. Radiation is in all directions. Some of it just happens to head down to the surface.

        Cheers
        Rob

      • Rob,
        ok. Thanks for the clarification of BS.
        Both of us eed to recognise that we have different, but equally valuable areas of expertise. If I anwser in blog like that with: I consider that sentence or view incorrect, I mean that we need to calibrate our definitions or point of view.In order to do that I have to state my view.
        Per example, thermalisation and local thermodynamic equilibrium are the key ingredients of the radiative transfer theory within the atmosphere up to about 70 – 100 km height.
        From a microscopic point of view. A photon that starts at the surface excites a vibrational state of a CO2 molecule and gets reemitted within the natural lifetime and gets back to the surface did not heat or cool any matter on its path.
        Those are the photons that according to my opinion are called photons that are absorbed and reemitted. This absorption and reemission does not change the temperature.
        Photons that increase temperature need to be absorbed and thermalised. That is what thermalisation means. Individual Photons that are emitted because of temperature are not connected with the earlier absorbed photons. Therefore the term “reradiation” or “reemission” for them would be ambiguous.
        One note of caution dealing with a microscopic interpretation of radiative transfer theory. Radiative transfer theory is a classical theory about the transfer of radiant energy, not about individual photons that get of course emitted in all direction. It is linked to absorption, thermalisation, scattering and emission from surfaces, clouds and molecules, but on an averaging level.
        Radiative transfer theory “sums” or better integrates at first over the energies of all photons within a certain frequency band that travel through a unit area per unit time and calls this the spectral net flux per unit area per unit time per frequency interval.
        “Backradiation” sums or integrates over the energy of all photons per unit time within the longwave regime that travel from anywhere in the atmosphere and get absorbed at the surface.
        The last two paragraphs are very short and do not capture all that is within radiative transfer theory.
        Your understanding of these things might now be very different. You might just have another definition of reemission or of radiative transfer that doesn’t fit mine.

        Best regards
        Günter

      • Thanks Gunter – catch you later

      • Technically you can build an ensemble of single molecules bathed in a photon field emitted from a warm body. Sunlight for example is in equilibrium with the solar surface where it is emitted from. To make this simple assume that the molecules start out in the ground quantum state No, and only can transition to a single excited state Ne with energy Eo

        The net effect (which is not so hard to calculate) is that at first the temperature of the ensemble is 0 K. At first much more energy is absorbed from the photon field than emitted to it as energy is passed from the field to the molecule and little returns. At any time the temperature T’ of the ensemble is given through

        Ne/No = exp (-Eo/kT’)

        until, at some later time the temperature T’ of the ensemble reaches T where absorption and emission are balanced.

        You can obviously complicate this by allowing interactions among the molecules, structure in the photon spectrum, etc. , but the bones of the issue remain the same as does the solution.

      • Eli,
        Technically, yes. You might assign a translational temperature in one direction to an individual molecule, if you want and can measure its kinetic energy, but already for rotational, vibrational or thermodynamic temperature the problem is ill-defined. For your ensemble in a radiation field you can define a rotational or vibrational temperature or temperature for the electronic transitions.
        However, if we don’t allow for frequent collisions the translational or kinetic local temperature could be very different from these other temperatures.
        Regards
        Günter

      • You can’t assign a temperature to a single molecules, you can assign a temperature to an ensemble of single molecules. It is an important difference and one that you and CJ simply don’t get. If you have local thermodynamic equilibrium (LTE) the temperature in all the modes of motion is equilibrated. That, for all practical purposes is a working definition of LTE, and a condition that applies in the atmosphere below 100 km or so.

        An example of what you are trying to get to is found in the upper atmosphere above 100 km, where density is low and collisions infrequent. In that case, there is no LTE, and it is difficult to talk about “temperature” although people do so, but only after haggling out what they mean.

      • Eli,
        I had a civilized discussion with Rob and you stepped by.
        I already said to Rob that it makes no sense to assign a temperature to an indivdual molecule from a thermodynamic point of view. You should reread my lines again.
        However, you came across with your ensemble in a radiation field as an example. I just mentioned that if molecules are in a radiation field local kinetic temperature, the translational temperature, might be different from rotational or vibrational temperature. This is different from a temperature bath with collisions.
        You also could know that in spectroscopy or molecular beam slang one speaks from “hot” and “cold” molecules, which is then an “equivalent” temperature representing an energy, since k is a constant. If you read my lines you will also see that I already mentioned that.
        With radiative temperature one technically does a similar thing, comparing with a black body.
        You seem also to have the bad habit I observe in the climate debate not to read and strive to understand what people say, but deduct very early that they don’t understand the physics or assume that they don’t.
        That is sloppy and really bad manners in real life. If you didn’t understand my lines, you could have just asked me.
        Regards
        Günter

      • Still the point is that when we are talking about the atmosphere, we are talking about a situation where the average time for thermalization of energy among the various degrees of motion is short compared to the average time for absorption or emission of radiation.

        Under those conditions it makes sense to talk about a single temperature imposed by the local thermodynamic equilibrium.

      • JimD,
        As I stated above. You need to augment the sentence with the clause: with respect to a reference system, with a lower CO2 concentration and if the incoming solar flux stays constant.
        The root cause for the surface warming going to the new stationary state is the sun together with the increase in CO2 concentration.
        The sun alone and the “backradiation” alone won’t do that.
        You cannot isolate “backradiation”, if you want to state the root cause for the surface warming. Isolating “backradiation” means that you have to restrain yourself to a sentence like: Increased “backradiation” leads to a decreased cooling rate, which in turn, since the heating rate caused by the sun is constant, leads to a new stationary state with a higher temperature.
        To complicate matters, it is common practice in the theory of heat transfer to assign the term heat flux to “backradiation”. However, if you want to explore the temperature of your stationary state, you need to consider all flux together and calculate the temperature in turn. However, calculating a temperature from different flux is not the same as identifying the physical root cause for warming.
        But this is my interpretation of the physics and the wording that I would use to explain surface warming with “backradiation”.
        Other people are entitled to use their own words. I have no problems with that.
        However, I do observe that a lot of discussion in the blogosphere deals with ambiguous sentences rather than with the physics. Therefore I’d asked the question how climate science views “backradiation”. I expressed my views to start with.
        Best regards
        Günter

      • Guenther, yes, there are subtle differences in how things are put. When dealing with equilibrium states, I would say an increase in back radiation would cause warming, just as a decrease in solar radiation would cause cooling. This is subtly different from saying back radiation causes warming or the equally wrong statement that solar radiation causes cooling in the above context. It is the changes that cause the warming and cooling when dealing with an equilibrium state. Climate science is mostly about equilibrium and gradual changes in that state.

      • JimD,
        Agreed. But I do think there is also a fundamental disagreement what physicists that are used to design experiments to prove something would call a root cause and scientist from fields that are restricted to observations of mostly stationary states would call a root cause.
        Per example if you compare two stationary states you observe increased “backradiation” in the state with higher temperature. The higher, increasing “backradiation” under local thermodynamic equilibrium is caused by a simultaneous warming of surface and atmosphere, which is caused by constant energy input of the sun and the reduced outgoing longwave radiation due to the increased CO2 concentration. These are the only energy flux that change the energy content of the earth system. So I would call the constant energy input of the sun and the reduced outgoing longwave radiation the root cause for warming of surface and atmosphere. Others might choose to do it differently.

        It is quite common in physics or chemistry that different field use slightly different mechanistic interpretations of the observations. One needs to appreciate the separate views or mechanistic interpretations as long as they are correct and complete, explain the data and do not violate 1st and 2nd law.
        Regards
        Günter

      • Guenter, it is just the level of the description that makes the difference here. You can think of the climate system as an equilibrium-keeping black box, where you only increase back radiation (or atmospheric emissivity in general to be more precise) and the surface warms, solar input being held constant. This shows cause and effect cleanly. Or you can try to understand what goes on in the box, and for climate you need to, because other things change in response that also have an effect on temperature, and these feed back to both the back radiation (via water vapor increase) and solar radiation (via clouds). The no-feedback response is just the black-box view which is a useful level to begin with.

      • JimD,
        I think what you describe is the view of an internal observer, who doesn’t know about the top of the atmosphere flux and assign the observation of increasing back radiation that he makes as the root cause.
        An external observer looking down from outside TOA sees the reduced OLR and the constant sun and assigns the root cause to this observation. Both see surface and atmosphere warming simultaneously. I do think the view of the external observer provides more information, because in the case of the external observer the climate system is not a black box, but rather an open system that changes due to the energy flux across the system boundary.
        Regards
        Günter

      • Guenter, I agree that the external view is a useful one, but it can’t distinguish the surface from the atmosphere in order to understand the effects of back radiation, so for this discussion a so-called internal view is more useful. I also employ the external view for other arguments.

      • JimD,
        But the internal observer under LTE conditions seems to be stuck in a circular argument. He observes increased “backradiation” and increased surface and atmospheric temperature simultaneously. And he knows that “backradiation” depends on temperature.
        The external observer can deduct from the first law that the energy content of the earth has changed.
        For the greenhouse effect the external view leads according to my opinion.
        So if Claes tells an external observer that backradiation doesnot exist, the external observer will not care because he has observed the greenhouse effect looking at TOA and the surface temperature. So, I do think for the discussion with G&T or CJ, the external view is preferable.
        Regards
        Günter

  8. Correction:
    My sentence should read:
    For me a physical cause is something that I can switch on and off, at least in a thought experiment, and get an intended effect.

  9. Class Johnson

    Judy: As I have said, the math model I analyze (basically the one considered by Planck) has not been considered seriously in the discussion, neither my new analysis based on deterministic finite precision computation instead of
    statistics, so what can I say, other than that I look forward to a real constructive discussion. Hopefully your post can open to this. Of course listening is essential in science but you also have to have something to say.

    • I thought Tomas was very helpful to you in his post. In terms of proofs and disproofs in physics, correct mathematical proofs are absolute. Having inconsistent physical units in an equation in any part of the proof (which you have and did not address when Tomas pointed it out) is evidence of a fatal error in the proof. From a physics point of view, one gedanken experiment for which the results don’t hold is evidence of a fatal flaw in the work. These weren’t addressed either. Apparently it’s axiomatic that critical questions are not part of a constructive discussion.

      All I saw was a positional approach from you – you’re right, and everything that says you aren’t is dismissed or ignored.

    • [youtube=http://www.youtube.com/watch?v=2eMkth8FWno&w=480&h=390]

      http://www.youtube.com/embed/2eMkth8FWno
      With CJ playing the black knight

  10. The problem with all these technical discussions is that they have a full value only for people with a standard natural scientist’s model of thinking, i.e. in a linear model of thought in terminology used by Jeroen van der Sluijts and many others.

    As Gavin Schmidt stated (in other words) in his reply email reproduced in the chain “Lisbon Workshop on Reconciliation: Part IV”, the disagreements are not originating from differences of opinion following this linear model, but from the fact that many people start from the other end and try (often in vain) to find justification from details for their belief that they are right in their conclusion. Here the basic policy attitudes or political philosophies have a strong role. People with libertarian way of thinking are absolutely convinced that those who try to use science as an argument to promote strong government control are wrong. Leftist ecological attitudes may lead to opposite conclusions.

    While I do not necessarily share their political opinions, I have learned to accept that it is very often wrong to condemn conclusions as automatically wrong, when they cannot be supported by correct linear arguments based on science. Even for natural scientists it is often easier to reach correct conclusions about the whole than to justify it by detailed analysis. For people without scientific experience this is true much more generally.

    The scientists may err exactly, because they want to base their conclusions on linear model, when there is a wide gap in the reasoning. Such a gap may be more difficult to handle than to jump directly to the conclusions, which may have support also from other directions or is more applicable for correct intuition.

    The science is settled on very many details and arguing on them is sometimes frustrating (but sometimes entertaining). It does not seem easy to get very far on the climate issues by continuing small steps towards better understanding of science, although spreading understanding of science is always a positive activity. But so is a positive activity to improve the understanding of scientists on the nature of their own activities. This applies to the interface with decision making, but it applies also to the deeper understanding of incompleteness of knowledge and uncertainties in the science itself and in the much slower progress in the well established scientific knowledge.

    • well said

      • Judith

        As I see it we have a large room. Some argue that there is a large Elephant squatting in the middle of it that will affect everything that anyone in that room wants to do. On the other hand we have a group of people who say that the creature in the room is no bigger than an insect and will have no impact whatsover on what the occupants of the room choose to do. Who is right?

        One of the last things that Hubert Lamb ever wrote was this-in connection with the reprint of his Book ‘Climate past and present’;

        “The idea of climate change has at last taken on with the public after generations which assumed that climate could be taken as constant. But it is easy to notice the common assumption that mans science and modern industry and technology are now so powerful that any change of climate or the environnment must be due to us. It is good for us to be more alert and responsible in our treatment of the environment, but not to have a distorted view of our own importance. Above all, we need more knowledge, education and understanding in these matters.”
        Hubert Lamb December 1994

        Having studied both the science and the histiory my present belief is;

        * Climate scientists know far less about the mechanics of the climate than they think they do.

        *In discounting natural variability and assuming linearity they come to a belief that Co2 is a much larger driver than it appears to be and that modern day ‘warming’ proves their position. (which is not to say that physics doesn’t demonstrate that it imparts some measure of warming)

        *They place too much reliance on data-Land temperatures/sea temperatures in their modelling. This basic data should be seen as more of a broad guide than a scientifcally accurate measurement accurate to tenths of a degree.

        It is my belief that Dr Mann’s work-notably with the hockey stick- has fundamentally influenced the debate as ‘proof’ that Co2 is much more important than natural variability. That it influences the Met office-the single largest contributor of funds and expertise to the IPCC is evident as they say this;

        Extract “Before the twentieth century, when man-made greenhouse gas emissions really took off, there was an underlying stability to global climate. The temperature varied from year to year, or decade to decade, but stayed within a certain range and averaged out to an approximately steady level.”

        http://www.metoffice.gov.uk/climatechange/policymakers/policy/slowdown.html

        I have askedthe Met office for reference to their studies three times but they won’t tell me.

        Natural variability is perfectly easy to discern-it brought down empires as Al Gore remarked in his book ‘Earth in the Balance’. It caused migration of peoples. It has had a fundamental impact on Humanity. Yet it is being ‘denied. One can only assume that if the influence of natural variability was accepted that it weakens the notion that it is Co2 that has the overwheming impact on our climate.

        The hockey stick has its graph upside down (don’t forget it only covered the NH)-far from temperatures declining throughout the 17-19th centuries it gradually rose. So the uptick in modern times is a continuation of a trend we can trace back to 1600-not some new phenomenen. It is exemplified here , but there are lots of other instrumental data and records that shows the generality of the trend

        http://c3headlines.typepad.com/.a/6a010536b58035970c0120a7c87805970b-pi

        What I find interesting is that we seem to have these periods that are characterised by warmth or cold-such as the Roman warm optimum the MWP, Modern warming and by the Dark Ages cooling and the LIA. But within it were periods counter cyclical-for example there are many records of cold during the MWP and many records of warmth during the LIA.

        The historic events were self evidently natural variability in action yet we seem to want to sweep them under the carpet.

        Like Lamb, I believe we need to keep our influence in perspective but that we need to be aware of the environment and there are meeting places between the two ‘sides’. For example the question of energy security should be paramount in our minds as a mutual goal for all those who accept that we need to develop alternative forms of energy if for no other reason than much of our existing sources of fuel are in the hands of often unstable nations, many of whom don’t like us very much.

        So to me, when someone asks if I accept that co2 warms, I will say yes but think its impact is hugely exaggerated. In turn those propounding their belief that Co2 is the main driving force of climate need to be rather more humble and accept that natural variability is huge and in consequence put their calculations of the impact of Co2 into a wider context.

        tonyb

      • tonyb,

        Giving opposing arguments proper weight is the big problem. The rapid polarization of the discussion led to a situation where open communication has been disturbed, and this makes the issue even more difficult to solve. While I give little weight on the existence of purposeful misrepresentation of facts by the scientists, the signs of bias are obvious in many places. The signs are clear enough to make me convinced about its reality, but too indirect and vague for estimating, how much it can finally influence the conclusions.

        The scientists are not helping very well people with an attitude like mine in making better estimates on the bias or the reliability of their conclusions, because the polarization has reduced their openness in discussing the problems encountered. They are afraid that openness from their side would be misused by their perceived opponents, and there is definitely same basis for this fear. How ever well placed this justification is, it makes it difficult for scientifically literate outsiders to judge the situation well.

        The hockey stick serves as an example. I believe that it was the result of sincere research activities where many problems of the analysis were taken into account. The criticism of McIntyre and others was based on a real weak spot of such analysis. The demonstration was simplistic and overstated to the extent that researchers had valid counterarguments. The critic’s central claim is in my interpretation that all attempts in analyzing the distant past by the present basic approach of proxy analysis runs a serious risk of suppressing real signal and giving a too smooth view of the past variations as well as too narrow error bands for the results. I believe that this is presently admitted by all parties, but again the strength of the bias is difficult to estimate and still undecided.

        The large climate models are another major subject of discussion. Here my largest concerns are related to issues that influence all models and cannot therefore be analyzed at all by looking at the agreements and differences in their results. Such problems originate from the requirement that the models must give some results and not diverge or fluctuate too randomly. Making models of very large systems stable enough for calculations limits the choices of modelers severely, perhaps to the extent that the results end up being seriously wrong. I am sure, modelers have looked at this problem, but again I lack access to their discussions and to the arguments that they may have beyond wishful thinking in favor of the validity of the models.

        The models have been tested against many details of the resent climate, but it is difficult to say, how much the results prove about their validity in forecasting a significantly different state of climate, and the whole point is to have knowledge on significantly different climate states.

        The problems that I have in learning more about the state of climate research, may unfortunately influence negatively also the actual research. This is the case, if there are problems of communication also within the science community itself or if the choices of research hypotheses are influenced in a biased way by the situation.

    • I agree completely with this analysis. Both approach are used in science imho, analytical ones, bottom to top, but alo holistic ones, based on some form of statistical arguments and conservation laws. Analytical approach is the classic one. The hollistic approach was traditional only resorted to when dealing with complex system where fine understanding was lacking. I feel that, since the end of 19th century, the hollistic approach has gained tremendously, up to a point of maybe making the bulk of physics. Statistical thermo, QD are prime example.

      I think that the second slayer article is an attempt to examine the earth climate along those lines. It is more an early start that something that will lead to prediction, but early start are worthy too.

      I am less sympathetic to the first slayer article, which in a way, is more analytical and try to fault (or reinterpret? after 800+comments, I am still not sure) a fundamental law obtained from a statistical approach. Given the experimental validation of black body radiation (which was a heavily investigated subject in the early 20th century), a reinterpretation is possible, but as emphasizing analytical against statistical, it goes somewhat against the times. If quantitative difference are claimed, better for them to be extremely low, or it is assured failure.

      C.J. has provided radiative heat exchange formula that differ significantly from S-B, except for small T differences between 2 black bodies. His failure to clarify either if his formula was a limit case valid only for small T difference, or S-B was false, while simultaneously ignore all attempts to use his formula in a numerical example (or provide himself such an example) killed his theory for me. The sidestepping dance when presented a pratical example of low freq radiation heating a high temp body (microwave oven), while his theory was supposed to prevent such phenomenon (or not? again, no clear-cut response on this) was icing on the cake.

    • Pekka,
      Excellent comment.
      I agree. Moreover, I would say there are probably a lot of people out there like myself, who know from every day experience in industry that you get the best decisions, if you separate the decision making process from the fact finding process and assign separate groups of people for it.
      Mixing these two processes and assigning the same people for it, results very often in biased decisions.
      This issue is of course not limited to climate change, but to all political areas and common in everyday live in industry.
      Therefore one needs to respect, if people get doubts, when they spot that people who present the facts, are also strong proponents for a certain political solution.
      Of course they are entitled to their political opinion and are invited to express it. However, they should not be disappointed, if it raises doubts in other people. This is just a healthy reaction within our democracies. People are free to express their opinion and their doubts. Our laws guarantee that this gets respected.
      Best regards
      Günter

      • good summation from Pekka and some wise comments from Guenter. Climate change has been the meta-narrative for environmental policy for the past 2 decades. Sadly it has suffered from the issues described by Pekka and Guenter.
        The issue we skirt around is that advocates for the seriousness of climate change as a policy issue do not perceive there to be any problems with science advocacy versus its role as an honest broker (to use Pielke Jr.’s terms), whereas those who consider the role of climate change to be both less dramatic and non-threatening err towards questioning the veracity of the science, when often they also question the role of science as an honest broker. Mix in a propensity for alarmism (an inherited trait of environmental communication, see Breakthrough) from one side and a propensity for libertarian objection to all things centralized and government on the other, and we have an often conflated mish mash of he said she said contradiction rather than intellectual discussion.
        As much as Claes appears disingenuous on one end of the spectrum, I would suggest the rejection of “settled science” by Gavin to equally as disingenuous. That so many people posted and so much energy was expended on that particular non-issue is to me the most frustrating and depressing illustration of how far we have yet to progress.
        What this blog is doing I would contend is airing out many of the issues and allowing those that wish to learn from one another to agree on
        1. what is not disputed
        2. what is and is not known, and why
        3. what might be evidential and or logical substantiation of various assertions
        4. what might bring greater unity of perspective.

        What this blog has not done (yet) is address some of the more axiomatic presumptions related to the non-science aspects of climate change. That is not its intention: it is focused on the science.

        But even if certainty did exist on the nature and extent of climate change, its possible impacts and their magnitude, it would not perforce determine our response.
        In simple terms, we can adjust the hazard or we can adapt to it. We a global community and some may prefer and be able to adapt far easy than others: part of the problem of the IPCC approach has been its presumption of a universal, one size fits all response to modifying the hazard — that has now unraveled, first at Copenhagen and now completely at Cancun.
        That does not mean the questions of climate science are any less interesting. It just means their use as a political contrivance is over (or at least, significantly more circumspect than prior to Climategate).

    • I always appreciate your critical analysis and social understanding when I see your comments about the activity of science.

      I think it’s possible to engage pragmatic and democratic responses to climate change, despite the challenges in relation to the activity of science in a highly bureaucratized, power divided society.

      Are you still teaching? Maybe a nice connection for students in your part of the world with students here in Canada is The Bubble Chamber blog (University of Toronto), part of a policy and science working group with a critical perspective informed by philosophers of science and historians, along with scientists.

      • Martha,
        My latest lectures where given in spring 2010. They were introductory lectures on energy (including something on climate) for first year university students. There are no further plans, but who knows, whether something will still come up. Probably not in the regular curriculum, but more possibly on a special topic.

  11. I learned, or was reminded of, how if your creativity would spawn some almost too heretic stuff, you’d better learn precisely what would trigger norm-group hostilities before you bring it up with it, unless I guess, your goal with bringing it up would be to just spread it regardless.

    The other way around, which is perhaps why many do not enter such a discussion, is that a counter-respondent would want to be sure about its position, unless perhaps he/she is immersed in it since too long anyway already for it to matter.

    • I did a lot of heretical derivations in physics. Then I checked them against reality and gedanken experiments to see why they were wrong, USUALLY found where the error crept in (this can be very very subtle), and accepted I needed to do something different. There’s heresy when the mainstream physics is wrong and you’re right, and heresy when you’re wrong (no matter the state of mainstream physics). CJ’s derivation is wrong.

      The issues involved aren’t as intuitive as they seem – famous physicists and mathematicians wrestled with coming up with a tight proof of Kirchoff’s Law for about 70 years.

  12. Judith,

    Many experiments miss the point that we are under pressure and this planet rotates. So, there are energies generalized and NOT explored due the “science is settled” mantra.
    The sun rotates and the energy curves as it hits this planet at different angles and a moving, rotating planet. The experiments also missed we are under pressure and in an electric field of energy all around us due to the electro-magnetic field. Our intake of food has to have water and minerals of what this planet was created from.
    I have yet to find a generalized law or theory that can survive being brought to the past when the whole composition was different and the planet was rotating faster.

  13. Judith,

    There has been a great deal of lower pressure anomalies that are occurring.
    http://wattsupwiththat.com/2011/02/05/some-interesting-thoughts-on-antarctic-peninsula-warming/

  14. Here is my response to Lord Monckton’s kind letter to me cited in the intro to this discussion by Judy:

    Dear Lord M
    If you believe that simple laboratory experiments that even children perform, can say something meaningful about global climate, I think you should reconsider your message. What is it that makes you so heavily oppose
    criticism of CO2 alarmism based on mathematics and physics? I really ask you to answer this question. Claes

    • Its called the scientific method. If school children can make a measurement that refutes your theory, then the problem is with your theory, not the school children.

  15. Fine Judy: But where are all the students and children eager to debunk my math? Or the physicists?

    • Tomas is a theoretical physicist and very effectively debunked your mathematics on the ‘Slayer Part 1’ thread. You still haven’t answered his specific points with anything other than handwaving. There is no need for anybody else to join in. The job is complete I’m afraid.

      • Having read the entire 3 threads on the subject, I’m in complete agreement with this sentiment. Well said.

    • The students and children have better things to do with their time. My students are scientists; they understand that they can debunk a theory about what goes on in the natural world by making measurements. For the hundredth time, your insistence that there is no infrared radiation from molecules such as CO2 that result in a measurable heat flux (from a sky pointing infrared radiometer) in itself is a refutation of the implications of your mathematical analysis. With regards to the correctness of your equations and your math, Tomas Milanovic’s identification of numerous substantive errors in your main equations is basically a death knell for your analysis.

      • Judy, I have rebutted Tomas’ objections, about dimensionality, the physical meaning of the terms in the equation, and the Fourier analysis. What more can I do? If you would let me talk to your students, I would be able to tell them some truths about math and physics. Are you willing to let me do that?

        With the battery of scientists you line up, I think it would be more fruitful to speak to students and children.

        There are several aspects in my analysis which a serious mathematical physicist could question: the finite precision computation, the intricacies of the Fourier analysis, the same temperature of all frequencies, the forcing, and more. But none of these essential points have been even touched by your battery, only inessentials reflecting confusion.

      • Claes,
        You have not rebutted the explicit errors that you have in your paper. You claimed that you did not understand a point that should be understood by every student after first course of differential equations and you did not answer anything to the second serious error indicated to you concerning basic integration by parts.

        This is a total failure.

      • I agree: Total failure.

      • Claes,
        I keep on reminding you that your basic equation is in doubt, as it allows for exponentially growing solutions and that the next step from this equation is based on clear mathematical error.

        You have written a paper, that is full of errors, holes and unjustified statements. It cannot be used for anything without a careful rewriting. If you believe that you can do that, please do, but do not claim that that paper shows anything as it stands presently.

      • My take on the discussion, particularly towards the end, was that equation (4) needed to be reworked and documented. However, the idea itself, if all the squiggly math can be rehabilitated, was yet another way to describe a phenomenon that did not invoke statistical physics (so much) and preserve a Thermodynamics approach. It was also suggested that the outcome would likely be the same as the present model development. This latter suggestion helped nudge me further along. Yet, I truely await a new and improved equation (4). We shall see what we shall see.

      • Everyone (except CJ) thinks that if the mathematical argument were correct, it would give the same result as the standard treatment. An interestinc exercise in maths, with no physical consequences. And it most definitely does not make the infrared emission from CO2 disappear.

    • A simple experiment with a thermos will debunk your math. You have an impedance mismatch with the rest of the world Claes. If we want to argue with you we can’t use words, can’t use dimensions, can’t use 20th century physics, must have the right occupation, can’t mention experiments. What else? Give us the exact spec for how to engage with you so that you will listen. If there’s any window open we will try to crawl in.

  16. Yes there is a big door to enter into a discussion with me: Read my math line by line and then start to question it. Again: I analyse a math model of radiation used by Planck and replace statistics with deterministic finite precision computation. Take this as a starting point of discussion, not endless other things of which I say nothing. As with radiation, emission without any form of reception means that the interaction is zero, and this is my experience of the discussion.

    • I give up.

      • How to win an argument:

        1) Make one or more ridiculous statements
        2) Side step definitive counterarguments
        3) Obfuscate
        4) Repeat until critics give up
        5) Claim your statement is now definitive
        6) Publish book
        7)Profit?

      • Please don’t give up.

        You asked for reactions from infrequent visitors like me. The wider climate science industry seems to be based upon the interesting and important proposition that anthropogenic greenhouse gases somehow modify the operation of the earth’s natural climate feedback processes. Processes that self evidently through history have kept natural variability within boundaries of survivability. When looking for evidence of something to be worried about I look first at long term characterisation of variability and the plausibility of the nul hypothesis that climate observations fit well within natural variability outcome ranges.

        But what do I know – I’m only a scientist/engineer/economist who has spent a career making decisions based on data, computer models and risk management. More constructively, I suspect a focus on data integrity and transparency, and a willingness to follow the study of uncertainty to its logical conclusions will eventually get us all to the right place. A focus on the details of the science, whilst interesting, will not serve to convince me or other agnostics that the whole soup to nuts CAGW narrative holds water.

        Thank you so much for your efforts with this blog, particularly your efforts to raise the game in matters of uncertainty analysis.

        DavS

      • Why should you give up? You have a lot of backradiation from students and ardent believers in CO2 alarmism.

        This reminds me about the debate between Bohr and Schrodinger about the interpretation of the mathematical wave function as solutions of the Schrodinger equations, which Bohr (unfortunately) won by simply repeating over and over: duality, statistics, complementarity, collapse of wave function…bla bla…until Schrodinger (sick from a flu) simply gave up, and left his baby: quantum mechanics.

      • I agreed to host this debate in the interest of settling the inexplicable debate on the the most straightforward aspect of this whole problem: the basics of gaseous infrared radiative transfer. Which theory, observations and models tell us is well understood. You don’t understand this. i give up in terms of convincing you via a rational scientific discussion.

        So I will move on and host discussions on the scientific issues surrounding climate change that are deemed to be more significant in terms of a real scientific debate.

      • Sometimes you don’t want to open the Overton window. The INTERNET has indeed proved to be a place where cranks can gather for mutual support and to the extent that they can join together they can swarm reason. For more see http://www.crank.net

      • Alien 1 to Alien 2

        “look at the dogs chasing their tales”.

    • Let me start with eq. (21). It appears to be a conclusion you’ve drawn from your math. Am I not allowed to start with your conclusions? Anyway I question it. I proposed a simple way to test it. If you don’t want to do a physical experiment you could as others have suggested apply this equation to various well-established heat transfer calculations from textbooks or engineering manuals and see if you get the same result.

      Since you have so many rules of engagement, let me give you a few of my own: You can’t now claim that I haven’t actually said anything. You can’t now claim that I missed your point because I’m addressing your conclusion and your equation. And it would be both incorrect and very bad form to imply that I am not a “real” physicist.

      • Eq (21) is just an approximation to illustrate that Nature (probably) does not
        work by cancellation of big + and – signs, but rather by net flux of energy directly.

      • Can you explain why your approximation works better than the standard approximation in explaining any real-life heat transfer situation?

      • Evidently you never took General Chemistry. This is exactly what happens in a chemical equilibrium where there are silmutaneous forward and back reactions. You are really ignorant about the physical world, and dangerous to others.

      • It starts to appear more and more likely that some people, who know perfectly well the facts, are systematically and purposefully using the dogmatic and by now largely obsolete definition of the concept of heat to confuse discussion. I cannot really understand their behavior on any other basis.

      • Pekka,
        I do not understand why one let them do that, if one can use the unambiguous and not outdated definition of radiative heat transfer. Radiative heat transfer is the net effect of the energy exchange between a hot and cold body via radiation.
        Regards
        Günter

      • Günter,
        What I mean by outdated or obsolete is that the importance of defining heat through a formally specified form of energy transfer was important when thermodynamics was discussed as an axiomatic abstract theory without any reference to the statistical or micro physical specification, what heat or energy or any other of the variables referred to. Presently physics is understood starting from micro level and deriving thermodynamics from that using mathematical statistics.

        Now there is an description and internal understanding of these components and the axiomatic approach is rather obsolete. This does not mean that the formulas of classical thermodynamic would not be of practical value, but now they do not have the same role as cornerstones of physics as they had before Boltzmann and to same extent a bit longer.

        With the present understanding it makes full sense to discuss simultaneous opposite and canceling flows of heat (or radiative energy) and denying that is not an attempt to improve understanding of physics but is rather used to create confusion by purpose.

      • Pekka – blue-ribbon level. Excellent.

      • Pekka,
        I am sure you understand all the physics.
        However, I disagree, one should not use heat and radiant energy as synonyms. Radiant energy is not the same as heat. A heating rate per example in radiative transfer theory is the divergence of the radiant energy flux according to Goody and Young. It is according to Goody and Young, the difference between the absorption from the mean field and the mean emission. So heat transfer between two bodies or a heating rate of a layer or a surface is always a difference between two radiant energy flux.
        Regards
        Günter

      • Günter,
        Heat is not synonymous to energy, or to radiant energy, but there are very many occasions where the difference is not relevant or where it is not obvious, which is the better word.

        Concerning typical radiative energy transfer as discussed here the energy is before transfer as heat (i.e. energy that is part of the local thermal equilibrium) of the source material, then it is converted to radiation, moves as a photon, is absorbed and almost immediately heat in the receiving body. There is really no strong reason to care much, whether one uses the words “energy transfer” or “heat transfer”.

        I implied already above that in the present thinking heat refers to to energy that is partitioned in local thermal equilibrium between all degrees of freedom in accordance with Boltzmann distribution and other laws that determine the statistical occupation of various energy states.

      • Pekka,
        This is good that you clarified that energy and heat are not synonymous. Thanks. Clarity and precision always pays off in science and especially in education. I only recommend distinguishing between energy and heat, when explaining the mechanism of the greenhouse effect. But, nobody needs to do that. However, I strongly think that many discussions in the climate change debate could have been avoided, if energy and heat would have been used precisely.
        Regards
        Günter

    • Claes Johnson,

      I am nasically a denier, bought the book and readt it, and found it interesting. As I do not have the ability to do the math, I cannot pas judgement on these things on my own. I found the 1st thread disappointing. This is mostly due to my ignorance.

      What I would suggest is the usefulness test. If something new is useful, that is if it explains things better in some way than the old, that is another way of proving its worth, and, in my limited opinion, the most important way. Can you show us how your work can be put to use and explain things?

      • Claes Johnson

        There are many things you can compute from a wave equation model and
        results are on the way.

  17. Judith,

    By all physics, the salt changes on top of the oceans should not have occurred and yet no one has investigated it by science as to the physics of how it could happen.

    Here is a WOW factor.
    Almost ALL of the northern hemisphere is currently covered in snow and ice.
    http://www.dailymail.co.uk/news/article-1353073/Winter-storm-Map-shows-Northern-Hemisphere-covered-snow-ice.html

    And currently most of the northern hemisphere is still under cloud cover!

  18. > Almost ALL of the northern hemisphere is currently covered in snow …
    > most of the northern hemisphere is still under cloud …

    “most” of what you see on a Mercator projection isn’t there in reality.

  19. Did you learn anything?

    I consider Tomas Milanovic’ quote:

    Anyway it is just a theory I developped from observation of climate debates during the last 10 years or so.
    But if it is true, then I would advise to AGW defenders to never ever use the words “GHE or CO2 etc heats the surface” and use instead “GHE modifies the cooling rate of the surface”.
    It is just a detail but I am sure that this detail would have avoided 90% of the polemics which are only due to an incorrect and/or misunderstood use of the word “warm”.

    as an eye opener and well worth a separate blog.

    • The quote is incorrect. During the day, the surface is warming, not cooling, and the GHE effect from CO2 further warms the surface (raises its temperature). At night, when the surface is cooling, the GHE effect can technically be thought of as reducing the cooling, but at a molecular level, it is actually adding energy to the surface rather than causing the surface to emit less. The net effect is less cooling, but the result is still a warmer temperature than without the greenhouse effect.

      • A further small point. Although the GHE always creates a warming influence – adding to the solar warming during the day and offsetting part of the night-time cooling – it can be argued that if one looks not at all energy sources, but only at the infrared (IR), the net energy transfer is from the surface upward, and back radiation merely reduces the imbalance. Even that argument is not strictly correct, however. In some regions, downwelling IR exceeds IR emitted from the surface, because heat loss from the surface is dominated by evaporative transfer of latent heat.

      • In case you need it, I’ll give you more proof that I am a buffoon.

        I don’t believe CO2 resonance has any affect on the earth’s surface temperature Tpeak (except to reduce it) and I don’t believe CO2 resonance has any affect on Tavg (integrated over as many 24 hour periods as you care to).

      • Ken – Here is my response. It has nothing to do with buffoonery.

        On the first “Slaying” thread, many of us who are well versed in both the principles and the observational evidence explained why the greenhouse effect is not only real but substantial – you may recall my invitation to discuss what happens when a very large number of floating passive black objects are brought to surround an active one. The explanations are correct, well verified by extensive data, and not in serious doubt, even among knowledgeable individuals who express skepticism about other climate issues.

        We are all fallible human beings. My conclusions and those of others I cited could be wrong. So might yours. It’s extremely unlikely we’re wrong, but it’s possible.

        Here’s the problem. If we’re wrong, we will go back to our ordinary lives, sulk a while, and move on. You, however, have proclaimed it your life’s mission to demolish greenhouse warming theory.

        That’s scary, Ken. If you’re wrong, your life’s mission will consist of pursuing a phantom. That’s something to think about. I say that not only because you are wrong in my considered opinion, but because I don’t need to be a psychologist or a mind reader to believe that some part of you already suspects that what we are telling you is correct.

        Do you want to pursue that mission anyway?

        You already know what I would recommend. Don’t take my advice, however. Ask your wife to review all the exchanges in these multiple threads and then tell you what she thinks you should do with your life.

        In any case, I wish you good luck.

        Fred

      • Hi Fred. I discounted your “many passive black bodies in space” thought experiment because I don’t see how it correlates to the surface of the earth surrounded by rarefied, IR-active CO2 molecules. In fact, I would take the opposite of the point you’re trying to make. That is, I analogize the many black bodies with the earth and a analogize the center black body with CO2 and I wonder how much the passive center mass can influence the temperature of the heated many. The answer? Not much. Immeasurable. I jump to a conclusion every time, fair or not. If it’s not measurable, I don’t care about it. There are an infinite number of things in our world that are not measurable. How you pick and choose between them, I’m not sure. But, have at it. Have fun.

        I look at the thermal mass of the earth (particularly water) and the thermal mass of the rarefied CO2 and I wonder, how can it be that the CO2 has any measurable impact on the earth’s surface? I know things with mass and delta-Ts radiate. I simply don’t know how you get any measurable work out of the radiation. It would take a huge amount of radiative energy to move the earth’s surface temperature. The sun is huge and its hot-hot-hot. It can do it. CO2 is small-small-small and there isn’t much of it. I look at the forth power in SBL and I say to myself: temperature has a huge affect on radiation field strength, but radiation field strength has minuscule affect on temperature.
        When you’re looking for a lever to move the world, you don’t pick the short ones, do you?
        You could try to convince my wife, or my mother, or my kids to agree with you and to disagree with me. That would be funny. E-mail me privately and I’ll give you their e-mail addresses. I am an engineer. My family lives in the real world. I don’t tell them what to think. They think what they want, but it’s not much different than my take on things. My daughter is married to an engineer. My wife knows what a 100W IR lamp feels like when it’s pointed at her head. My mother knows the climate has changed dramatically over billions of years…when there were no SUVs on the road or coal-burning power plants keeping her warm in the winter. My son is smarter than me and doesn’t believe any formula unless it’s been sanity checked first. Good luck.

      • Ken,
        You must know that those basic things that you list are taken into account in every estimate of the warming. You do not need to believe that the final results the climate scientists present are correct, but it is trivial for any engineer to check that your argument as it stands is worthless. The real numbers do not support your conjecture to the extent that they would support your point.

        This is something that you should do yourself, if you want to know the answer instead of wanting to ignore the answer and continue making such claims in public that you could not do honestly after finding out the correct answer.

      • Fred, you are much nicer than I am. I respect your patience but it’s quite clear you can’t convince people who make it their life purpose (or job) to not understand it. What is particularly scary is that Ken’s mission is aimed at demolishing something he doesn’t understand the first thing about. Until now, it has been rare to see such outstanding ignorance in radiative transfer and planetary climate, and to think the intellectual laziness is so extreme as to be unwilling to even crack a textbook on the subject.

        What I don’t understand is why this level of jibberish gains so much attention on the internet. I respect Judith Curry’s attempts to open “dialogue” about every new odd theory that people getting drunk at a bar come up with, but if you’re too lazy to even learn the early undergraduate level stuff then it’s just disrespectful to hope other people will pay attention to it, or to even waste the time of journal reviewers.

        People need to learn that this is science, it’s a tough discipline and it’s harsh to work in. I could see many people are intentionally trying to mislead people, and some are flat out confused. I won’t voice judgment on Ken’s case, but virtually everything he is saying on radiative physics is incorrect.

      • Chris, in case you haven’t made the connection, Ken Coffman is the publisher of the Sky Dragon book. He is not wasting the time of journal reviewers, rather he has a clever marketing plan for the book and it is selling reasonably well.

      • Judith, I have made the connection. I’m sure you know already but I for one don’t think that this book selling well is a good thing. It just generates confusion, and book buyers deserve better than either a) intentionally misleading or b) intellectual sloppiness in understanding the most fundamental physics. Regardless of his affiliation, it took two responses of Ken here (to me) to realize he doesn’t understand anything about the subject.

        Why would such a person be involved in promoting/publishing an entire book on the matter?

      • I agree that this book is not a good thing. If you scroll for his comments on the first dragon thread, you will get a sense of his motives.

      • Thanks for the help, but having read your contributions on this and other topics here it wasn’t at all necessary. Happily, nature doesn’t care what you believe. Unhappily, the world is teeming with other buffoons who will buy your nonsensical book.

        Oh, and it’s ‘effect’, not ‘affect’. I’d hope a publisher would have more command over language than science but apparently not.

      • You missed my misspelling of fourth above, which is more proof that I am a buffoon, in case you need any. Forth, fourth. Darn it. Just when I thought I was making such lucid points…then it all came undone. So it goes.
        I have a five-string too. Perhaps you’ve heard my Hockey Stick Blues? I think it turned out pretty good.

      • Wow, you got me. You were the one who introduced buffoon as a descriptive term. If the shoe fits…

      • Am I supposed to act surprised that progressive activists think I am a buffoon? Actually, thirty years ago, I would have been crushed. Fortunately, I’ve toughened up since then.

        http://kencoffman.blogspot.com/2010/12/ken-coffman-is-buffoon-banned-at-gather.html

        In case you missed it, here’s The Hockey Stick Blues (with five-string bass, though I’m not playing it):
        http://www.gather.com/viewVideo.action?id=11821949021918437

      • Interesting that it just goes back to politics every time. For the record, I myself am a long way from being a progressive activist. Nor do I have a dog in this fight professionally as I’m not a climate scientist (though I do have advanced degrees in physics and astronomy and have specialized in spectroscopy from a theoretical and experimental perspective during my scientific career).

        I’m a lurker who spoke up because our host asked us to do so. I’ve been following this blog with attention since shortly after it began (though I have almost never visited the other popular/active blogs on this topic). I have learned a lot in that time, and my understanding of AGW has become significantly more nuanced as a result. My opinion of the ‘Slayer’ book, however, is obviously not.

        And by the way Mr Coffman, my nickname here (which you seem to delight in mocking) refers to banjo, not bass. I wish you much enjoyment in your own musical endeavors.

      • I’m not mocking you, I’m trying to make a personal connection. My assumption that five-string referred to a bass guitar is my mistake.
        I like the banjo too, is there anyone cooler than Bela Fleck?

  20. Alan Sutherland

    Judith

    Thanks for the opportunity to comment on the matters you raise. Your blog allows each piece of the climate jigsaw to be exploded out for examination and then put back together as tested components fitting into a whole. Much like a car engine reconditioning.

    My difficulty with the “Slaying The Dragon” topic is so many comments refer to the hypothesis of back radiation and downwelling from green house gases as though it was proved by Physics. I look at the real world to see the effect and have noticed many temperature reading sites which have shown no warming over a century. It is tempting to dismiss these sites by burying them in the average, but lets explore a little deeper.

    If GHGs are well mixed (an IPCC assumption) and the physics applies at that site (why wouldn’t it?), then how can any particular site/s not show warming over 100 or so years? It would have to be physically impossible. Remember, 100 years is climate, not weather. Yet, scientists effectively ignore this phenomenon by averaging it out with all other sites. If there was any place on earth where the effects of gravity were absent, scientists would swarm to that place to discover its secrets, and certainly not say that on average gravity still works.

    Possible explanations for the lack of warming are numerous:-

    The physics is wrong. Or is of so little effect it is drowned by other factors.

    That site is more or less cloudy than other sites and this might change the back radiation

    The error bars around temperature measurements at these sites nullify the readings, but that raises the question as to whether measurements at other sites are accurate. Is the science just based on choosing the numbers you want to fit the theory?

    Climate scientists have not yet got around to adjusting the temperatures at sites which show no warming.

    So until some thought is given to the rogue sites (and there are many of them) then the physics is a test tube experiment with unproven application in the real world.

    Alan

    • Alan Sutherland

      I liken your rogue sites to the motion of molecules in Einstein’s teacup.

      There are so many molecules in any instant influencing the position of the globule of milk fat that it ought, by the statistical notion of central tendency, never move.

      However, we see milk globules in warm tea or coffee, and dust motes in warm air, and many examples of small particles in fluid media, where the movement clearly happens.

      We must alter our interpretation of statistics to explain what is actually seen.

      You see rogue sites that haven’t warmed in a century.

      It wouldn’t surprise me if there were many ‘ultrarogue’ sites that had cooled markedly, though I recall none.

      So?

      Some sites not warming does not overcome the observation of those that have.

      It’s a chaotic system with many idiosyncrasies.

      Declaring that the behavior of some sites must overthrow the whole of the trend, any more than declaring that one decade must disprove the theory, no less than declaring because one molecule does not influence the milk globule to move in a direction that the overall globule therefore will not move, is rampantly irrational.

  21. Judith,
    You asked from some thoughts from lurkers. I am pretty new to all this and in full disclosure say that I do not believe that the rate of warming due to Co2 has been quantitatively established,even to the extent that we could say that a slight change warms or cools.
    On the other hand I do believe that radiation models radiation, absorption wave length dependence, Kirchhoff law, BACK RADIATION, etc are all quite accurate as generally used by the climate community.
    The problem that I have with climate scientists is the propensity to oversimplify (particularly when they talk to the popular press) and treat their oversimplified models as valid because they are ‘basic physics’, or ‘simple physics’. A one dimensional, spherically symmetric, static model can be explained by ‘simple physics’ but the world climate system cannot. Please be more careful with the extrapolations. The fact that Co2 absorbs radiation is a fact, but that fact cannot be used like a basic conservation law such as Conservation of energy.
    A recent example showed up on another thread where a prominent scientist is quote as saying (I am paraphrasing) that ‘it is all about radiation and we have very accurate radiation models, therefore we are confident in our models. Well, the final transfer from the earth system to space is ‘all about radiation’, but the transfer from the Earth surface to space is clearly not ‘all’ about radiation it is probably not even mostly about radiation if you consider that the much debated back-radiation (at the surface) cancels most of the outbound in a way that is fundamental because of Kirchhoff’s law.
    The critics sense that something must be wrong because the models are not (empirically) working too well and they jump, mistakenly, in my view, on radiation models, which are arguably the most accurate element in the whole debate. This poor use of blog space. There is plenty of empirical evidence to support these radiation models.
    The very fact that we focus on average temperature changes of tenths of degrees when the regional temperature are thrashing around, relatively speaking, by tens of degrees, is a problem. Ocean currents are , winds, and storms are sloshing heat around in great quantities . This is definitely not simple physics and a good radiation model, which we have, is only a start. The bottom line for me is that I think that too much accuracy has implied for oversimplified models because of the beauty and apparent utility of simplicity. Unfortunately this beauty is only skin deep.

    • The models are radiative/convective models. No GCM attempts to model climate on the basis of radiative transfer alone.

      Climate evaluation doesn’t focus on average temperature changes, but rather looks at changes occurring in the large multiplicity of individual regions, taking into account seasonal, latitudinal, and other variations that affect climate response. It’s true that global temperature trends are derived from averaging individual anomalies, but regional and even local data are available and utilized in understanding what is happening.

      I do agree that it’s not simple.

      • Fred, just go look at the 351-year-old Central England Temperature Record (CET) and tell me if has any correlation with atmospheric CO2, either emitted by humans or otherwise. Likewise look at the 200-year-old Klementinum record (Prague). I see no human signal in either. No correlation = no causation. Thus I see undeniable and conclusive empirical evidence that the greenhouse gas hypothesis is busted.

    • We have had many discussion chains that were supposed to discuss radiation only (or non-feedback warming only) and leave other effects to later discussion. It is likely that the statement of a “prominent scientist” appeared in such a discussion and that the reference “it is all about radiation” has to be taken to apply to the settings of that discussion. I cannot imagine that any competent scientist would say that about the whole climate change issue or greenhouse effect with feedbacks.

  22. I was having a discussion by email with a professor of astronomy, I had read his review of “Heaven and Earth” by Ian Plimer. This appeared in the Australian some years ago, but I didn’t get round to reading it until recently. The review was so totally biased and so utterly unfair that I could not resist sending an email to the author.
    One thing I said was – science in never settled, I quoted the issue of peptic ulceration. Two doctors in Western Australia produced the idea that peptic ulceration was due to infection rather than gastric acidity. The overwhelming consensus at the time was that acid, or acid plus certain enzymes, caused ulcers. The West Australian doctors were laughed to scorn. They now hold a Nobel Prize.
    The astronomy professor replied thus. This is not verbatim, but the sense has been retained.

    Science is science. And good science will always triumph over bad science. There was no evidence whatsoever to support the acid hypothesis. The West Australian doctors did an experiment which proved that the rubbish their peers believed was wrong. They were quite rightly awarded a Nobel Prize.
    The level of consensus within the climate science community is much stronger than that within the medical community. Because of this, climate science is more soundly based than medical science.

    This is good thinking from a professor of astronomy. I don’t think that even a professor of theology could have done any better.

    Let’s us look at the man- made global warming hypothesis from the point of view of medical science…
    The system which almost all doctors accept today is the Cochrane system. Briefly, the Cochrane system works as follows.
    The strongest level of evidence is level one.
    Level one evidence consists of randomised controlled trials under extremely stringent conditions.
    The next level of evidence is level two.
    Level two evidence again consists of randomised controlled trials, but under less stringent conditions.
    Next comes level three.
    Level three evidence consists of observational studies.
    Even if the series studied is large and the correlation is tight, an observational study can never prove a hypothesis. Correlation does not prove causation. An observational study might suggest that a hypothesis is “reasonably likely” But the proof lies in the randomised controlled trial. Preferably this trial will have reasonably large numbers of patients and have the tight parameters of level one.
    Consensus gets low marks under Cochrane. Cochrane is about evidence rather than opinion. A single opinion has an evidential value of zero. Consensus might represent the opinion of 3,000 people. Multiply zero by 3,000 and the answer is still zero.

    Now let’s look at man made global warming.
    We can’t have a randomised controlled trial. We have one patient (planet earth). We don’t have even one single other planet to put in the control group.

    So let’s do an observational study.
    An observational study with one patient is not an observational study. It is an anecdote, or at best a case report.

    OK, let’s be practical. We only have one planet earth, so our single patient is very, very important to us.
    Since planet earth is so precious to us, we are prepared to accept an observational study with a tight observed correlation as being evidence enough to act on.
    We have to look at the degree of correlation over four time periods.

    There is the period 1900 to 1945. During this period the planet warmed.
    Compared to today, there was not much in the way of heavy industry. There were very few cars on the road. Not a lot of CO2 was produced… But the planet warmed. 1938 may not be the hottest year ever recorded, but it fits comfortably into the top 10. I think that it is very reasonable to say that the warming seen from 1900 to 1945 was not due to carbon dioxide. We were coming out of an ice age.

    Now let’s look at the period from 1945 to 1975. We are in the era of post war recovery. More cars are being built. More electricity is being generated. More heavy industry is starting up. More CO2 is being produced. But the temperature is in fact going down. How do we explain this? The IPCC says pollution with substances such as sulphur dioxide were the cause of the fall in temperature over this period.
    Now this is an assertion. There are no quantitative measurements of all the aerosols and other pollutants and computer models of how each pollutant would affect the climate. But lets be generous. Lets say that there is a seventy five per cent chance that this explanation is correct.
    Now we move on to the period from around 1975 to 1998. Her we have a period when a further rise in atmospheric carbon dioxide is associated with a further rise in temperature. There arte alternative explanations. It could be the sun. It could be something that we haven’t thought of yet. But let us be generous and say we are 90% certain that global warming from 1975 to 1998 was caused by man made carbon dioxide.
    Then we reach the period 1999 to 2010. We continue to produce lots of carbon dioxide. There is no statistically significant rise in temperature during this period.
    But the warmists say that this 11 year period contains three, or five or six of the warmest years on record.
    Well the temperature has been rising by fits and starts for about 150 years. We are on a high plateau, so it is not surprising that some of the temperatures we see are record ones. But are we going to stay on that plateau? Are we going to start moving gradually downwards in terms of temperature? Or are we about to take off with the climate getting warmer still?
    Well, of course, there is no way in which this question can be answered with any certainty. But lets be generous again. Let us say that the fact that several of the years of the last decade are in the top 10 ever recorded means that there is a 70 per cent chance that this period of time supports man made global warming.

    OK the global warming hypothesis rests on three lines of evidence.
    The first line of evidence has a 75% chance of being correct.
    The second line of evidence has a 90% chance of being correct.
    The third line of evidence has a 70% chance of being correct..

    If you roll a dice once, the odds against guessing the right answer are six to one.
    If you roll the dice three times the odds against you getting the right answer three times running are six to one multiplied by six to one multiplied by six to one.
    The odds against you are 216 to one.
    The odds in your favour are the reciprocal of this. They are 0.00463.
    Now we have three lines of evidence for the man-made global warming hypothesis.
    The odds in favour of each piece of evidence are, respectively 75%, 90%, and 70%
    Multiply 0.75 by 0.9 then by 0.7 and you get 0.475.
    There is a 47.5% chance that the hypothesis is correct.
    Does this show that the science is settled? Don’t make me laugh.
    I have no idea what the temperature will be like in 100 years time. Neither has the IPCC.
    (This blog has already been posted at Wattsupwiththat)
    Jim
    .
    .

  23. 0.04% CO2 content Back Radiation in Atmophere content can cause the mass of the Earth global warming? No magnitude sense!

  24. I keep hearing arguments using the words rarefied, trace, insignificant, etc. to describe the CO2 concentration of the atmosphere.

    In some contexts, this usage might be valid.

    However, looking at the elements other than Carbon, or the configurations of carbon molecules other than CO2, and thinking about their relative bioactivity and physical effects, I would have to dismiss these diminutive adjectives as ill-advised.

    What would we say were the atmosphere to suddenly contain 0.04% carbon monoxide?

    Suppose sulfur, mercury or radium permanently and increasingly permeated to the extent of carbon in the air?

    Maybe not lethal immediately, but certainly enough to grab the attention.

    Yes, the impact of CO2 on temperature is a marginal addition.

    Ask an economist about the importance of marginal additional returns over many compounding periods.

    Further, is it the concentration of CO2 that matters in this discussion, or the amount of CO2 that IR encounters between the surface and the TOA?

    Because while the planet is very big, in the past 15 years approximately 1×10^14 kgs additional anthropogenic carbon (did I do that right, just over 95 Gtons C from CO2 and other GHGs?) spread over over 5×10^14 m2 (or 5.1×10^8 km2?) is 200 grams of additional carbon IR light must travel through to get from the surface to space every square meter?

    0.02 grams of additional carbon per square centimeter?

    How many picograms of carbon does it take to turn the surface of a centimeter of paper black?

    Yes, I get that carbon and CO2 have different optical properties, and that black to the naked eye is not the same as black to IR, and that there are dozens of flaws with this illustration, however..

    Calling CO2 a trace atmospheric gas is simply babble.

    • As is comparing it to sulfur, mercury or radium.

      • TerryMN

        Absolutely so, if one takes the example out of context.

        Sometimes one wants to make an effort to speak the local dialect to show the locals one respects their culture.

      • Hi Bart,

        I used your context:

        What would we say were the atmosphere to suddenly contain 0.04% carbon monoxide?

        Suppose sulfur, mercury or radium permanently and increasingly permeated to the extent of carbon in the air?

        Maybe not lethal immediately, but certainly enough to grab the attention.

        Which I still think is a bit of a straw man argument.

        Also, can we agree that “permanently and increasingly permeated” is just a tad hyperbolic? Permanently, being forever, eventually makes your and increasingly unphysical. Every engine red-lines at some point.

        Have a good weekend!

      • TerryMN

        You missed my context, I keep hearing arguments using the words rarefied, trace, insignificant, etc.

        Cherry-picking the content free of that context-setting introduction is where you create your straw man, and then call it mine.

        As for permanence and increase, CO2 levels are pretty certain to rise throughout my lifetime; what permanence more do I need in the category of bioactivity?

        I defend the conservative and ordinary use of these terms that you quibble over, in the absence of something better to do with your time, such as contemplating maybe the actual meaning of what I posted, or preparing for the game.

        Enjoy the Superbowl.

    • Whenever I hear this argument, I must confess, it gets translated in my mind to something like this:

      If CO2 was CO, then it would be a monstrous health hazard.
      If CO2 was viper poison, then it would be a monstrous health hazard.
      If CO2 was Captain Voodoo’s magic death powder, then it would be a monstrous health hazard.

      Some things in small proportions are effective, powerful, poisonous, etc.
      So what? In contrast, there are things that are perfectly safe in much higher proportion, like Oxygen.

      If N2 was Pb, we’d all be crushed to death by our atmosphere. If I looked like Johnny Depp, I’d be a lot more popular with the ladies. If all of my pennies were gold, I’d be rich. You want babble? You got babble.

      • Ken Coffman

        I believe we come to the crux of the issue.

        You hear things in your mind.

        These things you hear, they are not there.

        You take the original from their context in the real world, and put them into the context of a (Captain Voodoo and being crushed to death by the sky?!) paranoiac fantasy.

        Between where you stand on the surface of the Earth and where the top of the atmosphere ends, the air you breath has had added to it a layer of extra new IR-opaque CO2 comparable in mass to the graphite from a pencil spread over every square inch in the past 15 years.

        This does not, in the parlance of optical properties, constitute ‘trace’.

        Dismissing this rather large change by exploiting the innumeracy of one’s audience is astounding abuse of the poor-between-the-ears.

      • Okay, Bart. Great. If you want to be accurate, how about if you stop using the word ‘opaque’. Outgoing IR radiation is not blocked. Outgoing IR radiation is not trapped. Outgoing IR radiation is not stored. What can trap or store radiation? 390PPM of CO2 is not blocking anything. You can say IR is reflected, diffused, or dispersed. You can say IR energy is dissipated (into the rest of the atmosphere). The mean path length can be lengthened. The transit time can be lengthened. It might take a long time for an IR photon to get to space, perhaps a millisecond. Perhaps a little more.

        Heat energy can be stored in dense substances that have thermal mass, then released slowly…resulting in a modulated radiation signature. But here, the radiation signature is an effect…it does not do any significant work. 390PPM of CO2 has little thermal mass.

        Honestly, I’m the buffoon, but I’m not proposing any kind of temperature-controlled, CO2-modulated radiation machine to control our surface temperature. Why don’t you patent that amazing machine?

      • Ken,

        All in all, we know CO2 absorbs IR light most of the atmospheric gases cannot, the energy in that IR light gets redistributed to the other gases very quickly upon absorption by a CO2 molecule and that some IR light is emitted back toward the surface of the earth by CO2 molecules. All of that is known from decades of lab and field measurements.

        I think there is a valid point worth making in that the explicit signatures from an increased greenhouse effect due to human emissions of GHG’s are ‘hard’ to detect. Ricky Rood showed me some slides once that he claimed were a true signature, but he didn’t really explain what he was showing, so it’s hard for me to place value on them.

        But for the sake of argument, let’s say there is no way at the current point in time to detect any signal from CO2 forced warming because the time series is too short (30 years of satellite data) and the noise is too large.

        Knowing that, what is the most scientifically rigorous statement we could make?

        I’d say: given our detection limit (combination of noise, measurement error, etc.), we cannot determine the magnitude of the contribution of human induced increases in the atmospheric greenhouse effect in the recent temperature time series.

        Continuing with our assumption of an undetectable effect, we CANNOT conclude that because we cannot presently measure such an effect, it must not exist. This would be a type II error in the your statistical hypothesis that the greenhouse effect doesn’t exist, ie a false negative.

        In all likelihood, even after many, many years of research into the future, if a true greenhouse signal didn’t come out of the noise, we could never full reject the hypothesis that the greenhouse effect, which is partly due to CO2, contributes in a meaningful way to climate at the local, regional and global levels. Because of the necessary rigor, improving the noise level in the data by increasing the time interval in the time series would simply lower the upper bound on the magnitude of the meaningful influence the greenhouse effect has on climate.

        I’m not sure if there is a real greenhouse signal in the time series data at present. It would not surprise me if there were not given the noise present upon zeroth order signal processing (looking at it). But even if there isn’t such a signal we can detect, it doesn’t mean the signal isn’t there. So because some scientists, journalists and politicians lose credibility due to their willingness to make ridiculous claims based on possible false positives, we have to make sure we don’t swing in the opposite direction and say something we cannot most rigorously prove as fact.

        What do you think?

      • I respect your viewpoint very much, Maxwell. I appreciate your courtesy and the depth of your knowledge. What we’re speaking about is personal choice and philosophy. Everyone chooses what they consider significant and what they feel safe in ignoring. Partly this is determined by the engineering trade. What we do is consider the big things first and once we’ve created a design that meets the requirements (which are often tolerance percentages including environmental changes, degradation over time, combinations of unrelated error sources, etc.), we’re done. The project is over. We move on to the next project. There is no time in the schedule or money in the project budget to explore small, insignificant factors. If I got tied up in tiny details, I would be a bad engineer. I would be wasting the company’s money. The tiny details might be true and valid…but they are irrelevant or I can increase my design margin a little bit and swamp out all of their effects. Job done.
        So, here we are. There are an infinite number of things that can’t be measured. I’ll add to this…if an effect is very tiny and very difficult to measure, forget it. I don’t trust the vested interests. I don’t trust humans to be properly objective. I don’t trust test equipment or test conditions. I ignore all of it. This might mean I miss a subtlety of climate science, or religion or mind-controlled spoon bending. So be it. A minimum amount of signal-to-noise ratio is required to be certain of anything. I have a digital signal processing reference that says 1:20 is a practical limit, but I’m generous, I’ll go as low as 1:100.
        In fact, if you don’t believe me, why don’t you take some of my money. It should be easy.
        http://www.gather.com/viewArticle.action?articleId=281474978566613

      • Ken,

        while I appreciate your trying to get this situation into terms and ideas with which you feel comfortable, I think there is something being lost in that translation.

        You say,

        ‘There is no time in the schedule or money in the project budget to explore small, insignificant factors.’

        from which the implication is that, when compared to an engineering project, as long as the ‘device’ works, we move on. There is no need to focus on possible malfunctions because they have not appeared yet (which is a stance I’m not sure all companies would take).

        In the world of signal processing of the global temperature time series, however, we don’t have that luxury. There is no meaningful analog to your engineering example in the sense you are using this analogy above. There is no ‘big’ signal that we have uncovered so far. So when you continue to bring up the fact that the greenhouse ‘signal’ is small, it’s really not as important as you think because ALL THE CONTRIBUTIONS TO THE TEMPERATURE SIGNAL ARE SMALL.

        It’s not as though we are comparing a tiny greenhouse effect on top of some gigantic solar effect that comes out of the noise. The noise in the well-conditioned data series (satellite era) is so large that there are really no meaningful long term signals we can extract via signal processing. That’s the whole problem here.

        So I agree with you assessment that we are having a hard time measuring the greenhouse signal in temperature, storm frequency and intensity, drought frequency and intensity and many other climate and weather parameters. But it’s not as though this fact is juxtaposed against some huge signal that is coming out of the noise for which we can derive the vast majority of the variation we’ve seen in climate in the last thirty years.

        Again, this is a matter of rigor, not personal preference. You are perfectly able to belief whatever you’d like about these physical processes and whether our involvement in them necessitates a specific political response. You are not perfectly able to simply compare away the meaningful nature of the true challenge we face in figuring out what the hell is controlling climate because the conclusion you reach in a specific simplification happens to conform with the political outcome you desire. That’s deceptive and manipulative. The two things that you dislike most about your perceived opponents in this debate.

        So if you demand a higher standard of behavior from climate scientists, journalists and politicians, you better make sure you live up to that standard as well.

      • Ken Coffman

        If you want to be accurate, how about if you stop using the word ‘opaque’.

        This is another of my pet peeves, being told what words to use, after they’re redefined by the spin of someone seemingly unfamiliar with the field and conventions of usage because truth is hard for them to cope with in their paranoiac fantasyland.

        To be accurate.

        To be precise, rather than also merely accurate, CO2 may be described as IR-opaque because it demonstrably produces bands in the IR range (non-saturating, non-overlapping with other atmospheric elements at approximately 2.5, 4 and 20 micrometers) when measuring absorbtivity.

        Saying it isn’t so doesn’t make it not so.

        Outgoing IR-radiation encountering this barrier, comparable to the graphite from a pencil for every square inch of sky, is — on the time scale of light, in the conventional physics model in use today to describe the activity of light and molecules — blocked, trapped and stored by this opaque barrier increasingly because human beings put this barrier there, more and more every day.

        Saying it isn’t so doesn’t make it not so.

        This blocked, trapped and stored energy escapes in random directions over and over again in a random path and/or excites heat in neighboring molecules and thence under the rules of conservation of energy the escape of other energy which does the same (during which time generally accepted convention dubs it ‘back-radiation’, apparently) until some final sink (generally space) blocks, traps and stores it all in an interval that at our human speed would be breathtakingly brief.

        Saying it isn’t so doesn’t make it not so.

        Until it finds space, this blocked, trapped and stored energy, by the best argument and evidence available to a high confidence level within uncertainty and not so far meaningfully refuted, leads to an increase in the equilibrium temperature of the climate.

        Saying it isn’t so doesn’t make it not so.

        Offer meaningful refutation. Don’t just lie.

      • Good work, Bart. You’re a good little soldier.

        Let me suggest a thought-experiment for you.

        Suppose the sun instantly turned off and at the same instant all of the heat energy stored and generated in the earth (land and water) instantly disappeared (earth and water = 3K). We all agree there is heat energy content in the air. How long would it take for the earth’s average surface temperature to reach 0C? I’m not looking for accuracy, just your general idea. A fraction of a second? A few seconds? Minutes? Hours? Days? Years?
        I’ll give you my answer…it would take quite a long time from a photon’s POV, perhaps as much as a few seconds. But, I’m a buffoon, so what do I know?

      • Ken Coffman

        You mistake me. Good soldiers stop talking when they’re lied to by civilians.

        Could you explain the meaning of your thought experiment, a little?

        From 3K to over 273K from just the heat content of the 17km of troposphere, do you mean for this to be measured at the boundary layer of the last atom of land and sea (which must be ice, no?) before atmosphere begins, or at a depth of some measureable size, to establish a meaningful sample?

        I believe by Le Chatelier, the answer is never, as precipitation of the atmosphere by parts, and the explosive structural changes of the surface of so sudden a change would make electing a boundary to take the average of impossible until substantial equilibrium had been achieved, which would be far below 0C.

        There is too little information in your question to provide a better answer.

      • Bart,

        Hyperbole does not make the discussion better:

        ” blocked, trapped and stored by this opaque barrier increasingly because human beings put this barrier there, more and more every day.”

        “human beings put this barrier there”

        What is the percent attributed to anthropogenic sources again?? Your BARRIER was there long before humans, according to consensus science, and was much larger than modern times when it was both warmer and colder.

      • kk

        Thank you.

        It’s good to have a sincere correspondent.

        What is the percent attributed to anthropogenic sources?

        As you can tell, I’m a simple guy. I talk about fairy dust and pencils, and relate the sky to one inch squares of paper.

        So, I take the simplistic approach.

        In 1750, and before that for hundreds of thousands or possibly millions of years, before the dawn of civilization almost certainly CO2 was mostly at or about 180-280 ppmv.

        Do I know this for certain?

        The chemical work on this is by most accounts very good, and the sigma level when all evidence is accumulated far, far higher than anything in the global temperature record for anything up to the current century at precisions necessary to say much useful or confidently about global warming, so this appeals to the skeptic in me as a better dataset.

        Perhaps 3 digits of precision is too great to claim for back of the envelope calculations, so 1.8-2.8 x 10^-4 parts of unity, roughly, accepting this as seasonal and regional variability and some epochal variation, coming to 2.3×10^-4 +/- 0.5×10-4 ppmv for over 99.5% of the past half million years, to be conservative.

        Now, we’re at 3.9×10^-4 ppmv, in a quarter millennium, from 2.7×10^-4 ppmv.

        That’s over a 44% increase. It’s more than double the natural variability range of the past half million years, as a single jump.

        It’s statistically implausible that this leap would be part of the previously observed dataset, and would allow us to reject the null hypothesis at something like the sigma 8 level given a plausible mechanism.

        We have a plausible mechanism.

        We did it. Human beings, human industry, human agriculture.

        We can be as certain we did it as we can be certain Earth will continue to rotate on its axis causing the apparent position of the sun to rise above the horizon in the morning.

        Certainly, we might not have done all of it; the record suggests some peaks above 3.0×10^-4 ppmv over the past half million years.

        However, Occam’s Razor applies, so we can say we’re responsible for 44% of the CO2 in the air right now.

        By our exploits and our policies of nations, we are the attributable source of this 44%.

        In the past 15 years alone out of that quarter millennium, we’ve scribbled a pencil worth of non-overlapping IR dimming in every square inch of sky.

        56% of the barrier that is there now was there before our current period, and for the last half million years the barrier was in the range of about 45% to 58% of what it is now, but that would be the baseline, the standard, the natural level.

        We’re way off the bubble.

        If asked would I for my own part consent to such a huge change to the air everything I know depends on, absent evidence of harmlessness and absent compensation for the risk, how could I as a skeptic and a person who values his own rights give any answer but no?

        How can you?

      • Bart R,

        http://noconsensus.wordpress.com/2010/03/06/historic-variations-in-co2-measurements/

        Interesting discussion by Ferdinand Englebeen and a very knowledgeable gentleman disagreeing with him in the comments. Lots of excellent reference material I will probably never make it to.

        Basically, like so much else in Climate Science, you are way too sure of yourself when you talk about a 280ppm pre industrialization “natural” level.

        Have a good evening.

      • Only two instances of “conservative”.

        The two instances are even proper uses of the word.

        Must be a very good thread.

      • kk

        *squint*

        Er, how is “2.3×10^-4 +/- 0.5×10-4” equal to 280 ppm?

        Heck, how is 2.7×10^-4 equal to 2.80×10^-4 either in value or precision?

    • Each breath you exhale contains 40,000 ppm CO2.

      Try inhaling 40,000 ppm carbon monoxide! Let us know how that goes for you.

      These type of arguments do nothing for the credibility of a failed hypothesis. Quite the reverse in-fact. It is the kind of last resort Special Pleading you would expect when all else has failed.

      It does not wash and produces ever diminishing returns.

      • Will

        What is my carbon monoxide argument?

        I know some get it, as I’ve been told bluntly that people get it, so I have to entertain the notion that those who don’t get it are especially challenged in their reading skills.

        Pull the straw out from between your ears, man!

        My carbon monoxide example, with mercury and radium — how much mercury and radium in ppm do _you_ exhale, btw? — along with the later examples of estrogen, testosterone, gold, angels’ tears and fairy dust was provided for categorization, and was scaled to ambient CO2 levels in air, not in exhaled breath. For that matter, try inhaling 40,000 ppm CO2 for prolonged periods while active.

        What is rarefied, trace, or insignificant for global CO2 levels?

        Since (unlike O2 and Nitrogen) CO2 is optically active, this would depend on the amount of CO2 in an optical cross-section as (IR) light passes through it, not on its concentration.

        Since CO2 is biologically active, this would depend on differential bioresponse in complex systems of interacting and competing species, which is shown to be significant in the 200+ ppm range.

        These cheap straw man shots you take show why so few accord you any credibility at all, one imagines, and won’t answer your feeble challenges.

        Like your incredibly ill-designed and clumsily spun experiment in a lemonade bottle, where you attempt to argue you are modelling the 17km thickness of the troposphere in the 17cm constrained by your plastic tube, and then use some of the least accurate thermometers on the market to try to track this 1/10,000th as large temperature difference, you have not produced a meaningful case.

        You appear to be using words you heard somewhere and never clarified what their definition is.

        I wasn’t presenting any hypothesis, I was ranting about the ill manners of people using flat out lies by calling what is in fact an obviously significant amount of CO2 in the context of atmosphere.

        The carbon component of the CO2 added by man in the past 15 years alone is 20 milligrams per square centimeter of sky, so the CO2 would be over 70 milligrams per cm2 — almost half a gram per square inch!; given its IR-opacities (absorbtivity bands) in the 11+/-9 micrometer range, it is coincidentally not unlike the opacity of graphite in the visible wavelengths for a similar mass on a similar cross-sectional area. Your experiment puts how much of this optical density to the test in 1/10,000th the depth of field (and under so badly constructed conditions as to be otherwise embarrassing to read about)?

        Sure, my pencil on onionskin, like your lemonade jug, is an analogy, a model intended for illustrating with more clarity the situation that cannot be seen by bringing it into terms that can be seen.

        That is the point of an experiment, to simplify meaningfully and aptly the key elements of a hypothesis or phenomenon, in order to better apprehend the mechanisms involved.

        Which would be the opposite of your experiment in lemonade jugs, Will, and too the opposite of what you do when you spin your writings.

      • Sorry Bart,

        but I struggle to understand psycho-bable. However I’ll do my best.

        My only point as you well know, unless you are being slippery, underhanded and outright dishonest about it, is that you cannot simply use arbitrary apples and oranges comparisons to make a fallacious point. It just makes you look even more dishonest. Sorry if that touched a nerve old boy but that is just the way things are in the real world and it is high time you came to terms with it.

        With regards to your criticism of my experiment, again you display nothing but contempt for reality.

        The test is simply a comparison of CO2 against Air. It is not a model of the atmosphere.

        Click the link and all can see that I clearly state that this is a “simple test, a comparison of the infrared absorption of CO2 against Air.”

        “AGW Debunked again.pdf”

        At no point do I claim to be modelling the atmosphere. To imply that that is what I am trying to do with this test is again an indication of the level of your own ignorance and/or dishonesty.

        You are employing a straw-man technique and it is a favoured trick employed by all con-artists. Somewhat over used by AGW fraudsters and ever so slightly predictable at this stage I have to say.

        For your clarity:

        The test is a simple comparison of CO2 against Air. The conditions are identical for each bottle. The temperature difference in every test I have performed is always around 1º C in favour of Air, so these thermometers are perfect for the purpose.

        If you have a criticism about the accuracy of the thermometers I have used you need to take that up with the manufacturer. I have spoken with them myself and they have assured me that these digital thermometers are spot on and that is good enough for me.

        With regards to your other scary data, the amount of Carbon added to the atmosphere each by humans is around 7 Gt per year, which is equivalent to 4 ppm. If you want to get in a flap about 4 ppm fine, but while you’re trying to convince real people of the dangers of 4 ppm, instead of recommending economic ruin and carbon slavery for the majority, while making a select few disgustingly wealthy to the tune of hundreds of trillions of dollars trading in other peoples human rights to emit CO2, why don’t you do something to mitigate this
        insignificant 4 ppm, like lobbying for tree planting projects which could easily mitigate 4 ppm, or something useful like that? Perhaps then at least, you will appear genuine and credible.

        Let me give you a hint Bart. If 4 ppm was a real problem, there are real solutions to that problem and those come in at a fraction of the cost of any cap and trade proposals. It is well accepted that cap and trade will NOT mitigate human emissions of CO2. In fact it will simply knock out all the competition for the big polluters who will continue to emit and pass on the expense to the consumer.

        Hypocritically moralising to others, while attempting to pick their pockets is a game with serious consequences. Let me know how that works out for you.

        Over the last decade have come to the obvious conclusion that there are three types of AGW proponents.

        Liars, fools, and lying fools.

      • Bart,

        I also note that bellow you have accused RiHo08 of using straw-man arguments. Yet you have failed to produce any actual evidence of that. You merely contradict him and then falsely accuse him of using a straw-man argument.

        Above I have given a clear example, hard evidence, that you are using the straw-man technique that you have, without out clear evidence, accused RiHo08 of doing.

        Let it be noted that Bart R, has serious credibility issues on these threads.

      • Will

        A double post. One is honoured.

        Let me say to your last, “Let it be noted that Bart R, has serious credibility issues on these threads,” that I prefer to call it credulity issues. However, in either case, high praise in the books of any skeptic, the source notwithstanding.

        Oh. I see, you were attempting the technique known as ‘poisoning the well,’ to establish a halo of questionability about everything I write in the hopes of inspiring rejection of my claims by careless readers.

        Well, good!

        It is my skeptical hope that people will develop the habit of questioning what they read, confirming claims for themselves by independent and objective research, and decide for themselves what to accept or reject.

        RiHo08’s straw man?

        You do know the definition of the term straw man, yes?

        I speak of surface area, and below RiHo08 substitutes concentration by volume for surface area. He introduces a new argument as if it were my own, and then attacks it. This is his straw man.

        One need not present separate evidence of a thing when the thing speaks for itself. What bizarre standard of evidence is this of yours? What better evidence than the original itself need there be, or ought a skeptic accept in replacement?

        Now, as to your so called hard evidence, wherein you .. well, it’s hard to follow what you do, though one imagines it gets easier with repeated innoculation, but it appears you claim I hold your reality in contempt (true), and that your experiment — which by definition ought model a separate phenomenon in the hopes of better explaining it — is indeed only a model of itself, unrelated to AGW.. in a link named “AGW Debunked..”

        One would be hard pressed to call this anything but the most bald-faced of lies.

        Here, let me try.

        Mmmph.

        Nope.

        Let me try again.

        Hnghrph!

        Nope.

        Still can only call it a bald-faced lie.

        For your clarity: your experiment is flawed in nearly every aspect of design, and meaningless in the context you offer it. While you may have discovered something, given that you _always_ get exactly the same result (which would make any experimentalist seriously question the whole thing in any event), and claim to have perfect equipment, the interpretation of what that something is remains dubious.

        Also remember, I am not making an AGW claim.

        I’m making a CO2 claim, and explaining a backradiation mechanism.

        I don’t predict how much warming there is or will be, so much as describing the best model of the mechanism I’ve seen laid out in terms easily grasped by laymen with commonplace experiences.

        This is not so much apples to oranges, as apples to cider, bringing a simplified version of the original to an audience for their consideration, absent pips and pectin, worms and bruises, rusts and rinds. Your characterization thus is amiss.

        How can you be honestly and sincerely unable to grasp this simplifcation as simplification, other than by wilful blindness or disingenuity? (Redundant question, I know.)

        And how is my data scary? It’s true, it’s honest, it’s simple and accessible to practically anyone. Is it that practically anyone can see truth honestly scary to you?

        And how, by comparison, is your “..economic ruin and carbon slavery for the majority, while making a select few disgustingly wealthy to the tune of hundreds of trillions of dollars trading in other peoples human rights..” not scary simply to scare, without any truth, honesty, or accessibility? Where’s your proof of ruin? What’s your slavery evidence? What the heck are you talking about that I have ever proposed or supported?

        Otherwise, why are you making stuff up like this?

        What do you know of tree planting, and have you not read the detailed studies of for how limited a time a stand of trees will act as an effective carbon sink? Perhaps you mean a scheme to grow trees through their peak growth, harvest them, and carbonify or petrify them underground as a form of carbon capture? If that’s what you mean, then say so, and spell out its costs. I’m not objecting, but if you want to do that, then it sounds like you’re a proponent of carbon slavery.

        My proposal about CO2 emission has always been directly charge a fee for emission and turn that fee directly over per capita to one’s fellow citizens in fee for rent of the common resource. This would be the few who benefit paying to the many who do not. This is the opposite of what you say I say.

        Who, then, is the fraudster conning people, with his hands in their pockets, and exploiting their rights without benefit to them?

    • Bart R:
      From a “back of an envelop calculation” and ascribing a 5 C themalization of CO2, its impact upon the rest of the atmospheric gases, and in this case for a 3% water vapor estimate concentration (1% is the lip cracking dry of the polar winters and 6% is the saturated water vapor at the tropics), a simple proportion of concentration relative to atmospheric partial pressures of the gases (@77% N2, 20.9% O2, 3% H2O, 0.9% Argon, 0.039%CO2), yields an additonal 0.00039 Celsius for CO2. CO2 is a trace gas and its impact is also trace no matter how many photons it absorbs or emits. Since photons zip along close to the speed of light, the heating and cooling of CO2, with its low heat capacity occurs really really fast. CO2 is a bit player in this inequilibrium theater and I don’t understand why all the drama, it can’t be the physics.

      • RiHo08

        So, not ~33 Celsius?

        Why does your envelope so differ from the envelopes of others?

        I’ve got a pencil that I can draw a square inch with, and fill in, over and over again page by page (Peter, forgive me for taking my pencil up again) to show me a graphically a visual light approximation of what IR encounters in the cross section of the sky it transits from ground to TOA, until the half gram of graphite representing additions by man since 1996 is all spent.

        How is this trace, how optically rarified, how IR-insignificant?

      • Bart R
        I am sure you know that within a few meters of earth’s surface, partial pressures of gases begin to decline, By 18,000 feet they are less than half and by 20,000 feet the partial pressure of CO2 is nearly inmeasurable. Relating activities to the Top Of the Atmosphere, another 140 kilometers further for a CO2 effect is obviously fictitious. If the gas does not exist through the entirety of the Standard Atmosphere, it is hard to imagine that it would have an effect. Water vapor on the other hand, does exist within the Standard Atmosphere and does play a role in slowing the cooling of the the earth. Ascribing special properties to CO2 to overcome its very puny concentration is … well, unphysical. Maybe you should put your pencil down and pick up a good book to read.

      • RiHo08

        Okay, so when faced with a pencil you don’t like, you stop making sense entirely.

        Cross section is a measure of area, not of volume.

        The dimension of altitude does not exist for the purposes of surface area.

        (Though it does for depth of field, which gets complicated, and as I can see you don’t do well on more than two dimensions, I’ll spare myself the special pain of going there at the moment.)

        So, please, while the tracelessness and insignificance of the CO2 between 17km and 140 km might obvious, and it fits the definition of rarefied in that context, your arbitrary cherrypicked high-altitude strawman plays no role in my model, taking perhaps less than a quarter of an inch from the length of the pencil.

        I do thank you, however, for the well-written off-point reply; I suggest you are now ready for step 6 (http://judithcurry.com/2011/02/04/slaying-a-greenhouse-dragon-part-iii-discussion/#comment-38391).

      • Bart R
        Self publishing a book would leave me with 4999 copies lying around that would just clutter an already cluttered library.
        You are right of course, area and volume are not the same. The ideal gas equation has volume as a component so I do not understand to what you are referring. Please clarify. Thank you.

      • RiHo08

        Ideal gas equation well and good, but what the heck does it have to do with optical properties?

        I take a ton of dust in a tube with a 1″ cross section and peer through it, I’m going to have significant oclusion whether that tube is a 50′ long or 50 miles, so long as the dust is scattered across the cross section. (Sure, there’s some differences in total transmissivity, which is where depth of field comes in, but it’s not a gas concentration thing.)

      • Sorry, forgot my audience.

        My example of dust above does not imply I am asserting that each square inch of sky contains a ton of dust.

        Replace ‘dust’ in the above example with ‘blueberries’, to get a sense that I am also not saying there is a ton of blueberry in every inch of sky.

        People are perhaps not well-used to dealing with the IR optical properties of gases, since in the visible range most gases* appear transparent from their casual observations most of the time, and IR is even less easy to observe.

        *The atmosphere above Los Angeles, being obviously brown-to-orange, a notable exception.

        Is it easier to see through 17 cm of light fog, or 17 km?

        As fog gets thicker, does it get harder to see through, and darker?

        We’ve had a 44% thickening in the IR fog effect of CO2, from the ground to the top of the atmosphere.

        My comparison to a pencil and a square inch stands.

      • CO2 is a gas, so the ideal gas equation is a reasonable place to start. The concentration of that gas does matter, no matter whether it has photon absorbing properties or not. CO2 is a trace gas and in relation to the other gases of the atmosphere, it can not act greater than its concentration. In my thought experiment, I calculated how much would CO2 heat other nearby gases molecules if we assumed that it had the ability to achieve a temperature 5 C greater than surroundings. How much would CO2 heat the surroundings? not much, only 0.00039 C more. By your implication of CO2 having IR capabilities up to TOA through a more opaque atmosphere to long wave IR, belies its very low concentration. CO2 can’t heat molecules anymore than it is in contact. Another analogy: If I we were in the La. Superdome, you were in section 640, the top row nosebleed section and I on the fifty yard line and all was dark and still, if I lit a kitchen match, you could see it. If you had dragged a spectograph up with you and pointed it at the match, you could likely measure such lines of light that you would surmise I had lit a kitchen match. However, you would not be able to see your way down the pitched steps to the exit in the light of the match on the 50 yard line. The issue is relevance. That one can use instruments to see and measure CO2 and IR does not mean that it is relevant to the surface heating from CO2. The partial pressure of CO2, compared to the other atmospheric gases, and in particular to water vapor which is 2 orders of magnitude greater in concentration, has a broader spectra in IR wave absorption, and has a considerably greater heat capacity, means that CO2 is a bit player, a trace, a footnote.

      • And.. can you walk me through how your calculations work for the temperature of the moon?

        What exact calculations do you do, by the way?

        I used TOA as a convention because describing the distribution of the atmosphere, not really my bag.

        Heck, describing the heating of the atmosphere isn’t part of my argument, either, so how does your straw man apply to what I said?

        We’ve come full circle, with no progress.

        I’ve said no more, nor less, in my claim than that taken as a cross section, collapsing the vertical dimension, one accumulates almost half a gram of additional man-made CO2 per square inch of sky in the past 15 years.

        That half a gram is has high IR absorbtivity in some narrow bands.

        There’s no case for calling that amount of occlusion ‘trace’ in optical terms.

        If you want to call the amount trace for your unrelated equations regarding heating, you’ll have to produce your equations in a more fulsome manner, and answer the objections of others whose equations throw yours into doubt, won’t you?

        And you still won’t dislodge that for at the very least the issue of occlusion of IR, CO2 cannot be called trace in the atmosphere.

        See, I don’t need global radical changes to my air without my consent to be temperature-affecting to object to them, any more than a villager need to prove that the town drunk peeing in the village well is unhealthy to be disgusted by it.

    • Bart,

      “Ask an economist about the importance of marginal additional returns over many compounding periods.”

      You don’t have any compound period effects of CO2, just simple addition effect. CO2 can only absorb a trace amount of IR energy at 15um and other minor bands of wavelength not the full spectrum of IR radiation. Then, like all other gases, quickly (within a second) re-radiate back to atmosphere to the space. Compared with atmospheric IR energy contents, CO2 energy is trace or simple addition of 10ppm of CO2 per decade to have a temperature effect is trivial. Remember W is J/s.

      In a power plant situation, CO2 content in flue gas is about 15~18%, CO is usually less than 100ppm, and Hg depends on coals, usually in less than 10 ppb or zero if enough activated carbon or unburnt carbon (soot?) is present.

  25. Per Fred’s comment above, I (KLC) talked to the people who I have easy access to…my wife (Wifey), sister-in-law (SIL) and father-in-law (FIL, age 93).

    KLC: Do you believe human activities can increase the earth’s surface temperature?
    SIL: Yes.
    KLC: Is that a good or bad thing?
    SIL: Bad.
    KLC: Do more people die in the winter or the summer?
    SIL: Summer.
    KLC: Do you believe people should drive smaller cars to reduce emissions?
    SIL: No. I’d be killed if I had an accident in my little car. It scares me. I want a bigger, safer car.
    KLC: If I said most CO2 emissions in the U.S. came from coal-fired power plants, what would you say we should do about that?
    SIL: I don’t know.
    KLC: Do we have the right to tell people in India and China to reduce their rate of economic development to reduce emissions?
    SIL: Yes, we can say whatever we want.
    KLC: Should we make them do what we say?
    SIL: We can give them facts and try to influence them.
    KLC: Would you drop nuclear bombs on them if they refused to reduce emissions?
    SIL: No.
    KLC: Have I ever tried to influence you opinion on these topics?
    SIL: No.
    KLC: If I did try to influence your opinion, would it work?
    SIL: No.

    KLC: Do you believe human activities can increase the earth’s surface temperature?
    FIL: Yes.
    KLC: If humans were increasing the earth’s surface temperature, would that be good or bad?
    FIL: Bad.
    KLC: Do you think people should drive smaller cars to reduce emissions?
    FIL: I think people should drive what they want. It might better if people drove smaller cars, but it will never happen because rich people like big cars.
    KLC: Rich people like me? I like a big car.
    FIL: Yes. People like you. The big problem is there’s so many people. How will we reduce emissions if the population keeps growing so quickly?
    KLC: What should we should do about population growth?
    FIL: I don’t think there is anything we can do.
    KLC: If I told you that most emissions in the U.S. came from coal-fired power plants, what would you suggest we do about it?
    FIL: I don’t understand the question.
    KLC: Do you think we should tell people in India and China to reduce their rate of economic development to reduce emissions?
    FIL: Sure, we can tell them and give them advice, but I don’t know what good it would do if you can’t enforce it.
    KLC: Do we have the right to tell them how to live?
    FIL: No.
    KLC: Have I ever tried to influence your opinion on these matters?
    FIL: No.

    KLC: Do you think humans have the ability to change the earth’s surface temperature?
    Wifey: Not now.
    KLC: What do you mean?
    Wifey: Look at the advances of nuclear power and stuff. Someday we might control enough power to change the earth’s environment.
    KLC: Okay. Do you think we should drive smaller cars to reduce emissions?
    Wifey: No.
    KLC: If I told you most of the U.S. emissions came from coal-fired plants, what do you think we should do about that?
    Wifey: What kind of emissions are we talking about?
    KLC: CO2.
    Wifey: I don’t have a problem with CO2, but I don’t like the other stuff. The pollution.
    KLC: So, you’d like to reduce particulates in the air.
    Wifey: Yes.
    KLC: Do you think we have the right to tell people in India and China to slow their growth to reduce CO2 emissions.
    Wifey: No.
    KLC: Have I ever tried to influence your opinion on these matters?
    Wifey: I know very well where you stand, but you’ve never tried to change my opinion.
    KLC: Would it work if I tried to change your opinion?
    Wifey: I don’t see how.
    KLC: I think you’re well grounded. If I thought you were real wrong, I might try to give you some facts and see if that changed your mind.
    Wifey: That would be okay.
    KLC: If some guy on the Internet, with great academic credentials, sent you an e-mail and tried to get you excited about human activities and global warming, would that influence your thoughts?
    Wifey: What? Are you kidding? I can figure things out on my own.
    KLC: One fellow in particular thinks I should talk to you about the time I spend arguing on the Internet…like it might be better if I did something else.
    Wifey: I think what you’re doing must be productive from your point of view or you wouldn’t do it. Exploring and putting your opinion out there is fine with me.
    KLC: If I make a lot of money on the Slaying the Sky Dragon book and send you back to Europe for another art trip, would that be okay?
    Wifey: Yes, that would be great. I loved my two weeks in Cadaqués and La Romita. How about France this time? Or Sao Paulo. I’d love to go to Sao Paulo.
    KLC: Whatever you want, babe.
    Wifey: We’re still going to Venice this summer, right? And England in the fall? I have my trip to Sante Fe coming up and British Columbia in October, but I could slide another trip in, maybe. I can’t really go anywhere in April because of the art show. We still have a big trip in 2012 for our 40th anniversary, right? Can you get time off from work? Where are we going?
    KLC: Sorry, I have to get back to arguing with people on the Internet now.

    • Ken – If you care seriously, take the time to share all the discussions here with your wife (and with others), and make clear that important decisions are involved – an hour or more may be needed, plus a few days of reflection after that.

      Most important – do it in private and don’t share it with us. That way, there will be nothing to prove, which will be critical for ensuring that all involved realize that what is at stake is your future and not our opinion.

      • I care a lot, Fred…about junk science like passive radiators increasing Tpeak or Tavg. Government money should not be spent on this nonsense. I really think you guys should get humble fast.

      • Junk Science:

        For the period from 1970 to 2000, after human emission of CO2 for 60 years, the effects of nature and human emissions of CO2 swapped places to produce the same global warming rate of about 0.15 deg C per decade as for the period from 1910 to 1940.

      • In the same period 1970 to 2000, more than half the worlds population has industrialised.

        Yet there has been no recorded break out move in the trend for average global CO2 content for that period, let alone in the temperature which is well within natural variability. CO2 still increases at a rate of 2ppm annually as it has done since “official records” began, (apparently).

        Any warming has already been more than accounted for by natural cycles.

        AGW is pure “Junk Science”. The motive is hundreds of trillions of dollars.

        Yet all the money in the world cannot buy experimental evidence that elevated levels of CO2 can warm air.

        Historical CO2 levels V’s Temps.

        “AGW Debunked for £3.50”

      • Are you still banging on, Will? I think you may be talking to yourself as everyone else has got bored with your distortions of the science and moved on to discuss other matters!

      • I think anyone can recognise the distinction between distortions of reality and “uncertainties” (contradictory evidence against the GHE hypothesis).

        Ignoring reality won’t change it.

        Still keep on spinning, its all you’ve got, you may as well use it.

      • When the only tool you have is a hammer (CO2), everything looks like a nail (AGW.

  26. Pekka, Fred,
    Thanks, I understand that context is important, and I may well not have it right. But the result for me was a feeling that an unwarranted sense of accuracy was being conveyed and it appeared that the fact that we do understand radiation physics very well was being used, inadvertently or not, to do this. I think that this was a case where we get later a response of ‘What he really meant was —‘. But enough of that . In the present context, I think that putting what is really well established front and center in the discussion puts it , well, front and center in the discussion. Then, since the subject is rather complex we get all kinds of discussion about violating thermodynamic laws and definitions of heat etc. etc.
    I see that Science of Doom is really wading into this issue. He may have a pretty good textbook when he is done.

    • I see that Science of Doom is really wading into this issue. He may have a pretty good textbook when he is done.

      I agree. SOD manages to remain accurate while describing the science in an understandable manner.

  27. Fred, I briefly saw you wade into RC. In RP’s response to you, you could cut air with his arrogant response.

  28. I tend to lurk rather than talk at these debates, Here is what I have learned from lurking.
    Carbon dioxide is a greenhouse gas. This fact is not in dispute. At certain infrared wavelengths it absorbs energy, reflecting it back to the surface of the earth and warming the planet. Some of the warming seen over the past century is almost certainly due to this.
    The relationship between CO2 and global warming is not linear. Once all the energy at the wavelengths absorbed by CO2 has been absorbed, the greenhouse warming due to CO2 is finished and further increases in CO2 will have no further effect on global temperature.
    Doubling the CO2 content of the atmosphere will occur within the next 50 to 100 years. On its own, this will increase global temperature by about 1.3 degrees centigrade. This fact is agreed upon by both the IPCC and global warming sceptics.
    If CO2 doubling is going to increase global warming by more than 1.3 degrees C there needs to be positive feedback from somewhere,
    Easy, says the IPCC. The temperature goes up and you get more evaporation of water. Water vapour is a powerful greenhouse gas. It provides positive feedback. Instead of getting one degree of warming you get two or five or six degrees. The polar ice caps melt and we all drown from the rise in sea levels.
    But more water vapour means more clouds,
    If you have ever flown over clouds in a plane you will have seen them as brilliant white. Low clouds reflect sunlight and cool the earth.
    So there may not be positive feedback. We may get only one or possibly two degrees of global warming… The effect of this would be mainly beneficial…
    Life on land depends on photosynthesis.
    Carbon dioxide and water combine to produce oxygen plus carbohydrate.
    Carbohydrate leads to plant growth and provides food, directly or indirectly, for every land based animal on this planet.
    Carnivores eat herbivores. If there wasn’t any photosynthesis the herbivores would die out and the carnivores would have nothing to eat either.
    So carbon dioxide is a life giving substance.
    But it is a trace gas in our atmosphere. About 370 parts per million. There are 2500 molecules of other gases in our atmosphere for every molecule of carbon dioxide. Plants have to struggle to get CO2. The stomata on the back of their leaves have to open wide to take in enough of this life giving gas. This means they lose water.
    If the carbon dioxide content of the atmosphere were to double, plants would grow faster. They would have more CO2 for photosynthesis. They could narrow their stomata and lose less water vapour.
    I am not saying that global warming will not happen. What I am saying is that we really don’t know what the climate will be like 100 years from now. It is important not to send ourselves bankrupt with huge carbon taxes because if we go bankrupt we won’t be able to fix anything.
    If you read “Climate: the Counter Consensus”1 you will see that the IPCC are not scientists but evangelists. They do not mould their hypotheses to fit their data. They manipulate their data to fit their hypothesis.

    • Jim Petrie

      I’m being reproved of late by some for accuracy (by which it appears they mean precision) of language, so offer the interpretation of the word ‘reflecting’ near the middle of your second paragraph to ‘by some net means transmitting’ to spare you that grief.

      Also, you have a gloss at “more water vapour means more clouds,” which is true in some context and in net, but not with perfect linearity.

      There is a serious and complex question whether and in what pattern the increase in water vapour meaningfully might alter the albedo of clouds in proportion to the increase in equilibrium temperature. This somewhat fatally weakens your case without better support than have I yet read.

      Even if your case is not completely undermined and collapsed at this juncture, “The effect of this would be mainly beneficial,” is a luxuriantly padded gloss of an argument (even with the subsequent simplistic support you offer) that overlooks the transition costs, and is largely fluff and prevarication.

      You use the term ‘trace’ gas to describe CO2, which in some contexts is quite appropriate.

      In the context of bioresponse, it is somewhat less that fully truthful to call CO2 trace. Different plants (especially, and especially favorably for weeds) and animals (to a lesser degree especially less with increasing size) have different and sometimes marked responses to lifelong exposure to CO2 at these levels, which in the chaotic world of life is truly unpredictable.

      Claiming evangelically benefit for life from increased CO2 in this context is Nostradamus-like prophecy, and one can only conclude that divine agents are whispering this knowledge directly into your ear because mortal and rational agency cannot produce it to any confidence-level.

      In the context of optical properties in the IR spectrum, the amount of CO2 added by human industry is comparable to taking a fresh pencil and filling in a one inch square on a sheet of onionskin paper over and over again page after page until the pencil is used up entirely, and then lining up the squares in a sheaf and holding that stack of onionskin up to the light of the sun.

      As onionskin is called by some ‘tracing paper’, one must conclude this is what you mean by your use of the word ‘trace’.

      Here is what is being discussed, if we could see IR: Every inch of the sky darkened by the graphite of one pencil in human hands.

      • Jim Petrie writes “On its own, this will increase global temperature by about 1.3 degrees centigrade. This fact is agreed upon by both the IPCC and global warming sceptics.”

        Sorry, this is not a fact, and it is not agreed by many global warming sceptics. The number of 1.3 C is a hypothetical and completely meaningless number which has never been measured, and probably can never be measured.

      • Please someone take the pencil away from Bart R before he takes someone’s eye out with it.

        Bart R, to be blunt, you’ve made your point.

      • Peter317

        Pencil down.

        Thank you, sir.

  29. As a lurker here (and hardcore denier), I respond to Dr.Curry request, and finally took a look at blogs of Prof. Claes Johnson. IMEO, he has fundamental difficulties with physics and classic mechanics. At first I thought he was just confused with terminology (possibly a language translation issue) and used the term “thermodynamics” where a normal physicist would use “convective heat-mass transfer”. He uses again the term “thermodynamics” when presenting Navier-Stokes Equations (in some very basic form), which are in fact approximations for “non-equilibrium thermodynamics”, which is sort of “anything but thermodynamics”. Then I realized that Prof. Johnson completely dismisses statistical physics, which explains why he confuses systems in thermodynamic equilibrium with systems that are fundamentally far from equilibrium (and therefore cannot be treated by classic thermodynamics).

    When presenting his account for fluid dynamics, Prof Claes Johnson again confuses general viscous NSE with non-viscous equations of ideal fluid which people call “Euler Equations”. He apparently does not realize that assuming fluid viscosity to exact zero results in a change in the rank of differential equations, such that spaces of solutions cannot possibly match anymore in topological sense.

    Regarding climate issue and greenhouse effect,
    http://www.slayingtheskydragon.com/images/stories/PDFs/climatethermoslayer.pdf

    Here is my impression. While Section1 contains few reasonable statements and Section2 describes CORRECT physical idea of greenhouse effect (“effective emission height” coupled with constant lapse rate), generalizations in Section3 seems totally irrelevant to anything.

    Prof Johnson begins his attack on AGW in Section4 with a clearly wrong initial premise:

    “Climate alarmism as advocated by IPCC is based on the assumption that radiation alone sets an initial lapse rate of 10C/km, which then in reality is moderated by thermodynamics to an observed 6.5C/km.”

    This is a clear straw argument. Prof. Johnson substitutes correct description of Sec.2 with bogus claim of lapse rate being defined by radiation. Every child of climatology, every climatard and even deniers like myself know that the above statement is blatantly false, that the 10C/km (9.8 actually) is a result of purely mechanical process called “convective stirring in the presence of field of gravity” in dry air, and it has absolutely nothing to do with radiation; that the reduction to 6.5C/km is a result of moisture cycle, and again has nothing to do with any radiation. The rest of the article re-introduces the atmospheric lapse rate as mechanical phenomenon and is therefore redundant and trivial once we realize this strawman in the initial premise.

    In Section2 Prof. Johnson criticizes the standard AGW theory:

    “In this argument the outgoing radiation is connected to a lapse rate (decrease of temperature with increasing altitude) supposedly being determined by thermodynamics.”

    Yet the remaining sections of the article are devoted to proof that the lapse rate is indeed determined by “thermodynamic”, which clearly contradicts to the initial statement of Sec.2

    Also, everyone knows that adding another trace of CO2 does not change mechanical properties of air to any sensible degree, and therefore the lapse rate is not directly affected at all. The last three paragraphs of Section4 are therefore total nonsense.

    In short, the chapter “Climate Thermodynamics” is an in coherent compilation of self-contradictory statements and straw arguments, and does not bring anything new to the table.

    P.S. Quoting Vladimir I. Arnold in the context of this essay is a grave insult to him.

  30. Claes Johnson

    Yes I have a difficulty with statistical thermodynamics like everybody else including Arnold. If you are interested in how I dealt with this difficulty, read my books on turbulent flow and thermodynamics. How did you handle it?

  31. Reply to Bart R
    Carbon dioxide does not absorb at all wavelengths.
    Ian Plimer argues exactly as you do – once CO2 has done its job of absorbing at the wavelengths it can absorb at, its job is finished. No additional CO2 can do any more to the planet energy balance.
    I thought that was a key plank in the ‘sceptics cannon.
    No?

    • Jim Petrie

      No?

      Really no.

      Very much no.

      I won’t speak for anyone’s planks or canon, and will sidestep pirate quips until official Speak-Like-A-Pirate-Day, but I will expand somewhat.

      CO2 wouldn’t need to absorb all wavelengths, nor is that anything remotely like what I said.

      CO2’s principal claimed non-saturating non-overlapping bands by absorbtion spectroscopy are principally around 2.5, 4 and 20 micrometers. There is some overlap otherwise with water vapour or other gases and CO2’s absorbtivity.

      And again no, CO2 doesn’t finish its job, though the job it does is significant.

      You, or perhaps Ian Plimer (who I doubt argues anything like I do overall), might suggest that it only takes a little CO2 to quickly grab up all the energy in those narrow bands and exhaust its potential to heat the air, and so any additional CO2 won’t matter.

      This intuitive view simply isn’t how the atmosphere must work with regard to IR, as we well know from familiar and plentiful observation of that other greenhouse gas, water vapour.

      More CO2 means longer resident energy, means higher equilibrium temperature, with a very slow rate of diminished marginal increments.

      What you describe is, however, how cloud albedo works.

      Only takes a little cloud to reflect light back, because it’s _reflecting_ visible light that is transmitted without absorbtion, not _backradiating_ resident IR that is transmitted only after absorbtion, so a thin layer of cloud does not become a better reflector by addition of thickness while every addition of CO2 becomes a better absorber.

      Since surface area increases as a square while volume increases as a cube, cloud albedo is likely quite insensitive to additional water vapour and never catches up with the greenhouse effect, falling farther behind the warmer it gets.

      At this moment, I’d like to irrelevantly mention also that the ballyhoo about the surface of the Earth always being the warmer body and the air always being the colder is frankly wrong much of the time.

      Air moves. Water moves. Warm air often flows over colder ground, and colder currents of water often flow beneath warmer air masses. Look to the Chinook winds or the Mistral, the many ocean gyres, monsoons, and thermals moving through mountains.

      Where these happen, the warmth from the warmer sky is plainly spread to the colder surface below.

      Not really a GHG effect, but wanted to get that off my chest.

  32. How did I handle statistical physics? Just as Maxwell, Boltzmann, Plank, Gibbs, Chandrasekhar and Landau and Lifshitz did. Arnold was joking, and mostly objecting to the method of teaching thermodynamics in formal axiomatic way, the same way as he hated the senseless Bourbaki axiomatism in mathematics. Please also note he was referring to “elementary course in thermodynamics”, not “statistical physics”, which has a rigorous mathematical foundation.

    I am not familiar with “statistical thermodynamics”. I remember some things from statistical physics/mechanics and kinetic theory, from which the classic thermodynamics can be derived under certain reasonable assumptions and expansions. I don’t think I am interested in reading your books, especially on turbulence – it was sufficient to visit your blog and realize the depth of your expertise in the topic.

  33. Of course you don’t have to read my books, but ignorance is not a virtue in science although it is frequently used as argument. Like Einstein and Schrödinger I don’t consider statistics as physics in the sense of describing the evolution of real physical systems, only fictitious non-physical ensembles. But you are right, Einstein and Schrodinger were kicked out from the “grand canonical ensemble” of “consensus” of taking global mean-values. But this does not mean that they were wrong. Or does it?

    • It is incorrect if every point on this planet is completely and independently different from different angles of energy input from the sun, to the different magnetics due to being a rotating round planet. Atmospheric pressure smooths these events over distance.

    • Unlike certain rare types, I don’t consider myself in any proximity to the league of Einstein and Schrödinger. However, I do know that scientific knowledge evolves and diffuses across disciplinary boundaries, and I am willing to assume that fathers of science were not completely familiar with certain logical and mathematical possibilities developed in early seventies of last century on the basis of Poincare ideas. It turned out that “He” does not play dice with the Universe, but plays billiards with negative curvature all the time, sometimes shuffling all ~10^19 little balls in each cm3 at a rate of about 10^9 hits per second. If you look up something like “Sinai billiard” or “Bunimovich billiard”, you might get the idea, unless you also dismiss the entire atomic theory of matter as you dismissed the QM and the virtual method of canonical ensembles.

  34. Reply to Bart R
    Sorry Bart. I may not be a climate scientist but I am not a complete idiot.
    You rabbit on about graphite. This is elemental carbon. It is not a component of the atmosphere.
    You also talk about carbon monoxide. This is poisonous, but like oxygen, nitrogen and argon it has two atoms to the molecule and is not a greenhouse gas.
    CO2 has three atoms to the molecule, as has water vapour and is a greenhouse gas.
    So where are you oh so superciliously coming from?
    Jim

    • Jim Petrie

      Nice attention span there.

      Might one suggest, READ HARDER?

      Bolding text is not going to make its meaning any clearer.

      You see a word or phrase and your knees jerk and you’re off in a tizzy before you get to the end of the paragraph.

      You ignore conditionals, context, relational prepositions, and subordinate clauses.

      Not a climate scientist?

      Sir, you’re not a master of your own thoughts, if you cannot train them to apprehend what is actually set out and must instead let them fabricate straw men for you to attack.

      Elemental carbon, aka soot — WHICH I WAS NOT TALKING ABOUT — is very much a component of the atmosphere in some contexts, but that’s irrelevant to my remarks.

      Typing in all caps is not going to transmit meaning any better.

      When did I mentions carbon monoxide as a GHG, or as a poison? I mentioned carbon monoxide in context of the definition of trace gas. See, you’re missing out the whole category of the posting you’re all ramped up about. That’s a kind of basic reading skills error.

      Maybe I’m that unclear.

      Possibly, others too have a hard time figuring out that sentences are about ideas and paragraphs are about like ideas and written compositions of paragraphs generally link ideas in some sort of orderly progression.

      So, sure, for people who haven’t learned these precepts, I guess everything hops about for them when they read.

      Form a support group with them, and get together to practice critical reading.

      Which is to say, could you expand on your definition of the word complete?

      • Bart, consider your statement above,

        “When did I mentions (sic) carbon monoxide as a GHG, or as a poison? I mentioned carbon monoxide in context of the definition of trace gas. See, you’re missing out the whole category of the posting you’re all ramped up about. That’s a kind of basic reading skills error.”

        Then Bart, would I be wrong in stating that you yourself mentioned it in the following passage from your earlier post of Feb 5, 6:04pm?

        “What would we say were the atmosphere to suddenly contain 0.04% carbon monoxide? Suppose sulfur, mercury or radium permanently and increasingly permeated to the extent of carbon in the air? Maybe not lethal immediately, but certainly enough to grab the attention.”

        The context of this Feb 5 post certainly indicates that you are thinking of CO as a poison; after all, you related it to mercury and radium, and use the word “lethal”.
        Also, the context of your first sentence in the first quoted statement appears to be an accusation that Jim had said you had referred to CO as a poison . Here’s what Jim actually said,

        “You also talk about carbon monoxide. This is poisonous, but like oxygen, nitrogen and argon it has two atoms to the molecule and is not a greenhouse gas.CO2 has three atoms to the molecule, as has water vapour and is a greenhouse gas. So where are you oh so superciliously coming from?”

        You surely must agree that in the above Jim does not say you said CO is poisonous; the order of the sentences means he is saying it.
        Ron

      • Ron Durda

        Thank you for your patient reply.

        I mentioned carbon monoxide, mercury and radium as examples of things that fit into the category of ‘trace’ for some purposes, and not for others, depending on context.

        It may have been a mistake to list only things that alarm people as dangerous poisonous substances at some doses (the dose makes the poison), but they are the most ready examples to hand.

        I could have listed instead estrogen, testosterone and gold, I suppose, but they are more far-fetched and less likely to be meaningful in the context of gases in the air.

        I could have mentioned fairy dust or angel’s tears, which one supposes were they detectable and proven by objective measure at 400 ppm in the atmosphere would be attention-grabbing.

        The point of the post you refer to, as set out in its beginning, repeated at its end, and developed in its body was the topic of the definition of the diminutive terms, most especially ‘trace’ in the description of CO2 in the atmosphere.

        These terms, as they are all ‘relational’ are relative to something.

        Is the ‘trace’ prospect of any detected, proven, scientifically-accepted angel’s tear insignificant, or kinda important as it is so unlikely and so profoundly excites the core beliefs of so many?

        Any true and real fairy dust with magical power to make dogs and children fly, in so small a quantity so long as it is still proven, meaningless, for similar reasons?

        Were estrogen to fly about the air and cause some biological changes in those breathing it, or testosterone, however trace and limited these changes, be of no concern whatsoever to anyone?

        Could someone mine the air for gold, were it even at the prospect of refining a ton of atmosphere to optain a mere gram of the metal, would it be utterly uninteresting?

        Are you no going to go around saying that I’ve asserted that the air contains angel’s tears and gold, fairy dust and testosterone?

        Or are you going to accept that my point, my context, the progressive rational development of my theme in my posting is about how abused and misdirected the terms used to belittle the concentration of CO2 appear?

        Because you can build these cherry-picked straw men until the angels weep, and sell all the fool’s gold you wish, it will not give any more weight to your fairy dust claims.

        If you really don’t get what I’ve said here, I believe you too would benefit from a support group for remedial reading.

  35. Judith,

    Multi-dimensional problem solving is currently beyond the concept of current science. So generalized theories are currently the norm.
    This is where science cannot possibly understand the complexity of a round planet, rotating, with different longitudes of circumference with different height variables, in an enclosed atmosphere.
    The slowing speed of the planet adds another dimension to a planet in constant change along with the slowly moving away from the sun plus the speed of the solar system moving in space.
    Add to this gases, chemicals, minerals and water in it’s many forms depending on heat or circumstances for evaporation to occur.

    You have to understand how this planet changed in the past from speed of rotation to chemical changes to understand how it is possible to have life today. Centrifugal force has been put into a pseudo-science category when plant and animal life need it to survive and grow.
    Salt changes on the ocean surface is a centrifugal force event that has always been present along with differing pressures.

    Rather than bickering and falling behind in our understanding of this planet for self gratification, we should work together for a much more better understanding in our knowledge of the planet and how it operates.

    • Judith,
      It is amazing how much you can learn by following the path of a simple molecule and adding in the multi-complex barriers that it has to face.
      Defection and absorption into material along with soring energy are a few more additions.

  36. Point an IR camera at the clouds and you will record the total bandwidth of available IR. CO2 can only ‘see’ three very limited bands. The camera cannot tell you which, or how much, IR radiation is due to CO2.
    Nasif Nahle pointed out the emissivity of CO2 is 0.001
    A very poor ‘blackbody’ indeed.

  37. As a ‘lurk-ish’ occasional commenter, here’s where I am.

    I came to Climate etc because I knew nothing about climate science and yet I felt suspicious about the political demands for urgent action and wanted to base my view on something a bit more substantive. In the last 4 months I feel I have learned a great deal. The various greenhouse threads have completely convinced me that the GHE exists but I don’t yet agree that it can be quantified in terms of an accurate climate sensitivity. I am also still concerned about our understanding of other factors such as natural variability, the sun, clouds and other feedbacks and forcings and in particular how they intereact and relate to each other in the complex and chaotic climate system. I guess that makes me a luke-warmer. So basic physics of the GHE? No problem. Bring on the rest of it, please.

  38. Judith, I’ve read these threads with a kind of appalled, excruciating, embarrassed amusement. At first, jumping here from The Blackboard, I thought this must be part of some extraordinary practical joke – then, maybe, they’ve just been badly garbled by a google translation – but then a machine algorithm could never randomly invent this weird world of illogical rambling – and no one, not even Google, could write so appallingly! – and, finally, maybe an asylum is missing quite a few inmates! Your generosity seems to know no bounds! But, please, no more, the agony it’s to much!

  39. John Q. Lurker

    Prof. Curry:
    I’m a lurker with a sketchy science and math background. Back in early December, I read some of the book or materials that Claes Johnson had/has online (not “Slaying the Sky Dragon”). I don’t recall everything I thought at the time, but I do recall receiving the impression that here was a sophisticated version of the argument that the 2nd Law precludes the greenhouse effect. I had always thought that argument was wrong, so if I had been forced to guess, I’d have guessed that Prof. Johnson’s argument was wrong.

    Your comments threads here have been helpful to me. If it makes any sense to say that now I’m surer of what I would have guessed in December, I am, but I don’t think it does make much sense.

    However, for many years, I had been interested in the problem of rebutting the 2nd Law argument, and it irked me that nothing I had ever read did that well enough to make any of its proponents with as little or less background than I have, really think they might be wrong. Then Roy Spencer came along with his “Yes, Virginia” essay. I thought that was marvelous, but still I wasn’t fully satisfied, and here’s why:

    The 2nd Law argument depends on a confusion between heating as from a heater, and heating by a progressive burdening of cooling. That’s what needs explaining, but it seems to be incredibly difficult to find a good “English only” (very little physics or math) way of explaining it, because they can always say, “Well, you are talking about heating, so…” Dr. Spencer’s essay is very good, but I think that confusion needs to be directly addressed.

    • John Q. Lurker,
      The second law has a long history related to the observation that a perpetum mobile cannot be built. The theory of thermodynamics developed to be a mathematically elegant theory based on axioms, describing correctly a wide range of physical phenomena, but with an abstract formulation, which has always been difficult to understand intuitively even among people, who apply its formulas regularly in their professional work.

      In the classical thermodynamics it does not make sense to discuss two simultaneous flows of heat between two bodies, as all formulas of the theory know only the net flux and even the definition of heat is based on that. This applies equally to heat conduction and to the radiative heat transfer.

      With the present approach to physics, most physicists who consider radiative heat transfer have a different intuitive way of looking at it. With radiation one can place measuring devices between the two bodies and measure separately radiation emitted by one body towards the other, or one can measure separately infrared radiation emitted by the earth surface and infrared radiation coming towards the surface from above (it is emitted by the greenhouse gases of the atmosphere and by the clouds, but this is not as easy to verify empirically).

      People who wish to shed suspicion on climate science have picked the old axiomatic thermodynamic formulation, which knows only about the net flux of heat, and claim that the sentences picked from that theory would apply to all correct descriptions. This is pure nonsense. When physics is discussed in more details by looking separately at the two parts of a whole, the old sentences do not apply to these new concepts unknown to the original formulation.

      It is conceptually impossible to present a simple proof that the second law allows for the greenhouse effect, as the second law is about what is not possible and not about listing what is possible. As I tried to explain, the greenhouse effect is not among those things forbidden by the second law. Every specific claim attesting that the greenhouse effect would be against the second law can be proven erroneous, but that may get messy, if the other side is clever enough in obscuring the facts and may also intervene the discussion repeatedly by new erroneous claims.

      • John Q. Lurker

        Pekka Pirilä:

        One other thing. It is absolutely false that a given concentration of CO2 heats the ground, once equilibrium is reached. What heats it is an increase of the concentration. I’m sure you’ll agree with this, but my point is that it is just plain wrong to say that atmospheric CO2 “warms the earth,” and the proponents stumble on this.

      • What does the word “heats” mean.

        An oven heats a house even, when the temperature is kept the same, if the temperature would fall without the oven.

    • John,
      It is actually easy to defend the greenhouse effect against the false claim that it violates the 2nd law.
      Because the argument is usually presented incomplete and the second law stated for systems that are isolated.
      But the earth is an open system. In an open system the 2nd law states only that the production of entropy within the system is greater or equal to zero. In steady state the net flux of entropy across the system boundary balances the entropy production within the system.
      Such a non-equilibrium system can maintain a state with fairly low entropy if the net entropy flow across the system boundary is an export of entropy. The sun provides energy with a low-entropy value to the earth and the earth emits energy with a high entropy value. Within the earth therefore a lot of different states and processes are in principle possible. Per example Life.
      Because all the incoming energy into the earth system comes from the sun and this is energy with a low entropy value, the greenhouse effect no matter how it is described with or without “backradiation” does not violate the 2nd law.
      So all the arguments I know that try to refute the greenhouse effect ignore the energy input with low entropy. If one considers all radiant energy fluxes including the sun everything is ok.
      Regards
      Günter

  40. John Q. Lurker

    Pekka Pirilä:
    I disagree. The 2nd Law argument imagines that atmospheric CO2 is supposed to be a “heater” and says that, being cooler than the ground, it cannot heat the ground, so it is not a heater. In fact, the CO2 is not supposed to be a heater. The sun is the only heater. The CO2 is only a heater in the sense that it partially defeats a cooling mechanism, but this is a counter-intuitive usage of “heater.”

    The 2nd Law argument proponents see that the CO2 is often said to heat the ground, and this together with the talk of back-radiation makes them think that the CO2 really is supposed to be a heater.

    Having said that, I’m thinking I may have been wrong to say that it was “incredibly difficult” to explain the matter “in English.” It’s just that I’ve been baffled by the fact that the 2nd Law argument proponents won’t see it.

    • John,
      Now you are doing precisely that error that I was trying to explain.

      The second law does not say absolutely anything about partial process, it makes an statement about the whole.

      A heats B can be interpreted to say that there is a energy flux from A to B which ends as heat in B. The second law does not forbid this for any pair of temperatures, but it tells that if this is the case and if B has a higher temperature, then there must also be another process that takes heat from B and transfers it to A.

      In another words for the case of heating of the surface: There are radiative energy fluxes both from the surface to the atmosphere and from the atmosphere to the surface. The first is larger and there is a net heat transfer from surface to atmosphere. Compared to the case of no greenhouse effect the radiation from the surface does not change, if the temperature is the same, but the other energy flux is not present. Thus we have a much stronger net heat flow out from the surface. The greenhouse effect warms the surface compared with the case of no greenhouse effect. There is nothing wrong in using the word “warms” in the sense of this comparison.

      The second law has no implication even remote close to forbidding the existence of greenhouse effect.

  41. John Q. Lurker

    Pekka Pirilä:

    The greenhouse effect warms the surface compared with the case of no greenhouse effect. There is nothing wrong in using the word “warms” in the sense of this comparison.

    No. There is no such thing as warming “compared with the case of.” Warming is a process. You can say that the earth warms as the concentration of GHG’s rises, but this is not “compared with the case of.”

    The second law has no implication even remote close to forbidding the existence of greenhouse effect.

    I agree! How do you see me as not agreeing?

  42. John Q. Lurker

    Pekka Pirilä:

    I meant to say, there is no such thing as warming “compared with the case of,” unless we are talking about some other case of warming, i.e., two different warming processes.

  43. John Q. Lurker

    I give up. This is real life, not sophistry.

    I don’t know how you can say that when I agree with you that the 2nd Law argument is wrong. What I’m talking about is the way that the proponents of that argument see things, not the way things really are.

    • I apologize if I erred.

      The discussion just started to appear too similar with some continued by people who obviously understand all the arguments well, but pretend otherwise to confuse other readers.

  44. The other case was there. It was the case of atmosphere without greenhouse gases, but assuming the same temperature for the earth surface.

    What else could the total greenhouse effect be than the change from no greenhouse gases to the actual greenhouse gases.

    • John Q. Lurker

      What else could the total greenhouse effect be than the change from no greenhouse gases to the actual greenhouse gases.

      That’s easy. It’s an ongoing effect. It would exist even if the earth had been brought into existence with the GHG concentration exactly as it now is, and the concentration had never changed.

  45. John Q. Lurker

    The other case was there. It was the case of atmosphere without greenhouse gases, but assuming the same temperature for the earth surface.

    Warming would occur as the GHG’s were added, and that’s the process I’m talking about. There is no warming with a given concentration of GHG’s, if equilibrium has been reached. The “proponents” I’ve been talking about imagine, perhaps unconsciously, that a given concentration of GHG’s is supposed to be an ongoing source of warming.

    What else could the total greenhouse effect be than the change from no greenhouse gases to the actual greenhouse gases.

    Off the top of my head, I’m not sure of how to answer that.

    • Well, considering that the absorption and emission from the greenhouse gases continues, how is this not the case? Essentially you are arguing that a pot of water on a stove is not warmed by the hot plate if it has reached an equilibrium temperature. You could argue that, but people would just roll their eyes, at least the polite people.

      • You are arguing semantics, all without proper physical definition. If you check dictionaries (as I have to do frequently as non-native English speaker), you would find out that the term “warming” is “often followed by up”. This is the very same meaning used by IPCC and supporting crowd of climatards, meaning that the climate “body” is “warming up”, or properly defined heat content of it’s parts is increasing, which may or may not results in increase of “global averaged temperature”. When the heat transfer regime of the pot on a stove reaches the steady state, it will not be “warming up” anymore, it will be steady, while “the emission and absorption” from whatever is around will continue as ever before. As I see, you also are in the state of perpetual confusion between the steady magnitude of greenhouse effect and the change in its magnitude from small changes in air composition.

      • Yes, people are rolling their eyes. You could look that up too.

        You might also look up that the CO2 mixing ratio has changed from about 280 ppm to 380 ppm, which is no small difference and that there are even larger percentage changes in N2O, CH4 and tropospheric ozone.

        Greenhouse gases warm the surface. Increasing concentrations of greenhouse gases will increase the warming of the surface.

      • John Q. Lurker

        Eli Rabett:
        The hot plate is analogous to the sun, not to the GHG’s. I am suggesting that people who make the 2nd Law argument are imagining that scientists claim that the GHG’s are a “hot plate”; they sing the refrain “a cooler object cannot heat a warmer object,” which is true but irrelevant; it would only be relevant if the scientists did claim that the GHG’s are a “hot plate.”

        I recognize that it may seem unbelievably odd that anyone would imagine the scientists as making that claim. Well, I don’t mean that these people explicitly and clearly imagine it. Their thinking is too muddled for that. It might be better to say that they assume it, without realizing that they are doing this, except that I think it’s somewhere near to being explicit in their thinking, without being explicit.

        When they hear that GHG’s “warm the earth,” they understand the GHG’s as supposedly being analogous to some sort of hot plate, and when they hear talk of back-radiation, they are reinforced in this idea.

        Ordinarily, one does not say that one thing “heats” another, unless the 1st thing is warmer than the other. For example, we would not say that the insulation in a house “heats” the house. (The furnace heats the house.) The people who make the 2nd Law argument are confused by statements that GHG’s “warm the earth.”

        I do not believe that many of them, if any, are conscious deceivers.

    • I hope we can all agree that usually warming means “increasing the temperature of,” as in, “I’m warming the leftovers.” But sometimes, when discussing things scientifically, warming means “contributing positiviely to an equilibrium temperature,” or “reducing the rate of cooling.” Using the word this way may not bring clarity, but it is not evidence of idiocy nor mendacity.

  46. John Q. Lurker

    Make that “source of ongoing warming.”

    • John Q. Lurker

      David N,
      I do not claim that using the word this way is evidence of idiocy or mendacity. I merely say that it, together with talk of back-radiation, has confused the people who make the argument that the 2nd Law of Thermodynamics precludes the greenhouse effect.

  47. John Q. Lurker

    Apology accepted.

  48. In the comparison I was not discussing two realistic situations, after all we have only one set of laws of nature and only one Earth. Therefore the alternative in comparison cannot be realistic in every respect.

    In the comparison the alternative was chosen to express, how the greenhouse gases influence the radiative heat flows from a surface of a fixed temperature.

    The gross radiative heat flow is the same, because the temperature of the surface is assumed to be the same. The net heat flows are very different, because there is a strong radiative energy from the atmosphere to the surface with the GHG’s but none without.

    To make the case with GHG’s equal to the real world with must add radiation form the sun to contribute to the heating of the surface and also convective and latent heat flows from the surface to atmosphere. The unrealistic alternative for comparison would have all other flows the same, but no IR radiation from the atmosphere to the surface. In that case the surface would start to cool very fast. Heating is then taken to mean the same as preventing this cooling (as the oven heats by preventing cooling of the house).

    Comparisons can be defined in many ways. This is one of the simplest ways in my thinking.

    • John Q. Lurker

      Pekka Pirilä:

      The oven heats [the house] by preventing cooling of the house, but the oven is not analogous to the GHG’s. Would you say that insulation heats the house? I wouldn’t. (I recognize that most insulation doesn’t back-radiate much. I am only using an analogy.)

      • I would extend your comment a bit, John. Passive bodies don’t (and can’t) radiate much. We get useful radiated energy when the active source is some combination of big and hot (like the sun), but nearly nothing when the passive source is small and cool (like the earth). Without the huge thermal mass of water on our planet, and the integrating effects of conduction and convection and spinning on our axis, oh boy…our world would be a much less friendly place…which we’ll rediscover when this lovely, warm interglacial period comes to an end.

      • “I recognize that most insulation doesn’t back-radiate much”

        Wrong. As soon as the layer of insulation assumes room temperature (as any good thermal insulator should), it backradiates exactly the same amount as all other walls. (The oven’s surface still emits more of course, but it’s surface is usually much smaller than the whole wall’s area).

        That’s why if you put your palm on a surface of styrofoam, you feel “warmth”. This is because your body sensors and brain processing expect certain (normal) rate of skin cooling into open air, but the styrofoam blocks the lower open-air radiation and back-radiates at the temperature of your palm, so you feel the discrepancy as “warmth”.

      • “As soon as the layer of insulation assumes room temperature (as any good thermal insulator should), it backradiates exactly the same amount as all other walls.”>/i>

        That’s incorrect. Its rate of radiation will be a function of both temperature and emissivity. If its emissivity is less than that of the other walls, it will radiate less at the same temperature. In fact, if the room were an isolated room outdoors (all sides exposed), and if one side were insulated and the opposite uninsulated, you could measure the radiation emanating out from the surface in each case, and would find less coming from the insulated side. If three sides were insulated, a very disproportionate amount of heat would escape from the uninsulated side.

        If all four sides were insulated, you would be observing something roughly equivalent to the “greenhouse effect” that CO2 exerts in our atmosphere, although the mechanism would not be exactly the same (because the walls inhibit both convective and radiative heat loss while greenhouse gases warm only via radiative effects, with convection actually playing a cooling role that partially offsets their radiative effects).

      • Sorry I mistyped the closure for the italics.

      • With insulation, reduced conduction from inside to outside is a further important means of reducing heat loss.

      • Fred wrote: “That’s incorrect. Its rate of radiation will be a function of both temperature and emissivity.”

        No kidding. Now consider that all walls are usually painted with same paint, including the insulated wall in question.

        Not sure what you are objecting to and splitting hairs. I was trying to convey that any surface “backradiates” (depending on which direction you are assuming as positive) in accord with its temperature (and emissivity, thanks), including “insulation”, or even other fuzzy surfaces made of 20km-thick layer of CO2 dissolved in O2 /N2 mixture.

      • Al – You stated that the insulated wall would necessarily backradiate “the same amount” as the other walls. That is incorrect. It would only be true if it had the same emissivity as the other walls.

      • Fred,
        You may have the right ideas in mind, but it is difficult to understand the role of reflection from your message.

        The sum of emissivity and albedo (=reflectivity) is one for a non-transparent wall. Thus the wall with a smaller emissivity reflects more of the radiation originating from other walls and also the measuring device. If they all are at the same temperature, the result of measurement is independent of the emissivity.

        With a window that transmits IR and a low temperature on the other side, the absorptivity/emissivity influences the result.

      • Yes, the sum of absorptivity, reflectivity, and transmittivity should be unity at a specified wavelength, or integrated over all wavelengths. Their overall balance would probably involve, among other things, wall thickness and conductivity, because a significant amount of incident radiation in the infrared will be dissipated by neither radiation nor reflection but by conduction. For example, a very thin wall that is opaque to visible light might possess low IR emissivity and reflectivity but high IR transmissivity.

      • Al, I know you’re educated and smart person. Very good. If I put a heater in the vacuum of space and you want to believe radiation from a passive object has a measurable influence on the heating element, I think that’s excellent. In addition, you can believe radiation from the earth increases the temperature of the sun. You can believe radiation from your body increases the temperature of your campfire on a cold night. You can believe radiation from passive atmospheric CO2 increases the earth’s surface temperature.
        I don’t necessarily disagree with the concept, but I think the effect is small, immeasurable and insignificant. I don’t care about things that cannot be measured.

      • I don’t necessarily disagree with the concept, but I think the effect is small, immeasurable and insignificant. I don’t care about things that cannot be measured.

        After a stinking hot day in Bhubaneswar the evening has cooled down to 25 °C. I leaned out the window just now (7 pm Wed) and pointed my MicroTemp MT-250 at the sky. Gosh, you were right, 0 °C. No measurable DLR (back radiation) from the sky at all.

        Just kidding. 0 °C is of course 273 K, which by Stefan-Boltzmann is 315 W/m2 of back radiation. Cloudless and not terribly humid this evening whence a fair bit of that is going to be coming from CO2. However it is also somewhat smoggy and aerosols radiate too (as a black body rather than via emission line spectra) which could account for a big chunk of the DLR.

        When I point the instrument down instead of up it registers 25 °C or 447 W/m2 of upgoing longwave radiation, ULR (not to be confused with the outgoing kind which is that leaving the top of the atmosphere).

        This demonstrates (a) that there is a net flow of heat upwards of 447 − 315 = 132 W/m2, and (b) that the flow is bidirectional since we can measure each direction separately using the instrument itself to block the other direction. The argument that observation (b) is impossible because nonzero DLR would violate the second law of thermodynamics is shot down by observation (a), that the net flow is from hot (Earth’s surface) to cold (the sky).

        While one doesn’t need a thermometer to theorize correctly about this, it helps enormously in preventing incorrect theorizing with the help of actual measurements. I would not call either of the ULR or the DLR insignificant.

      • That’s very annoying. Every time I put in a link to Amazon, WordPress automatically replaces it with an image from Amazon. This would be cute were it not for the fact that in doing so it deletes the preceding text, making the post incomprehensible. I will make a point of not putting links to Amazon in my comments on any WordPress blog in future.

        Fortunately I kept a copy. The first paragraph should have read

        After a stinking hot day in Bhubaneswar the evening has cooled down to 25 °C. I leaned out the window just now (7 pm Wed) and pointed my MicroTemp MT-250 infrared thermometer at the sky. Gosh, you were right, 0 °C. No measurable back radiation from the sky at all.

      • Well, Vaughan, this is something I’m thinking about. I have not drawn any conclusions yet…I’m just thinking out loud. You point your thermometer upward and you measure a temperature of something. What? How about the temperature of the air…which is coupled to the WV and CO2 the meter actually images…making the WV and CO2 a proxy for air temperature. I’m not arguing…if there was a blackbody in thin air, then it would have the temperature you measure in order to create the measured IR. I’m not trying to be hostile or critical, I’m just trying to figure out what is going on.
        I think the pyrgeometer is an even more interesting apparatus, but I’ll think about that later.

      • I’m not completely unsophisticated. I have a contactless IR thermometer with a focal length of about an inch. On the deck outside my office, my deck is -2C, pointed down 2 feet from my deck, I read -3.5C, pointed horizontally, I get -12C and pointed at the sky I get -26C. I know its an SBL meter, but I’m not sure how SBL translates into air temperature measurements.
        Pointed horizontally and focused on my deck rail, I get -3C. I step back a few feet and I measure -12C. What does this mean? The air temperature is not -12C, its 3C.

      • First off, if it’s a focusing type then it will behave somewhat differently from one with a specified D:S ratio. Even so, if you’re consistently getting -2 C an inch from the deck and -3.5 C two feet above it, my guess would be that your instrument is seeing the warmth of it and your hand reflected in the deck at close range (just as shining a flashlight at your deck will reflect some of it back or you wouldn’t see the glow of the beam). At longer range this contribution is diluted by being spread over more of the deck.

        The unit may conceivably be calibrated to take this reflection into account when in focus, in which case it would register lower than the correct value when the reflection is missing. You could test this by putting the unit in the freezer along with a shield (sheet of cardboard or something) to cool it down and then taking the measurements at 1″ and 2′ with the shield preventing radiation from your hand lighting up the deck. If that worked you should see less difference.

        Your rail has lower thermal mass than the deck and is therefore easier to cool if the air temperature is falling, which may be why it is a degree cooler than the deck. If the air temperature is rising the rail should be a tad warmer than the deck. When you step back from the rail you include the surrounding landscape, which apparently is somewhat colder than your house. -26 from the sky is quite typical for the ground temperatures you cite, but it’s highly variable, depending on humidity, cloudiness, extent of smog (very smoggy where I am right now in a city in India at 20.35° N 85.82 ° E, hot too), and air temperature over the whole column above.

        What is the model of your unit? Since it specifies a focal length then its optics will be giving it a negative D:S because the cone it is sensitive to will have its apex at the focal point and there will be additional optics internally to focus that apex on the thermopile. The instrument will still register at other distances, but unless the manual offers a D:S (distance-to-spot ratio) for out-of-focus conditions then you can’t tell what the unit is looking at when it’s out of focus since you can’t tell how badly out of focus it will be.

        Thermometers with no specified focal length but instead a specified D:S ratio of d:1 use different optics and accept all incoming radiation within a solid angle of π/4d² steradians, e.g. π/100 for 5:1 and π/10000 for 50:1. There is no such thing as the image being in focus in that case, everything is equally out of focus at all distances.

      • Most IR thermometers have a maximum measuring distance of approximately 100 feet (30 meters), depending on atmospheric conditions.
        http://www.coleparmer.com/techinfo/techinfo.asp?htmlfile=IRTherms_faq.htm&id=377

      • Ken,

        Vaughn’s unit is listed as having a distance to spot ratio of 5:1. Does that mean at 5″ it would read a 1″ spot? Any idea what an effective limit would be?

      • Yes, your got it, but all they care about is that the object you’re trying to measure is larger than the spot. I don’t know what the effective range is, but I have a suspicion that if there is no “object” then the range does not matter…you’re not really measuring anything and whatever the reading is…is bogus.

        Just the horizontal test of my deck rail (-3.5C) compared with the same direction pulled back a few feet (-12C) is interesting and tells you something is seriously wrong with trying to use an IR thermometer to measure air temperature.

      • The instrument is actually measuring pure energy entering it, without regard for its spectrum, which is all I want. The temperature readout is predicated on the energy being black body, but if you invert the readout (under the same assumption) to recover the energy then that’s exactly what you get: pure energy without regard for the shape of its spectrum. There does not need to be an “object” in front of it for it to measure the energy entering it.

      • The exact statement at that site is “Most IR thermometers have a maximum measuring distance of approximately 100 feet (30 meters), depending on atmospheric conditions.” I take that to mean that accuracy of measurement of the remote object falls off when the distance increases beyond 100′. The intervening atmosphere will absorb some of the radiation from the object of interest while injecting some of its own, depending on distance. Furthermore the object of interest would have to be as big as all outdoors for it to register on your instrument when a long way away, depending on D:S.

        However if you’re using the unit to measure radiation at the instrument itself, as I’m doing, then the distance and even the existence of a remote object is irrelevant. All the unit is doing is collecting and measuring whatever radiation has arrived at the unit, it doesn’t care where it came from.

      • Vaughan,
        The range of IR detected influences the range that can be obtained in surface measurements. I have understood that many devices have a cutoff around 14 um to reduce the influence of CO2 in air and to reach a longer range. The influence of CO2 is to bring the reading towards the temperature of the air.

        It appears likely that your device has a broader spectrum, but then it is less suitable for surface temperature determination from a distance. Many other manufacturers tell the spectral range but Microtemp has very little information on their devices on the net.

      • The path to the target must be unobstructed. Water vapor, dust, smoke, steam and radiation absorptive gases present in the atmosphere can attenuate emitted radiation from the target and cause the thermometer to read low.
        http://www.omega.com/literature/transactions/volume1/thermometers2.html

      • The path to the target must be unobstructed.

        Certainly. That’s a corollary of the fact that the instrument can only measure the radiation arriving at it, it can only indirectly infere what the strength of the radiation is at the target assuming no absorption in between.

      • d. It is not possible to take measurements through transparent materials (glass, plexiglas, etc.).
        e. It is not possible to measure air temperatures.
        http://www.lacrossetechnology.com/ir101/manual.pdf

      • Ken,

        another interesting comment from you last link:

        “If the emission factor for the object does not correspond to the preset value of 0.95, the temperature will not be calculated correctly. Most surfaces have an emission factor in the range of 0.8 to 0.98.”

        I think CO2 is about .4?

        Lots of problems trying to use instruments for things they aren’t designed for apparently.

      • The more interesting thing to study is the pyrgeometer…these are much more serious (and seriously expensive…your tax dollars at work) instruments. It’s fun to play with the inexpensive IR thermometers, but they are designed to measure the surface temperatures of objects. It’s quite a mystery what you’re measuring when there is no object. The comment about them being useless beyond 100 feet is interesting. If you’re measuring something colder than the sensor, then the meter is radiating outward and supposedly outgoing radiation correlates with temperature. That’s very bogus, but no one expects much from a cheap IR thermometer.
        The pyrgeometer on the other hand… This will upset the warmistas, and I have not fully thought this through, but I think the manufacturers are filling a market demand for something, anything that will give reading to justify the Trenberth/Kiehl downwelling radiation schematic. Elsewhere I called these things magical Trenberthian Dowsing Rods and I have a suspicion what I’ll be told when I suggest my IR heat lamp (calibrated to 300W/m^2) is applied to their sensor. I’m sure it will saturate bigtime and it might even damage the sensor.
        Any kind of help to figure these things out is very much appreciated.

      • Pyrgeometers can only measure the energy entering them. They are unreliable for drawing inferences about remote objects or “the sky” for the reason you give: an unknown proportion of the energy will be lost on the way to the pyrgeometer.

        It would be interesting to know what one gets from an expensive pyrgeometer that cannot be had with a $30 IR thermometer. One thing would be a USB connection allowing a computer to record a stream of readings. Anyone seen an IR thermometer with USB at a reasonable price?

      • d. It is not possible to take measurements through transparent materials (glass, plexiglas, etc.).

        Mostly true. I have a glass window and a salt window of equal size. When I point the MT-250 at a boiling saucepan (with black tape on it to increase its emissivity to near 1) through the glass window it registers only the temperature of the glass. Through the salt window it registers the temperature of the saucepan, which is some 60 °C hotter than the salt window (at least it is the same temperature to the touch as the glass window). Salt is completely transparent to IR below around 18 μ, glass only below around 2.5 μ.

        e. It is not possible to measure air temperatures

        Again mostly true. I suspect they mean air temperature at the surface. The radiation coming from above is the collective radiation from the whole column above (up to the clouds if there are any) and therefore is in some sense the average temperature of the whole column, suitably weighted according to altitude. It certainly isn’t the air temperature at any specific altitude, which can vary over 100 °C or so as a function of altitude.

      • Even more weird: my comment was restored just now without the picture from Amazon. So pay no attention to the immediately above unless you get the Amazon picture.

      • Vaughn,

        ” Cloudless and not terribly humid this evening whence a fair bit of that is going to be coming from CO2.”

        the other units this company makes are designed for relatively short range and cover 5-14um. Can’t find the same detailed specs for this cheap unit. CO2 emits primarily at 4 and 15um in our atmosphere. In other words what you are reading probably has no CO2 emissions included unless from slop. You are reading water vapor and possibly ground emissions. Any trees or buildings close to where you are using it for sky measurement?

        Have you tried to validate its accuracy? As their site claims it is for measuring the temp of surfaces at relatively close range I tend to doubt its efficacy for your use.

      • Where did you read 5 to 14 microns?

        The MT-250 is is spec’d for the temperature range −55 °C to 250 °C. Since the median energy for black body radiation at those temperatures is at respectively 18.8 μ and 7.7 μ, the MT-250 would not work very well if its thermopile did not have a pretty flat frequency response between those wavelengths.

        Now the atmosphere is not hot enough to emit any significant energy below 5 μ. Essentially all (> 99%) of the CO2 emission should be coming from the range 14-17 μ, peaking at 15.5 and representing around 300 emission lines strong enough to emit perceptibly. Hence all the CO2 radiation should be within the flat portion of the response.

        The way a thermopile works is to convert incoming energy to voltage. That part assumes nothing about whether the energy is narrow band or black body, as far as the thermopile is concerned it is just pure energy warming it up.

        The readout assumes the radiation is black body and converts power density to temperature on that basis using the Stefan-Boltzmann law. If you invert that conversion you recover the original energy, which will be correct regardless of whether it is narrow band or black body energy.

        Put another way, if a substantial portion of that energy is narrow band, the temperature readout may be misleading with regard to actual temperature, but after conversion back to energy that issue disappears. (All this assumes the instrument is set to 100E or unit emissivity.)

        The MT-250 is cheaper than the higher end units because it consists of little more than the thermopile, a microprocessor, and two little pieces of plastic, and because its distance-to-spot (D:S) ratio is 5:1 (meaning it’s reading a 1-foot diameter circle on an object 5 feet away). The high end units have D:S = 50:1 which requires 20 dB more sensitivity due to picking up only 1% of the energy the MT-250 gets. And the MT-250 in turn costs more than the MT-100, which besides only reading in Fahrenheit has a D:S of 1:1, getting another 25x more energy to make its job easier. The high end units are also much larger and more complex units.

        These considerations suggest that the MT-250 should work well at picking up CO2 emissions. One would have to repeat all this for the H2O lines, but those between 7.8 μ and 18.8 μ should register just as faithfully on the MT-250 as the CO2 lines.

      • Pratt,

        It makes no difference what you THINK it does. It is marketed as being useful to measure surface temperatures. As such the emissivity will be set to about .9. CO2 has an emissivity of about .4. Do you THINK this might make some difference to your calculations of the energy? Apparently not.

        Since it is designed to read a solid surface approximating black body from a limited distance do you THINK that trying to read a very large depth and field size MIGHT affect you computation of energy? Apparently not.

        I’m sure the built-in stop watch enhances the accuracy also. You think?

      • It makes no difference what you THINK it does.

        I agree that everyone needs to think this through for themselves and not take someone else’s word for it. However there are many articles on the web describing how IR thermometers work, and they all work by heating a thermopile, so one should not simply bury one’s head in the sand and repeat “we cannot know, we cannot know.”

        It is marketed as being useful to measure surface temperatures. As such the emissivity will be set to about .9.

        That’s the case for the MT-100 (though I would expect it to be set to .95 since the MT-250 shifts with that as the default setting). With the MT-250 the user gets to set the emissivity to anywhere between 0.01 and 1.00.

        CO2 has an emissivity of about .4. Do you THINK this might make some difference to your calculations of the energy? Apparently not.

        You’re missing the point that the instrument is measuring radiation at the point of measurement. The emissivity of CO2 only becomes relevant when you try to infer something about CO2 from the energy received at the instrument. That may be an interesting secondary goal, but my primary goal is simply to measure total DLR at the point of measurement, regardless of its source. That’s precisely what an IR thermometer (when pointed upwards outdoors) does first. It then converts that measurement to temperature using the Stefan-Boltzmann law. That conversion is easily reversed to recover the power density in W/m2 entering the instrument from a solid angle of π/100 steradians (another way of expressing D:S = 5:1 for a conical shape).

        I’m sure the built-in stop watch enhances the accuracy also. You think?

        If you’re merely looking for people to argue with then we’re done here.

      • No Vaughan,

        you are making claims using an instrument incorrectly. It is made to measure things relatively close with a solid surface. You have made no effort to find out the parameters of the unit. You do not know its rated error much less its actual error. You claim it measures based on Stefan-Boltzman apparently without understanding how it is done and then throw out claims of values.

        You also have not responded to the information our friend was nice enough to post which shows that this type of unit is NOT for measuring air.

        You apparently ignore the stated purpose the unit was marketed for. Really, would you also use an anal thermometer to measure air energy flux?

        Yes Vaughan, we are DONE.

      • Sorry I lost my cool back there. Happy to respond to substantive questions and objections.

        You claim it measures based on Stefan-Boltzman apparently without understanding how it is done.

        Are you suggesting it might be done some other way?

        You don’t have to take my word for this. The majority of articles on “how it is done” all say that the Stefan-Boltzmann fourth-power law is used. If you know of any other method of inferring temperature from IR using sub-$100 equipment you have my full attention.

        You also have not responded to the information our friend was nice enough to post which shows that this type of unit is NOT for measuring air.

        Huh? I responded in the second half of this comment, where I agreed it was not for measuring air temperature in the sense that term usually means, and moreover can’t be used to measure atmosphere temperature at any definite point. I’ve also said several times that I’m not using it to measure temperature but power density in W/m2 at the instrument. The fact that it can’t measure air temperature is irrelevant.

        Let me repeat that for emphasis and full detail. I am using the MT-250 to measure power density at the instrument in W/m2 in a solid angle of π/100.

        Every sub-$100 IR thermometer in the known universe works by measuring power density at the instrument.

        You apparently ignore the stated purpose the unit was marketed for. Really, would you also use an anal thermometer to measure air energy flux?

        Not unless it worked for that purpose.

        Googling “new-life reuse” in quotes returns 6,000 hits. New-life reuse is the reuse of something for another purpose. If you have some scientific or philosophical objection to new-life reuse it would interesting to understand what it is. If you believe there exists a sub-$100 IR thermometer on the planet that does not work by measuring power density at the instrument then let’s have it.

      • Have you tried to validate its accuracy?

        As I wrote earlier it’s on my list to calibrate it against a professional quality pyrgeometer. It will be interesting to see whether my reasoning above is borne out.

        As their site claims it is for measuring the temp of surfaces at relatively close range I tend to doubt its efficacy for your use.

        If I were trying to measure the DLR at altitude, your point would be a good one because the GHGs in between will absorb potentially a lot, and all in the case of clouds. However all I want to know is the DLR (and ULR when I’m pointing it downwards) an inch in front of the instrument, i.e. essentially at the surface of the Earth. My understanding is that the instrument is good for distances of considerably more than an inch.

        In order to measure DLR at altitude one would have to take the instrument up with a plane, balloon, kite, or whatever.

      • Ken,

        ‘I don’t care about things that cannot be measured.’

        So you’re admitting that you don’t care about the source of ‘global warming’ then, right?

        Because any source of global warming is too small to measure at this point.

        At least now I know why you don’t take these ideas very seriously. You don’t care.

        It all makes sense now…

      • I don’t grasp your point, Maxwell. Obviously, I’ve spent a lot of time thinking about this stuff, but mainly with suspicion about the motives and methods of the An Inconvenient Truth crowd. I would be a great advocate and ambassador for the cause in the engineering community if I bought the thesis and I’ve tried to have as open a mind as a stubborn old guy can have.
        Once you accept expensive central planning based on immeasurable parameters, where do you draw the line? Do you disagree there are an infinite number of theories to choose from if you lower your threshold of evidence far enough?
        Could you elaborate a bit more so I can get your point, Maxwell?

      • Ken,

        my point is that by making note of the difficulty we have measuring a signal in the temperature time series due to an increased, human-induced greenhouse effect is not juxtaposed to a larger, more important signal. That is the implication of your constant analogy to device engineering, as I pointed out in an earlier, longer comment.

        All of the signals we’re trying to find a VERY SMALL. That’s the difficulty of this particular scientific problem.

        Furthermore,

        ‘Do you disagree there are an infinite number of theories to choose from if you lower your threshold of evidence far enough?’

        yes, I disagree. There are not nearly an infinite number of theories that can explain the same level of variation in atmospheric composition, TOA radiation fluxes, temperature profiles of the different layers of the atmosphere or the globally average temperature time series. An increased greenhouse effect can at least account for some of what we’re seeing. There are ‘higher order’ terms that one might have to account for as the detection became infinite, but we’re talking about what even Pielke Sr. refers to as a ‘first order climate forcing’.

        I am also having trouble understanding the context of Al Gore or his movie in this conversation. I don’t think that he, his movie, or the people that believe every word of it number that many or are motivated as much as you think.

        We just had a Democratically controlled Congress, a Democratic President and an oil spill the size of 100 Exxon Valdezes and still there was no climate legislation. So I don’t know what ‘centralized planning’ you are referring to.

        But even if I did or there were such plans, constructing oversimplified analogies that use science to achieve your particular political goals are just as mistaken and fallacious as those used by Al Gore and his lot.

        Do you really want to be in the same boat that badly?

      • John Q. Lurker

        If I’m wrong (i.e., if insulation does “back-radiate much”), this only makes the analogy better.

  49. John Q. Lurker

    Pekka Pirilä:
    I am not claiming that “GHG’s warm the earth” is easily avoidable. I am saying that it confuses the people who make the 2nd Law argument.

    • John,
      People who even start to make the 2nd law argument are at stage already so confused that it is very difficult to help. The whole idea is so far from correct physical thinking that I cannot know, how to get them out of this wrong track.

      When a totally wrong argument is presented, i.e. an argument that starts with a erroneous assumption, it is difficult to tell anything else than that the initial assumption is totally unjustified and not based on the 2nd law.

  50. Okay, I have too much time on my hands.

    http://ourhydrogeneconomy.blogspot.com/2011/02/greenhouse-gases-and-climate.html

    My traffic cop analogy :)

  51. To Claes J., Ken, Will, J.Q.Lurker, etc, regarding “backradation” and alleged absence thereof. Here is a challenge for you folks:

    Suppose you have a long room with two flat black walls, each having a certain temperature, T1 and T2, of about the same level. It does not matter who and what “heats” these walls; let’s consider only some short time interval when the walls keep these temperatures relatively steady.

    Say, you are standing near T1 wall; it is warm, and therefore emits the radiation energy flux I1=eps*sig*T1^4. If a surface has a temperature, it emits. Would you agree with that?

    Now, you step to the opposite wall; it is also warm, and from your new position, also emits something, I2=eps*sig*T2^4. Now take a glance at the opposite wall that you left just a moment ago. Did it stop emitting all that spectrum of electromagnetic waves just because you walked to opposite wall? Heck no. Assuming you have radiometers of various kinds, will you still be able to register the directional flux of electromagnetic waves in IR range from that distant wall? Certainly you will. Is the other wall radiates at you? Remember, it does not matter how the other wall has acquired its energy, through electric heater or green-cheese laser; as long as this energy gets converted into certain temperature, the surface will emit. Therefore, it is an obvious absurd to say that this [back]radiation does not exist. QED.

    • Al, I don’t care what Will says about you, I think you’re a good man. We all agree, if there is a delta-T, then we get radiation. It gets interesting when the walls are the same temperature and you’re moving your IR radiometer around the room. I think we don’t really have a good visualization of the radiated energy…I wonder if it might be better to consider the radiation as lines of flux created between things with different temperatures. But, never mind that now.

      Can we move the room into space so we’re sure to be talking only about radiation? Now, the walls have the same temperature, so there’s no delta-T and we agree they don’t influence each other. Now, let’s introduce Al’s cadaver between the walls. He’s stone-cold expired. The question is, is there any possible way Al’s attractive, well-dressed, passive body floating between the walls can make either of the walls measurably warmer?

    • Isn’t it strange that physics books and physicists don’t give an answer to this elementary question? What are physicists payed for if they have no answer, to this elementary question?

      • Any book that discusses radiative heat transfer has all information needed for that. They cannot help the fact that there are some people who do not believe these descriptions.

        Science of Doom has even collected copies of relevant pages from several widely used test books. Anyone interested can certainly find that article.

    • John Q. Lurker

      Al Tekhassski:

      I do not claim that back-radiation does not exist. I claim that the people who think that the 2nd Law precludes the greenhouse effect have been confused by talk about it. They think it has been invented in order to argue, in effect, that a cooler object can heat a warmer object. I believe that their trouble begins with the concept of heating or warming.

      In the everyday idea of heating, heating as by a warmer object dominates. We may say that a blanket “warms up” a person, but a person with little scientific education, asked to examine this statement, may say that actually it does not; the person is the source of the warming. I contend that this is legitimate. For example, it serves to distinguish an ordinary blanket from an electric blanket.

      It is even more legitimate than that. The earth only warms up while the GHG concentration is being increased. This isn’t quite true; it neglects the time required to reach equilibrium after the increase ceases. But it’s close enough. It’s conceptually valid. From that standpoint, it is illegitimate to say that GHG’s warm up the earth, because a fixed concentration of them would not do so.

      Dr. Curry is interested in getting AGW skeptics who accept the 2nd Law argument to “get past” it. I say that this can only be accomplished, if at all, by understanding their reasoning and taking it into account in talking with them. If you or others think it is impossible to get them past it, maybe you are right, but you should be telling this to Dr. Curry.

      I am a person with some talent for understanding the reasoning of people who are mistaken or slow- witted. In school, I irritated my teachers with this. I’d be waving my hand to get the teacher’s attention, and saying, “I can explain what he’s not getting!” Here in this thread, I’m trying to use that talent on the problem of “getting them past” the 2nd Law argument.

      • The concept of “backradiation” has been invented to support CO2 alarmism, by seeking to give the false impression of massive radiative exchange of heat energy between the atmosphere and the Earth surface of the size of 300 W/m2, while the net exchange is only about 60 W/m2.

        The number 300 instead of 60 opens to alarm by saying: Take 1% of 300 W/m2 (a small perturbation) to get “radiative forcing” of 3 W/m2 and plug that into Stefan-Boltzmann to get a global warming of 1 C, and then inflate it to 3 C by invented feedbacks = ALARM!

        But the 300 is fiction, only 60 is real and 1% of 60 is 0.6 and global warming of 0.2 C = NO ALARM! even with 3-fold feedbacks.

        So the fictitious invented “backradiation” of 300 W/m2 is the cornerstone of CO2 alarmism. If this corner stone is removed, as is done in the Sky Dragon book, the alarmism crumbles.

  52. Al, if wall A is now heating wall B so that its temperature rises and in turn it B increases its radiant enegy back at wall A; then wall A must further warmed by the radiation it originally sent to wall B, but is now back at Wall A adding even more to its temperature so that……..er……..you’ve just discovered perpetual energy! Johann Bessler eat your heat out! Al’s shown us why we need never have to pay for central heating ever again! Thanks, Al.

    • John, did it ever occur to you that there might be some difference between “radiance flux heating something at constant rate” and “heating up” of that something? Could you please try to be a bit more humble and at least try conceal your ignorance?

      • John Q. Lurker

        Al, did it ever occur to you that you might not be understanding me?

      • John Q. Lurker

        I thought that one-liner message was directed to me, not John O’Sullivan. I apologize for that, but not for my longer message to you just now.

      • John Q. Lurker

        And sorry for calling it a “one-liner.” My message was the one-liner, but I wasn’t thinking straight.

      • Al, if my ‘ignorance’ is in question then yours can be in no doubt. Just two words utterly destroy your bogus thought experiment: Poynting’s law.
        To elaborate, when calculating any transport of electro-magnetic energy (radiation) all such flow is to be determined by field vector calculus (i.e. Poynting’s vector (see Poynting’s theorem) which is an energy conservation law: see John David Jackson (1998). Classical electrodynamics (Third ed.). New York: Wiley. ISBN 047130932X. http://worldcat.org/isbn/047130932X).
        Vectoring laws specifically require that when the direction of such energy flow is equal and in opposite motion ( as per your “long room with two flat black walls, each having a certain temperature, T1 and T2, of about the same level” ) all such flows must be summed to zero. This is a law of science and not an option! Thus, just like conduction and convection radiation can have no BACK FLOW.

      • John,
        That statement is strongly – very strongly – misleading.

        It is true that there is only one value of electric and magnetic fields at every point. Therefore the local transfer of energy occurs at every moment to one specific direction. When the radiation is described by photons, the direction is, however, not constant. Sometimes it is up, sometimes down. The net value is the average of these fluctuating values. The physicists of 19th century accepted after a while the notion of Boltzmann, that the laws of thermodynamic are true as averages, not at every moment.

        With quantum mechanics, we have additional space for interpretation, but that does not change the conclusion. There is nothing wrong with the description that includes backradiation.

      • Al, to further educate the ‘ignorant’ amongst us just ponder this: there is radiation, and then there is radiative HEAT transfer. Radiation can be regarded as an EM field that surrounds objects at any temperature. But HEAT is only transferred when one body’s temperature is higher than another’s. Distance is also a factor, of course, due to the inverse square of light propagation: The sun, for instance, may be 5775 K, but its radiation cannot heat the earth to that temperature. As Alan Siddons points out – there’s no evidence that a cold body’s coldness “seeps in” to a warmer body via conductive heat transfer, there’s no evidence that a cold body’s radiation heats a warmer body either. Rather, the direction of heat transfer is one-way alone, from warm to cool. This “net transfer” and “flux” business is just verbalistic camouflage.
        The premise of greenhouse theory is that two equal and opposing flows of energy ADD (rather than cancel), such that 240 W/m² going up and 240 W/m² going down will generate 480 W/m² (but only on the emitting surface!). Since such a notion is childish and reprehensible, the same must be said for “climate science” too. Our civilization’s present climate alarm is based on a false and stupid premise, embarrassing as that will prove to be. Future generations will laugh at us – so please wise up.

      • John,
        The childish notion is a straw man. It is not, how physicists think about it. It is invented to confuse real understanding.

      • What drivel and such a non-statement in response to a hard, cold scientific law that you cannot refute; all such motion of energy in three-dimensions MUST be addressed by vector calculus and when such energy flows in equal strength, in opposite directions, Poyntings law says the sum must be zero.
        We GHE ‘Slayers’ don’t care that a few warmist physicists don’t like to “think about it” because it refutes their ‘back radiation’ fallacy. The truth is that you, Curry, Spencer, a corrupted NASA et al. failed to fit the greenhouse gas theory into the laws of physics and thus it is junk science. QED.

      • John,
        My previous message refuted your previous message and refutes also your later one – whether you understand my refutation or not.

        Even if being skeptical of the final conclusions is understandable, you are certainly wrong, when you start to contradict well established physical knowledge. You are also certainly wrong, when use these erroneous arguments against climate science.

      • John, sorry, nobody but your band of coauthors for sky dragon book is buying this stuff, at least among the crowd who knows anything about physics. Even self-described “denier” physicist Tomas Milanovic says this is bunk.

      • Sorry, Judy, lots of people are buying the book and it’s message that “backradiation” is unphysical , and without “backradiation”
        CO2 alarmism radiates away into empty science.

      • Sorry, Judy, Pekka. But your hand waving appeals to a discredited ‘authority’ just don’t cut it in REAL science. I’ll say again, all such motion of energy in three-dimensions MUST be addressed by vector calculus and when such energy flows in equal strength, in opposite directions, Poyntings law says the sum must be zero.
        You’re arguing from ignorance, we’re sticking to the LAWS of science. The empirical evidence also proves you wrong-there is no detectable GHE in the atmosphere. The jig is up. QED.

      • Sorry John, the jig that is up is any real scientific physics-based credibility for the Sky Dragon and the arguments presented therein.

      • Judy, Claes Johnson has exposed the fact that you did not read the book thus your assessment is invalid. In the book, just as above, we see how GHE believers cannot address the vital premise that all such motion of energy in three-dimensions MUST be addressed by vector calculus and when such energy flows in equal strength, in opposite directions, Poyntings law says the sum must be zero.
        That you decline to now address this inescapable vectoring law exposes the weakness of your whole ‘back radiation’ argument.

      • John, get over it. I read the book. My assessment agrees with everyone else here who knows physics. You completely misunderstand Poynting’s theorem.

      • Judy, for some reason your reply (below) to my comment doesn’t appear to show a ‘reply’ button for me to refute your erroneous suggestion that I misunderstand Poynting.
        Please be so kind as to explain your interpretation of how Poynting should be applied here.

      • John, that would take to long, I have other things on my plate that I regard as higher priority. Let me recommend Goody and Yungs text Atmospheric Radiation: Theoretical Basis

      • Dr. Curry,

        $185.00??? I wont’ be reading that anytime soon if it isn’t in the local library, which I doubt.

      • Judy, finding the time to read so many books is, I agree, incredibly time consuming for us all. Another example supportive of my argument is the section on heat transfer by radiation in Perry’s Chemical Engineering Handbook which was written by Hottel. Plus the rest of the chapter on Heat and Mass transfer, and the chapters on thermodynamics and fluid dynamics, but I would imagine the maths (in particular the dimensionless numbers such as Reynolds, Nusselt and Prandtl)

  53. And here, my friends (and that includes you, Al , Pekka and Maxwell), we find joyous beauty in Madame Curry’s noisy blog. Right here, the Warmistas and Skeptics can grab what they believe and boil it down to their 20-second elevator pitch for Joe Sixpack. Then, JS will have all the information he needs to decide for himself whether to rip up the death train (coal train) railways, de-commission coal-fired power plants, drive an expensive plastic hybrid car, spend hard-earned money on wind and solar energy projects, support candidates promoting carbon cap and trade schemes, allow bureaucrats to classify CO2 as a pollutant and regulate-to-death every industry they can point their accusing collectivist fingers at, and whether to fund the grants and paychecks of unrepentant Climategate players with names like Schmidt, Santer, Jones, Hansen, Mann and Trenberth.
    And there, Judith Curry, you earn the Etc. in the title of your blog. Very good.

  54. Ken Coffman interviews Claes Johnson pursuant to the discussion at Climate Etc.

    http://www.slayingtheskydragon.com/latest-news/104-professor-claes-johnson-dissects-the-failed-greenhouse-gas-theory

    CJ’s concluding statement:

    CJ: My experience is that academia, including its scientific (royal) societies, has given up independent thinking and is controlled by climate politics of governments and governmental institutions. Even worse, alarmism “skepticism” is actively being oppressed and harassed at universities. My scientific work of analyzing the mathematical models of climate has been censored at my university The Royal Institute of Technology KTH , followed up by a media campaign to discredit my work and reputation (described on my blog as KTH-gate ).

    No one at KTH raised a voice against this censorship and oppression, not even the single other skeptic professor at KTH, evidently because of fear of harassment.

    I criticized The Royal Swedish Academy of Sciences for uncritically supporting IPCC (also on my blog ) without any scientific analysis of its own, to no effect.

    My direct experience of academia is thus frightening. It appears that critical independent thinking is no longer allowed nor desired—as if 1984 with its new-speak of doublethink and thinkpol is already here.

    • I think that this needs some clarification.
      Claes Johnsons work has not been censored, as a matter of fact it is still available from his homepage at KTH. What has happened though is that KTH does not want to use it as a course book for undergrad studies, which is something completely else. It is also not very controversial as every teacher and institution must be free to choose which books to use in their courses. ( And I can´t really blame them for that decision. It is hard to justify using a book that that goes against most of the physics developed since 1905.)

      • The “clarification” by Reason is a misrepresentation of facts and events which I have described in detail on my blog under “KTH-gate”. In short, what happened was that my text was censored at KTH by KTH followed by a media campaign from KTH to discredit my scientific work and my person voiced by the President of KTH, all along without any contact with me whatsoever. I have asked the Faculty of KTH to investigate these even in Swedish academia extraordinary events, but the Faculty has not responded, not even acknowledged receipt of my request.

        It seems that Reason speaks on behalf of KTH (the President?), and supplies the motivation for the censorship that my work “goes against most of the physics developed since 1905”. The claim appears to be that
        my work represents some form of “entartete kunst” which has to be burned in public. I ask the censors why my math formulas are so dangerous that they have to be forbidden? I think Reason knows the reason.

      • I can not see that your work is “censored” in any usual meaning of the word – it is still available from your official homepage at the KTH web. Which I think it should be.

        That it is not usable as a text for undergrad studies is something completely else. I do not “censor” authors when I choose to switch textbook in one of the courses I teach. And it would require something extraordinary for me to use a book that deny the existence of the photon, more than 100 years after it was shown to exist.

      • Reason,

        you claim photons were shown to exist more than 100 years ago. Why then do people like John Gribbin claim that quantum physics proves the non-existence of particles more than their existence?? In the book In Search of Schrodinger’s Cat, he leaves one with the distinct impression that quantum mechanics means there are no particles.

      • This question really must go to John Gribbin.
        (I have not read this book, but that the photon is one of the elementary particles is at the foundation of modern physics, including quantum physics. So it would surprise me if he actually argues that it does not exist as such.)

      • Reason,

        yes, the idea that a photon is a particle is the basis for much confusion in modern physics. The two hole experiment shows why this is. In the book it is explained on pages 163-171. If solid particles were going through the 2 holes from a single source you would get a simple additive pattern on a solid surface decreasing smoothly from the peaks inline with the holes. (apparently Feynman liked to use a machine gun as an example) With electrons and photons you get an interference pattern, as we have alway seen from experiments with waves in liquid and experiments with light.

        Photons and electrons move in probability patterns that do NOT follow simple vectors as we are used to seeing with solid objects. The wavefronts “collapse” to a particle when interacting with another particle making it seem as if they were particles all along EXCEPT they do not follow the simple vectors of solid objects, they follow the vectors of waves and wave interactions.

        Due to this the idea that any particular photon from CO2 seemingly directed at the earth HAS to impact the earth, to me, simply isn’t necessarily a high probability when there are so many other photons of the same wavelength which are all probability wave fronts interacting to confuse the issue.

        This is why it is more reasonable to talk about NET energy flows as there really isn’t a way for us to compute what is really going where and whether the interaction is a quantum change in an atom on the surface, a molecular bond energy change, or a wave interaction before reaching the surface.

        I am continually amazed that hard, proven, physics is simply ignored to reach the required result. Reflection, absorption, emission, scatter, cancellation.

        I keep hearing that the photon can’t know the temperature of the material that it is interacting with, yet, the photon or electron in the 2 hole experiment, as described in the book, KNOWS THE COMPLETE EXPERIMENTAL SETUP!!!!! Yeah, I am still flabbergasted, but, it is apparently something that has been repeated so many times there is no doubt of the validity. We also see the recent experiments with paired electrons where one knows the state of the other at distance. WTF?!?!?!?!?! Yet the photon should have a problem KNOWING the temperature of what it is approaching?? It may very well NOT KNOW, but, how the heck do WE know it does or doesn’t!! Of course, approaching a material with a higher temperature there will be more probability waves (photons) with a possibility of interacting with the photon from the CO2. Why is it so hard to see that some of this interaction will collapse some of the wavefronts from the CO2 without it reaching the surface??

        Quantum mechanics is all about statistics. Wouldn’t this be the statistics of how many wavefronts interact also??

        This would appear to me why the best way currently to look at the issue is simply NET FLOW!!

        Obviously I am babbling and will hopefully be corrected by those knowledgeable. This is just what I understand from reading the book and other disjointed exposure.

        From page 174:
        “The physics is impossible, buth the math is clean and simple, familiar equations to any physicist. As long as you avoid asking what it means, there are no problems. Ask why the world should be like this , however, and even Feynman has to reply, “We have no idea.” Persist in asking for a physical picture of what is going on, and you find all physical pictures dissolving into a world of ghosts, where particles only seem to be real when we are looking at them, and where even a property such as momentum or position is only an artifact of the observation.”

        Here is an interesting presentation I found on this issue which also includes some discussion fo the 2 hole experiment:
        http://www.dipankarhome.com/What%20is%20light.pdf

        The basic issue for me is that until we understand the nature better we cannot ignore the fact that wave mechanics work and particle mechanics work. Trying to ignore either until we clearly understand how to differentiate would seem to be foolish.

      • I can only recommend you to take the step up from the popular science books to the actual textbooks used in academia! You will find them much more rewarding, clearer to understand and that they will answer a lot of your questions in a much better way than I can ever do here in a blog comment. (Although they will require some more work to go through than the pop.sci. book, but there are, regrettably, no such thing as a free lunch.)

      • Kuhnkat,

        All attempts to explain quantum mechanics with classical concepts fail at some level, because quantum mechanics is different. One cannot understand quantum mechanics from classical concepts, but what means “understand” anyway. Some things are so familiar that we think that we understand them. The new features of quantum mechanics are not familiar, as they are not observable without special equipment.

        Some consequences of quantum mechanics influence us all the time, but they are things that we do not particularly try to understand, we just notice that things are like that.

        What quantum mechanics is cannot be described without mathematics, because everyday observations have not led to the development of the necessary concepts and language. Physicists have developed the mathematical description and the talk about it using some words, but they do not have clear meaning without the mathematical formulas.

        Popular science books may try to explain quantum mechanics, but they cannot really do it for the above reasons. Some other writings may take the other extreme and claim that quantum mechanism is not true as much as classical physics is, but that is not true.

        If you want to learn what quantum mechanics is and why there is very strong reasons to believe that it is a faultless theory of very many physical phenomena, essentially of all physics, where relativity is not important, and in its relativistic form also much of the remaining physics, there are no other choices than learning the theory of university textbooks. Due to the new concepts and the type of mathematics involved learning quantum mechanics is a major task for everyone, and many physics students fail in learning it well, but it is not impossible.

        There are still open issues in the philosophical interpretation of quantum mechanics and some physicists think that there might be some new physics behind these problems. That does not change at all the fact, that quantum mechanics with all its strange features has been one of the most successful physical theories, if not the most successful of them all. It provides extremely precise predictions for a very wide variety of problems, and where it does not provide predictions, it is because the calculation become too difficult. Even in these cases there are no signs that quantum mechanics would be wrong, except that there are sometimes problems when relativistic effects start to be important. The elementary particle physicists try to develop the theory to handle also situations, which exceed the limits of validity of the present theories. When and how they succeed remains to be seen.

        (I have given several courses of quantum mechanics, both basic and more advanced, in my youth and my first field of research was theoretical elementary particle physics. That phase of my carrier ended in 1980.)

      • Pekka,

        None of what you say conflicts with the 2 hole experiment, in fact it comes from this experiment. Why do you try to imply that I have stated something wrong? My conclusions are occasionally wrong, but that is what I need correction on, not vague, you can only do it with quantum mechanics.

        Where does quantum mechanics state there is NOT a probability front that collapses into a particle at a particular point or interaction?? Where does it say the 2 hole type experiment does NOT show interference patterns when the photons are not observed? This Popular Science book suggests that even electrons behave like this. Is it wrong??

        Like with other issues, repeating calls to authority without pointing out which items are wrong and which authority claims the correct item or understanding is pointless repetition.

      • How can I guess, what you really expect?

        I wrote a comment after reading your earlier comment. Can’t you see anything which leads thoughts to a different direction from, what I wrote.

        Of course various interference experiments are in full agreement with QM, both when they are done with photons and when they are done with particles like electrons.

        The state collapse in QM is not precisely the same thing than a collapse of a probability front. When the observation is the determination of the position of a particle, then the wave function collapses to one, where the particle is at the position of observation, but that is not necessarily all of it. The wave function may for example describe also another particle with a correlated position, or something else depending on the total settings of the experiment.

        The 2-hole experiment shows interference pattern, when the pattern is observed. When a system to observe is set up the interference pattern will form, but its status before it is really observed belongs to the philosophical difficulties of interpreting QM.

        I am not calling to authority. Telling my background was meant to help in interpreting my writing. I could have been a terrible lecturer, who does not know what he is talking about, and even if I knew 40 years ago, I might have lost the understanding since. The only authority I accept in web discussions is the one that is earned in those discussions.

        What I was really saying, was that understanding QM requires a lot of work and learning from university textbooks. Everything based on popular descriptions or net discussions remains unsatisfactory due to the special nature of QM. From those sources one may pick some correct bits and pieces, but certainly also very misleading thoughts. There is so much that one must unlearn from the traditional thinking that it does not succeed without the possibility of replacing it with the full mathematics based theory.

  55. Well the Sky Dragon book is still available outside KTH, but it is banned on KTH by KTH. Of course people are free to chose text books, but that is never
    accompanied by media campaigns telling that the university had to stop the use of a certain book against the will of the teachers giving the course, as in
    present case. Why does Reason have to defend this, unless Reason = KTH?

    As concerns photons I don’t prevent anybody from believing they exist if they
    want. I give my arguments, that is all, others can give theirs.

    • I thought it was you who had forbidden KTH to use the book ? Because you wrote in your blog: “Continued use of the book at KTH without my consent will be a violation of my copyright.” As far as I understand it, the teachers wanted to continue use selected parts of the book, which you did not allow. (Yes, I am swedish, so I can read all of your blog. But I am not at KTH, so your insinuations are misplaced.)

      And I hope that you agree that unsubstantiated arguments does not really make good textbooks for undergrad students?

  56. When KTH said that certain parts of the book were “unacceptable” and had to be “torn out”, I responded that I did not accept that my work was cannibalized,
    to no effect of course. What “unsubstantiated arguments” does my text contain?

  57. One example is your claims regarding the photon, which are given without any references to support them. Claim such as: “To observe individual photons as “particles” without both mass and charge seems impossible, and so the physical reality of photons has re- mained hypothetical with the main purpose of explaining black-body ra- diation and the photoelectric effect.”

  58. What are the observations of individual photons?

    • That was not the question. The question is where your source is for writing that the main purpose of the defining the photon is to explain black-body and the photo-electric effect. (Where the latter is an obvious example of observation of individual photons. Which I have no illusions that you will accept as such.)

      Another example is where you postulate that there is a smallest increment of time, i.e that time itself is quantified. I fail to find any arguments of why this should be, or references to any experiments showing that this is the case.

      • You raise basic questions of physics for which I do not claim that I have the final answers, nor see that anybody else have. I don’t understand why you are defending censorship of mathematical models and analysis at a Swedish university today.

      • As I said, you make unsubstantiated arguments, which makes the text, IMHO, unsuitable for use in undergrad education.

        And your use of the word “censorship” does not seem to coincide with any commonly used definition of the term. Nobody has stopped you from distributing the text from your homepage at the KTH system except you yourself. Who also have forbidden KTH to use it in any way, shape or form, according to your own blog.

  59. To Reason: If you revealed your identity I would be able to understand why you defend censorship in Sweden today.

    • Much more interesting than my name is your avoidance of answering direct questions about your theories, both in this thread and in part I of this blog post.

  60. Juidth, If you’re still interested, I have read the skydragon threads having come across it late. I see no substantial reason not to believe ClaesJ (and G&T) that “backradiation” is unphysical. The negative critques did not hold argument, just as the published commentary on the G&T paper did not stand up to scrutiny in my view.

    I suspect the problems in understanding come from climate science’s assumption that infrared spectra equate to measurements of total heat energy transmission and climate “outsiders” view that there has been way too much politicking and bias in climate science.

    That Claes has published in a a book is a manifestation only that we stand on the brink of politicians committing billions of dollars towards “tilting at windmills” trying to manipulate climate, and not that what he says could be sensible.

    I’ll be much happier when climate scientists can give a clear exposition of how negative feedback control systems deliver stable results in the face of perturbations and not runaway scenarios shown by the computer models which are unable to demonstrate any stabilty.

    • BLouis79,

      I have very much in doubt about the “Back Radiation” as you are and as well as so many skeptics including Claes. But the warmists/AGWers side have pyrgeometer measurements as their weapon to measure the “Back Radiation” . The AGWers consider these pyrgeometer as solid evidence of back radiation. I have never used a pyrgeometer before and don’t know how true it is to measure “Back Radiation” unless I found the methodology of using that pyrgeometer to derive those back radiation data. The biggest doubt I have on this pyrgeometer is “Is it truly measuring back radiation or something else like ambient air temperature of the pyrgeometer.

      I think all skeptics should look at these pyrgeometers to debunk those data. Even though the pyrgeometer is truly measuring “Back Radiation”, it is very much in doubt that CO2 can contribute to 32W/m2 as claimed by K&T’s 1997 Global Annual Mean Radiation Budget.

    • BLouis, you must have missed Tomas Milanovic’s devastating critique of Johnson’s argument.

      • I didn’t find Tomas Milanovic’s critique devastating, since Claes’ responses seemed just as credible, and the maths of both is a bit complex for me. Bottom line is that the unanswered problem is of the relative contributions of conduction, convection and radiation in free gas molecules and the “unphysicality” of net transfer of heat from a cooler to warmer object by radiation.

        The argument does not seem a lot different from measuring eddy currents in electrical circuits or the reactive force applied by a floor to a chair – phenomena that are measurable and have not much real effect in terms of ability to do work.

  61. BLouis, Dr Curry,

    I found the biggest problem with the radiation budget is why all solar radiation absorption by clouds and atmosphere are radiated right back to space and not radiated all directions. If they also radiated all directions, I believe they are, then the pyrgeometers are measuring more than just “Back Radiation”. So any modeling results and conclusions based on “Back Radiation” measurements are flawed. Correct me if I am wrong.